Инфоурок Математика Другие методич. материалыМетодические указания для студентов по проведению практических занятий по дисциплине ЕН.01. «Математика» для специальности 230701 «Прикладная информатика (по отраслям)» базовой подготовки

Методические указания для студентов по проведению практических занятий по дисциплине ЕН.01. «Математика» для специальности 230701 «Прикладная информатика (по отраслям)» базовой подготовки

Скачать материал

Министерство образования Нижегородской области

Государственное бюджетное образовательное учреждение

Среднего профессионального образования

«Пильнинский агропромышленный техникум»

 

УТВЕРЖДАЮ:

Директор ГБОУ СПО  «Пильнинский агропромышленный техникум»

 

__________ М.А.Беспалов

 

 

«     »                     20          г.

 

 
 

 

 

 

 

 

 


Методические указания для студентов

по проведению практических занятий

по дисциплине ЕН.01.  «Математика»

для специальности

230701 «Прикладная информатика (по отраслям)»

базовой      подготовки

 

 

 

 

 

 

р.п.Пильна, 2014 год

 УТВЕРЖДЕНА

Методическим советом

Протокол №____

от «_____»___________20__г

 

Председатель

_____________/Т.И.Бабичева /

 

 

 

 

 

 

 

 

Председатель

_____________/Т.И.Афонина/

 
РАССМОТРЕНА

ПЦК дисциплин

 общеобразователного

цикла

Протокол №___

от «____»_______20___г

Председатель

_____________/Г.Г. Агафонова/

 

 

 

 

Автор-составитель: Грищенко З.Н., преподаватель математики ГБОУ СПО «Пильнинский агропромышленный техникум»

 

 

 

 

 

 

 

 

 

 

 

 

 

Содержание

1.      Пояснительная записка…………………………………………………………4

 

2.      Практическая работа №1 "Выполнение действий над матрицами"……………5

 

3.      Практическая работа №2 "Решение систем линейных уравнений" ……………13

  1. Практическая работа №3"Решение задач с применением

различных видов уравнений прямой на плоскости" …………………………………….17

  1. Практическая работа №4" Решение задач с применением уравнений кривых второго порядка на плоскости" ……………………………………………………………..24
  1. Практическая работа №5 "Вычисление пределов числовых последовательностей"………………………………………......................................29

 

  1. Практическая работа №6 " Исследование функций на непрерывность. Вычисление пределов"…………………………………………………………………………….33

 

  1. Практическая работа №7 "Нахождение производных функций" ………………..42
  1. Практическая работа №8 "Нахождение производных второго порядка"……….44

 

  1. Практическая работа №9 "Геометрический смысл производной»………………46

 

  1. Практическая работа №10 "Исследование функций и построение графиков"…..52

12    Практическая работа №11"Интегрирование простейших функций"……………..54

13    Практическая работа №12"Вычисление простейших определённых интегралов"…63

14    Практическая работа №13  "Решение прикладных задач с помощью определённого интеграла"……………………………………………………………………………..65

15    Практическая работа №14"Исследование функций двух переменных"…………..68

16    Практическая работа №15"Решение дифференциальных уравнений первого порядка"………………………………………………………………………………..75

17    Практическая работа №16"Решение дифференциальных уравнений второго

порядка"………………………………………………………………………………..85

18    Практическая работа №17"Определение сходимости числовых рядов"………….93

19    Практическая работа №18"Исследование на  сходимость степенных рядов"…...102

20    Практическая работа №19"Разложение функций в  степенные ряды"……………107

21    Практическая работа №20"Решение задач с применением формул алгебры логики"…………………………………………………………………………………112

22    Практическая работа №21"Погрешности простейших арифметических действий"……………………………………………………………………………....117

23    Практическая работа №22"Численное решение алгебраических уравнений"…………………………………………………………………………….120

24    Практическая работа №23"Решение простейших задач на определение вероятности с использованием теоремы сложения вероятностей"………………………………..127

25    Практическая работа №24"Построение закона распределения дискретной случайной величины по заданному условию"………………………………………………….133

26    Практическая работа №25"Нахождение математического ожидания и дисперсии дискретной случайной величины, заданной законом распределения"……………137

27    Литература……………………………………………………………………………..

 

 

 

 

 

ПОЯСНИТЕЛЬНАЯ ЗАПИСКА

Сборник практических работ служит для организации практических занятий по математике в объеме 50 часов. Данное пособие предназначено  для студентов 2 курса специальности «Прикладная информатика (по отраслям)» и разработано в соответствии с рабочей программой по математике.

        Целью практических занятий является формирование учебных практических умений по математике и содействие оптимальному освоению  студентами учебного материала. Выполнение студентами практических работ направлено на обобщение, систематизацию, углубление, закрепление полученных знаний по конкретным темам, формирование умений применять полученные знания на практике, формирование профессионально значимых качеств таких,  как самостоятельность, ответственность, точность.

        В сборнике содержится 25 практических работ. Практическое занятие проводится в учебной аудитории, продолжительность занятия – 2 часа. Оценки за выполнение практических работ выставляются по пятибалльной системе.  Практические работы  выполняются  в специально заведённых для практических работ тетрадях.

        Методические  указания предназначены для оказания помощи студентам при выполнении практических работ по дисциплине «Математика»

Методические  указания состоят из двадцати пяти разделов, расположенных в порядке проведения практических занятий, в каждом из которых изложены цели занятий, основные знания и умения, краткие теоретические сведения по соответствующей теме, приводятся примеры решения задач и даются задания для студентов.

В результате проведения практических занятий по дисциплине студент должен:

уметь:

  • уметь выполнять операции над матрицами и решать системы линейных уравнений;
  • уметь применять методы дифференциального и интегрального исчисления;
  • уметь решать дифференциальные уравнения:
  • уметь применять основные положения теории вероятностей и математической статистики в профессиональной деятельности;

знать:

  • иметь представление о роли и месте математики в современном мире, общности её понятий и представлений;
  • основы линейной алгебры и аналитической геометрии;
  • основные понятия и методы дифференциального и интегрального исчисления;
  • основные численные методы решения математических задач;
  • решение прикладных задач в области профессиональной деятельности.

 

Практическая работа №1 «Выполнение действий над матрицами»

Цель работы: закрепление практических навыков выполнения действий с матрицами.

Ход работы:

1)повторение теоретического материала;

2)выполнение заданий;

3)вывод.

1.Краткое содержание теоретического материала.

Матрица – это прямоугольная таблица каких-либо элементов. В качестве элементов мы будем рассматривать числа, то есть числовые матрицы. 

 Обозначение: матрицы обычно обозначают прописными латинскими буквами http://mathprofi.ru/f/deistviya_s_matricami_clip_image002.gif

Пример: рассмотрим матрицу «два на три»:

http://mathprofi.ru/f/deistviya_s_matricami_clip_image004.gif

Когда говорят о размерах матрицы, то сначала указывают количество строк, а только потом – количество столбцов. В рассмотренном примере матрица «два на три».

Если количество строк и столбцов матрицы совпадает, то матрицу называют квадратной, например: http://mathprofi.ru/f/deistviya_s_matricami_clip_image014.gif – матрица «три на три».

 

Умножение матрицы на число.

Пример:

http://mathprofi.ru/f/deistviya_s_matricami_clip_image034.gif

Для того чтобы умножить матрицу на число, нужно каждый элемент матрицы умножить на данное число. В данном случае – на тройку.

Транспонирование матрицы.

Для того чтобы транспонировать матрицу, нужно ее строки записать в столбцы транспонированной матрицы.

Пошаговый пример:

Транспонировать матрицу http://mathprofi.ru/f/deistviya_s_matricami_clip_image014_0001.gif

Сначала переписываем первую строку в первый столбец:

http://mathprofi.ru/f/deistviya_s_matricami_clip_image053.jpg

Потом переписываем вторую строку во второй столбец:
http://mathprofi.ru/f/deistviya_s_matricami_clip_image055.jpg

И, наконец, переписываем третью строку в третий столбец:

http://mathprofi.ru/f/deistviya_s_matricami_clip_image057.jpg

 Грубо говоря, транспонировать – это значит повернуть матрицу набок.

Сумма (разность) матриц.

 Для выполнения сложения (вычитания) матриц, необходимо, чтобы они были одинаковыми по размеру.

Пример:

Сложить матрицы http://mathprofi.ru/f/deistviya_s_matricami_clip_image061.gif и http://mathprofi.ru/f/deistviya_s_matricami_clip_image063.gif

Для того чтобы сложить матрицы, необходимо сложить их соответствующие элементы:

http://mathprofi.ru/f/deistviya_s_matricami_clip_image065.gif

Для разности матриц правило аналогичное, необходимо найти разность соответствующих элементов.

Пример:

Найти разность матриц http://mathprofi.ru/f/deistviya_s_matricami_clip_image004_0000.gif, http://mathprofi.ru/f/deistviya_s_matricami_clip_image067.gif

http://mathprofi.ru/f/deistviya_s_matricami_clip_image069.gif

Умножение матриц.

Чтобы матрицу  http://mathprofi.ru/f/deistviya_s_matricami_clip_image022_0000.gif можно было умножить на матрицу http://mathprofi.ru/f/deistviya_s_matricami_clip_image076.gif нужно, чтобы число столбцов матрицы http://mathprofi.ru/f/deistviya_s_matricami_clip_image022_0001.gif равнялось числу строк матрицы http://mathprofi.ru/f/deistviya_s_matricami_clip_image076_0000.gif.

Пример:

Умножить матрицу http://mathprofi.ru/f/deistviya_s_matricami_clip_image102.gif на матрицу http://mathprofi.ru/f/deistviya_s_matricami_clip_image082_0000.gif
Применим формулу :

http://mathprofi.ru/f/deistviya_s_matricami_clip_image105.gif .

http://mathprofi.ru/f/deistviya_s_matricami_clip_image107.gif

Пример:

Умножить матрицу http://mathprofi.ru/f/deistviya_s_matricami_clip_image094_0000.gif на матрицу http://mathprofi.ru/f/deistviya_s_matricami_clip_image096_0000.gif

Формула: http://mathprofi.ru/f/deistviya_s_matricami_clip_image109.gif

http://mathprofi.ru/f/deistviya_s_matricami_clip_image111.gif

В результате получена так называемая нулевая матрица.

Переходим к матрицам третьего порядка:

Умножить матрицу http://mathprofi.ru/f/deistviya_s_matricami_clip_image122.gif на матрицу http://mathprofi.ru/f/deistviya_s_matricami_clip_image124.gif

Формула очень похожа на предыдущие формулы:
http://mathprofi.ru/f/deistviya_s_matricami_clip_image126.gif

http://mathprofi.ru/f/deistviya_s_matricami_clip_image128.gif

 

Числоhttp://function-x.ru/chapter1/determinants_clip_image002.gif            (1)        называют определителем матрицы размерностью n строк и n столбцов.

 

Определитель второго порядка есть число, получаемое следующим образом:

http://function-x.ru/chapter1/determinants_clip_image008.gif(2)

Определитель третьего порядка – это число, получаемое так:

http://function-x.ru/chapter1/determinants_clip_image018.gif(3)

Запомнить эту формулу трудно. Однако существует простое правило, называемое правилом треугольников, которое позволяет легко воспроизвести выражение (3). Обозначая элементы определителя точками, соединим отрезками прямой те из них, которые дают произведения элементов определителя (рис. 1).

http://function-x.ru/image/pic_1.jpg

Формула (3) показывает, что со своими знаками берутся произведения элементов главной диагонали, а также элементов, расположенных в вершинах двух треугольников, основания которых ей параллельны; с противоположными – произведения элементов побочной диагонали, а также элементов, расположенных в вершинах двух треугольников, которые ей параллельны.

Пример . Вычислить определитель третьего порядка:

http://function-x.ru/chapter1/determinants_clip_image020.gif

Решение. Пользуясь правилом треугольников, получим

http://function-x.ru/chapter1/determinants_clip_image022.gif
http://function-x.ru/chapter1/determinants_clip_image024.gif

Определитель можно вычислить способом разложения по элементам первой строки. Этот способ будет рассмотрен в следующем примере.

Пример:

Найти обратную матрицу для матрицы http://mathprofi.ru/f/kak_naiti_obratnuyu_matricu_clip_image061.gif

Обратную матрицу найдем по формуле: http://mathprofi.ru/f/kak_naiti_obratnuyu_matricu_clip_image063.gif, где http://mathprofi.ru/f/kak_naiti_obratnuyu_matricu_clip_image065.gif – транспонированная матрица алгебраических дополнений соответствующих элементов матрицы http://mathprofi.ru/f/kak_naiti_obratnuyu_matricu_clip_image067.gif.

1) Находим определитель матрицы.

http://mathprofi.ru/f/kak_naiti_obratnuyu_matricu_clip_image069.gif
Здесь определитель вычислен способом разложения по элементам  первой строки.

Также не забываем, что http://mathprofi.ru/f/kak_naiti_obratnuyu_matricu_clip_image071.gif, а значит, обратная матрица существует.

2) Находим матрицу миноров http://mathprofi.ru/f/kak_naiti_obratnuyu_matricu_clip_image021_0000.gif.

Матрица миноров имеет размерность «три на три» http://mathprofi.ru/f/kak_naiti_obratnuyu_matricu_clip_image073.gif, и нам нужно найти девять чисел.

 Подробно рассмотрим парочку миноров:

Рассмотрим следующий элемент матрицы:
http://mathprofi.ru/f/kak_naiti_obratnuyu_matricu_clip_image075.jpg
МЫСЛЕННО вычеркиваем строку и столбец, в котором находится данный элемент:
http://mathprofi.ru/f/kak_naiti_obratnuyu_matricu_clip_image077.jpg

Оставшиеся четыре числа записываем в определитель «два на два»
http://mathprofi.ru/f/kak_naiti_obratnuyu_matricu_clip_image079.jpg
Этот определитель «два на два» и является минором данного элемента. Его нужно вычислить:
http://mathprofi.ru/f/kak_naiti_obratnuyu_matricu_clip_image081.jpg
Всё, минор найден, записываем его в нашу матрицу миноров:
http://mathprofi.ru/f/kak_naiti_obratnuyu_matricu_clip_image083.jpg

Как вы, наверное, догадались, необходимо вычислить девять определителей «два на два». Ну и для закрепления – нахождение еще одного минора в картинках:
http://mathprofi.ru/f/kak_naiti_obratnuyu_matricu_clip_image085.jpg
Остальные миноры попробуйте вычислить самостоятельно.

Окончательный результат:
http://mathprofi.ru/f/kak_naiti_obratnuyu_matricu_clip_image087.gif – матрица миноров соответствующих элементов матрицы http://mathprofi.ru/f/kak_naiti_obratnuyu_matricu_clip_image067_0000.gif.

То, что все миноры получились отрицательными – чистая случайность.

3) Находим матрицу алгебраических дополнений http://mathprofi.ru/f/kak_naiti_obratnuyu_matricu_clip_image090.gif.

В матрице миноров необходимо СМЕНИТЬ ЗНАКИ строго у следующих элементов:
http://mathprofi.ru/f/kak_naiti_obratnuyu_matricu_clip_image092.jpg
В данном случае:
http://mathprofi.ru/f/kak_naiti_obratnuyu_matricu_clip_image094.gif – матрица алгебраических дополнений соответствующих элементов матрицы http://mathprofi.ru/f/kak_naiti_obratnuyu_matricu_clip_image067_0001.gif.

4) Находим транспонированную матрицу алгебраических дополнений http://mathprofi.ru/f/kak_naiti_obratnuyu_matricu_clip_image065_0000.gif.

http://mathprofi.ru/f/kak_naiti_obratnuyu_matricu_clip_image098.gif – транспонированная матрица алгебраических дополнений соответствующих элементов матрицы http://mathprofi.ru/f/kak_naiti_obratnuyu_matricu_clip_image067_0002.gif.

5) Ответ:

http://mathprofi.ru/f/kak_naiti_obratnuyu_matricu_clip_image100.gif

Проверка: 
http://mathprofi.ru/f/kak_naiti_obratnuyu_matricu_clip_image102.gif

Таким образом, обратная матрица найдена правильно.

 

2.Самостоятельное выполнение задания.

Даны две матрицы А и В

1 вариант

А=, В=

2 вариант

А=, В=

Найти: 1) А+2В, 2) АВ, 3) А-1 с проверкой.

 

3.Вывод практического занятия.

 

 

 

 

 

 

 

 

Практическая работа №2 «Решение систем линейных уравнений»

Цель работы: закрепление практических навыков решения систем трёх линейных уравнений с тремя неизвестными.

Ход работы:

1)повторение теоретического материала;

2)выполнение заданий;

3)вывод.

1.Краткое содержание теоретического материала.

ПРАВИЛО КРАМЕРА

Рассмотрим систему 3-х линейных уравнений с тремя неизвестными:

http://www.toehelp.ru/theory/math/lecture14/l14image012.gif

Определитель третьего порядка, соответствующий матрице системы, т.е. составленный из коэффициентов при неизвестных,

http://www.toehelp.ru/theory/math/lecture14/l14image048.gif

называется определителем системы.

Составим ещё три определителя следующим образом: заменим в определителе D последовательно 1, 2 и 3 столбцы столбцом свободных членов

http://www.toehelp.ru/theory/math/lecture14/l14image050.gif

Тогда можно доказать следующий результат.

Правило Крамера. Если определитель системы Δ ≠ 0, то рассматриваемая система имеет одно и только одно решение, причём

http://www.toehelp.ru/theory/math/lecture14/l14image052.gif

Пример. Решить систему уравнений методом Крамера

  1. http://www.toehelp.ru/theory/math/lecture14/l14image074.gif

Итак, х=1, у=2, z=3.

МЕТОД ГАУССА

Ранее рассмотренные методы можно применять при решении только тех систем, в которых число уравнений совпадает с числом неизвестных, причём определитель системы должен быть отличен от нуля. Метод Гаусса является более универсальным и пригоден для систем с любым числом уравнений. Он заключается в последовательном исключении неизвестных из уравнений системы.

Вновь рассмотрим систему из трёх уравнений с тремя неизвестными:

http://www.toehelp.ru/theory/math/lecture14/l14image090.gif.

Первое уравнение оставим без изменения, а из 2-го и 3-го исключим слагаемые, содержащие x1. Для этого второе уравнение разделим на а21 и умножим на –а11, а затем сложим с 1-ым уравнением. Аналогично третье уравнение разделим на а31 и умножим на –а11, а затем сложим с первым. В результате исходная система примет вид:

http://www.toehelp.ru/theory/math/lecture14/l14image096.gif

Теперь из последнего уравнения исключим слагаемое, содержащее x2. Для этого третье уравнение разделим на http://www.toehelp.ru/theory/math/lecture14/l14image098.gif, умножим наhttp://www.toehelp.ru/theory/math/lecture14/l14image100.gif и сложим со вторым. Тогда будем иметь систему уравнений:

http://www.toehelp.ru/theory/math/lecture14/l14image102.gif

Отсюда из последнего уравнения легко найти x3, затем из 2-го уравнения x2 и, наконец, из 1-го – x1.

При использовании метода Гаусса уравнения при необходимости можно менять местами.

Часто вместо того, чтобы писать новую систему уравнений, ограничиваются тем, что выписывают расширенную матрицу системы:

http://www.toehelp.ru/theory/math/lecture14/l14image104.gif

и затем приводят её к треугольному или диагональному виду с помощью элементарных преобразований.

К элементарным преобразованиям матрицы относятся следующие преобразования:

  1. перестановка строк или столбцов;
  2. умножение строки на число, отличное от нуля;
  3. прибавление к одной строке другие строки.

Пример: Решить системы уравнений методом Гаусса.

  1. http://www.toehelp.ru/theory/math/lecture14/l14image106.gif

Вернувшись к системе уравнений, будем иметь

http://www.toehelp.ru/theory/math/lecture14/l14image108.gif

  1. http://www.toehelp.ru/theory/math/lecture14/l14image110.gif

Выпишем расширенную матрицу системы и сведем ее к треугольному виду.

http://www.toehelp.ru/theory/math/lecture14/l14image112.gif

Вернувшись к системе уравнений, несложно заметить, что третье уравнения системы будет ложным, а значит, система решений не имеет.

  1. http://www.toehelp.ru/theory/math/lecture14/l14image114.gif

Разделим вторую строку матрицы на 2 и поменяем местами первый и третий столбики. Тогда первый столбец будет соответствовать коэффициентам при неизвестной z, а третий – при x.

http://www.toehelp.ru/theory/math/lecture14/l14image116.gif

Вернемся к системе уравнений. http://www.toehelp.ru/theory/math/lecture14/l14image118.gif

Из третьего уравнения выразим одну неизвестную через другую и подставим в первое.

http://www.toehelp.ru/theory/math/lecture14/l14image120.gif

Таким образом, система имеет бесконечное множество решений.

2.Решите самостоятельно систему методом :а) Крамера; б) Гаусса:

 

Вариант 1     а)

       б)

Вариант 2      а)

        б)

 

3.Вывод практического занятия.

 

 

 

 

 

 

 

Практическая работа  №3 «Решение задач с применением различных видов уравнений прямой на плоскости»

Цель работы: закрепление практических навыков решения задач на применение различных видов уравнений прямой на плоскости.

Ход работы:

1)повторение теоретического материала;

2)выполнение заданий;

3)вывод.

1.Краткое содержание теоретического материала.

Определение. Любая прямая на плоскости может быть задана уравнением первого порядка

Ах + Ву + С = 0,

причем постоянные А, В не равны нулю одновременно. Это уравнение первого порядка называют общим уравнением прямой. В зависимости от значений постоянных А,В и С возможны следующие частные случаи:

•  C = 0, А ≠0, В ≠ 0 – прямая проходит через начало координат

•  А = 0, В ≠0, С ≠0 { By + C = 0}- прямая параллельна оси Ох

•  В = 0, А ≠0, С ≠ 0 { Ax + C = 0} – прямая параллельна оси Оу

•  В = С = 0, А ≠0 – прямая совпадает с осью Оу

•  А = С = 0, В ≠0 – прямая совпадает с осью Ох

Уравнение прямой может быть представлено в различном виде в зависимости от каких – либо заданных начальных условий.

Уравнение прямой по точке и вектору нормали

Определение. В декартовой прямоугольной системе координат вектор с компонентами (А, В) перпендикулярен прямой , заданной уравнением Ах + Ву + С = 0.

Пример. Найти уравнение прямой, проходящей через точку А(1, 2) перпендикулярно вектору http://www.mathelp.spb.ru/book1/line_%20on_%20plane.files/Eqn001.gif(3, -1).

Решение. Составим при А = 3 и В = -1 уравнение прямой: 3х – у + С = 0. Для нахождения коэффициента С подставим в полученное выражение координаты заданной точки А. Получаем: 3 – 2 + C = 0, следовательно С = -1. Итого: искомое уравнение: 3х – у – 1 = 0.

Уравнение прямой, проходящей через две точки

Пусть в пространстве заданы две точки M 1 ( x 1 , y 1 , z 1 ) и M2 ( x 2, y 2 , z 2 ), тогда уравнение прямой, проходящей через эти точки:

http://www.mathelp.spb.ru/book1/line_%20on_%20plane.files/Eqn002.gif

Если какой- либо из знаменателей равен нулю, следует приравнять нулю соответствующий числитель.На плоскости записанное выше уравнение прямой упрощается:

http://www.mathelp.spb.ru/book1/line_%20on_%20plane.files/Eqn003.gif

если х 1 ≠ х2 и х = х 1 , если х 1 = х2 .

Дробь http://www.mathelp.spb.ru/book1/line_%20on_%20plane.files/Eqn004.gif= k называется угловым коэффициентом прямой.

Пример. Найти уравнение прямой, проходящей через точки А(1, 2) и В(3, 4).

Решение. Применяя записанную выше формулу, получаем:

http://www.mathelp.spb.ru/book1/line_%20on_%20plane.files/Eqn005.gif

Уравнение прямой по точке и угловому коэффициенту

Если общее уравнение прямой Ах + Ву + С = 0 привести к виду:

http://www.mathelp.spb.ru/book1/line_%20on_%20plane.files/Eqn006.gif

и обозначить http://www.mathelp.spb.ru/book1/line_%20on_%20plane.files/Eqn007.gif, то полученное уравнение называется уравнением прямой с угловым коэффициентом k .

Уравнение прямой по точке и направляющему вектору

По аналогии с пунктом, рассматривающим уравнение прямой через вектор нормали можно ввести задание прямой через точку и направляющий вектор прямой.

Определение. Каждый ненулевой вектор http://www.mathelp.spb.ru/book1/line_%20on_%20plane.files/Eqn008.gif( α1 , α2 ), компоненты которого удовлетворяют условию А α1 + В α2 = 0 называется направляющим вектором прямой

Ах + Ву + С = 0.

Пример. Найти уравнение прямой с направляющим вектором http://www.mathelp.spb.ru/book1/line_%20on_%20plane.files/Eqn009.gif(1, -1) и проходящей через точку А(1, 2).

Решение. Уравнение искомой прямой будем искать в виде: Ax + By + C = 0. В соответствии с определением, коэффициенты должны удовлетворять условиям:

1 * A + (-1) * B = 0, т.е. А = В.

Тогда уравнение прямой имеет вид: Ax + Ay + C = 0, или x + y + C / A = 0. при х = 1, у = 2 получаем С/ A = -3, т.е. искомое уравнение:

х + у - 3 = 0

Уравнение прямой в отрезках

Если в общем уравнении прямой Ах + Ву + С = 0 С≠0, то, разделив на –С, получим: http://www.mathelp.spb.ru/book1/line_%20on_%20plane.files/Eqn010.gif или

http://www.mathelp.spb.ru/book1/line_%20on_%20plane.files/Eqn011.gif, где

http://www.mathelp.spb.ru/book1/line_%20on_%20plane.files/Eqn012.gif

Геометрический смысл коэффициентов в том, что коэффициент а является координатой точки пересечения прямой с осью Ох, а b – координатой точки пересечения прямой с осью Оу.

Пример. Задано общее уравнение прямой х – у + 1 = 0. Найти уравнение этой прямой в отрезках.

 

С = 1, http://www.mathelp.spb.ru/book1/line_%20on_%20plane.files/Eqn013.gif, а = -1, b = 1.

Нормальное уравнение прямой

Если обе части уравнения Ах + Ву + С = 0 разделить на число http://www.mathelp.spb.ru/book1/line_%20on_%20plane.files/Eqn014.gif, которое называется нормирующем множителем , то получим

 

xcosφ + ysinφ - p = 0 –

нормальное уравнение прямой. Знак ± нормирующего множителя надо выбирать так, чтобы μ * С < 0. р – длина перпендикуляра, опущенного из начала координат на прямую, а φ - угол, образованный этим перпендикуляром с положительным направлением оси Ох.

Пример. Дано общее уравнение прямой 12х – 5у – 65 = 0. Требуется написать различные типы уравнений этой прямой.

уравнение этой прямой в отрезках: http://www.mathelp.spb.ru/book1/line_%20on_%20plane.files/Eqn015.gif

уравнение этой прямой с угловым коэффициентом: (делим на 5)

http://www.mathelp.spb.ru/book1/line_%20on_%20plane.files/Eqn016.gif

нормальное уравнение прямой:

 

http://www.mathelp.spb.ru/book1/line_%20on_%20plane.files/Eqn017.gif; cos φ = 12/13; sin φ= -5/13; p = 5.

C ледует отметить, что не каждую прямую можно представить уравнением в отрезках, например, прямые, параллельные осям или проходящие через начало координат.

Пример. Прямая отсекает на координатных осях равные положительные отрезки. Составить уравнение прямой, если площадь треугольника, образованного этими отрезками равна 8 см 2 .

Решение. Уравнение прямой имеет вид: http://www.mathelp.spb.ru/book1/line_%20on_%20plane.files/Eqn018.gif, ab /2 = 8; a = 4; -4. a = -4 не подходит по условию задачи. Итого: http://www.mathelp.spb.ru/book1/line_%20on_%20plane.files/Eqn019.gif или х + у – 4 = 0.

Пример. Составить уравнение прямой, проходящей через точку А(-2, -3) и начало координат.

Решение. Уравнение прямой имеет вид: http://www.mathelp.spb.ru/book1/line_%20on_%20plane.files/Eqn020.gif, где х 1 = у 1 = 0; x2 = -2; y2 = -3.http://www.mathelp.spb.ru/book1/line_%20on_%20plane.files/Eqn021.gif

 

Угол между прямыми на плоскости

Определение. Если заданы две прямые y = k1 x + b1 , y = k 2x + b2 , то острый угол между этими прямыми будет определяться как

 

http://www.mathelp.spb.ru/book1/line_%20on_%20plane.files/Eqn022.gif.

Две прямые параллельны, если k1 = k2 . Две прямые перпендикулярны, если k1 = -1/ k2 .

Теорема. Прямые Ах + Ву + С = 0 и А 1 х + В1 у + С1 = 0 параллельны, когда пропорциональны коэффициенты А1 = λА, В1 = λВ. Если еще и С1 = λС, то прямые совпадают. Координаты точки пересечения двух прямых находятся как решение системы уравнений этих прямых.

 

Уравнение прямой, проходящей через данную точку

перпендикулярно данной прямой

Определение. Прямая, проходящая через точку М1 1 , у1 ) и перпендикулярная к прямой у = kx + b представляется уравнением:

http://www.mathelp.spb.ru/book1/line_%20on_%20plane.files/Eqn023.gif

 

Расстояние от точки до прямой

 

Теорема. Если задана точка М(х0 , у0 ), то расстояние до прямой Ах + Ву + С =0 определяется как

http://www.mathelp.spb.ru/book1/line_%20on_%20plane.files/Eqn024.gif.

Доказательство. Пусть точка М 1 1, у 1) – основание перпендикуляра, опущенного из точки М на заданную прямую. Тогда расстояние между точками М и М1 :

http://www.mathelp.spb.ru/book1/line_%20on_%20plane.files/Eqn025.gif (1)

Координаты x1 и у1 могут быть найдены как решение системы уравнений:

http://www.mathelp.spb.ru/book1/line_%20on_%20plane.files/Eqn026.gif

Второе уравнение системы – это уравнение прямой, проходящей через заданную точку М 0 перпендикулярно заданной прямой. Если преобразовать первое уравнение системы к виду:

A(x – x 0 ) + B(y – y0 ) + Ax0 + By0 + C = 0,

то, решая, получим:

http://www.mathelp.spb.ru/book1/line_%20on_%20plane.files/Eqn027.gif

Подставляя эти выражения в уравнение (1), находим:

http://www.mathelp.spb.ru/book1/line_%20on_%20plane.files/Eqn028.gif

Теорема доказана.

 

Пример. Определить угол между прямыми: y = -3 x + 7; y = 2 x + 1.

k 1 = -3; k 2 = 2; tgφ = http://www.mathelp.spb.ru/book1/line_%20on_%20plane.files/Eqn029.gif; φ= p /4.

Пример. Показать, что прямые 3х – 5у + 7 = 0 и 10х + 6у – 3 = 0 перпендикулярны.

Решение. Находим: k 1 = 3/5, k2 = -5/3, k 1* k 2 = -1, следовательно, прямые перпендикулярны.

Пример. Даны вершины треугольника А(0; 1), B (6; 5), C (12; -1). Найти уравнение высоты, проведенной из вершины С.

Решение. Находим уравнение стороны АВ: http://www.mathelp.spb.ru/book1/line_%20on_%20plane.files/Eqn030.gif; 4 x = 6 y – 6;

2 x – 3 y + 3 = 0; http://www.mathelp.spb.ru/book1/line_%20on_%20plane.files/Eqn031.gif

Искомое уравнение высоты имеет вид: Ax + By + C = 0 или y = kx + b . k = http://www.mathelp.spb.ru/book1/line_%20on_%20plane.files/Eqn032.gif. Тогда y = http://www.mathelp.spb.ru/book1/line_%20on_%20plane.files/Eqn033.gif. Т.к. высота проходит через точку С, то ее координаты удовлетворяют данному уравнению: http://www.mathelp.spb.ru/book1/line_%20on_%20plane.files/Eqn034.gifоткуда b = 17. Итого: .http://www.mathelp.spb.ru/book1/line_%20on_%20plane.files/Eqn035.gif

Ответ: 3 x + 2 y – 34 = 0.

 

2.Самостоятельное выполнение задания.

1 вариант.

Даны вершины треугольника А(0;7), В(6;-1) и С(2;1).Найти уравнения сторон, медианы АМ, высоты ВD, а так же уравнение прямой АК, параллельной ВС. Найти углы треугольника и угол между высотой ВD и медианой АМ.

2 вариант.

Даны вершины треугольника А(0;2), В(4;0) и С(1;-2).Найти уравнения сторон, медианы СМ, высоты АD, а так же уравнение прямой l, параллельной ВС, проходящей через точку А. Найти углы треугольника и угол между высотой АD и медианой СМ.

4.      Вывод практического занятия.

 

 

 

Практическая работа №4  «Решение задач с применением уравнений кривых второго порядка на плоскости»

Цель работы: закрепление практических навыков решения задач на применение уравнений кривых второго порядка на плоскости.

Ход работы:

1)повторение теоретического материала;

2)выполнение заданий;

3)вывод.

1.Краткое содержание теоретического материала.

КРИВЫЕ ВТОРОГО ПОРЯДКА

 

Общий вид линии второго порядка:

 http://edu.dvgups.ru/METDOC/ENF/VMATEM/WM/METOD/KONTR_MATEM/Pay_2.files/image002.gif.                                                (1)

К кривым второго порядка относятся: окружность, эллипс, гипербола, парабола.

 

1.  Окружность 

Окружность – это множество точек плоскости, равноудаленных от данной точки (центра).

 http://edu.dvgups.ru/METDOC/ENF/VMATEM/WM/METOD/KONTR_MATEM/Pay_2.files/image004.gif                                                        (2)

где http://edu.dvgups.ru/METDOC/ENF/VMATEM/WM/METOD/KONTR_MATEM/Pay_2.files/image006.gif- радиус окружности, http://edu.dvgups.ru/METDOC/ENF/VMATEM/WM/METOD/KONTR_MATEM/Pay_2.files/image008.gif и http://edu.dvgups.ru/METDOC/ENF/VMATEM/WM/METOD/KONTR_MATEM/Pay_2.files/image010.gif - координаты центра окружности.

Если центр окружности совпадает с началом координат, то уравнение имеет вид

 http://edu.dvgups.ru/METDOC/ENF/VMATEM/WM/METOD/KONTR_MATEM/Pay_2.files/image012.gif                                                                 (3)

http://edu.dvgups.ru/METDOC/ENF/VMATEM/WM/METOD/KONTR_MATEM/Pay_2.files/image013.jpg

Рис. 2

2.    Эллипс

Эллипсом называется множество точек плоскости, сумма расстояний которых до двух данных точек, называемых фокусами, есть величина постоянная (бóльшая, чем расстояние между фокусами).

Каноническое (простейшее) уравнение эллипса с центром в начале координат и с фокусами в точках http://edu.dvgups.ru/METDOC/ENF/VMATEM/WM/METOD/KONTR_MATEM/Pay_2.files/image015.gif и http://edu.dvgups.ru/METDOC/ENF/VMATEM/WM/METOD/KONTR_MATEM/Pay_2.files/image017.gif:

 http://edu.dvgups.ru/METDOC/ENF/VMATEM/WM/METOD/KONTR_MATEM/Pay_2.files/image019.gif                                                                          (4)

где http://edu.dvgups.ru/METDOC/ENF/VMATEM/WM/METOD/KONTR_MATEM/Pay_2.files/image008.gif и http://edu.dvgups.ru/METDOC/ENF/VMATEM/WM/METOD/KONTR_MATEM/Pay_2.files/image010.gif - полуоси эллипса, с – полуфокусное расстояние. Коэффициенты http://edu.dvgups.ru/METDOC/ENF/VMATEM/WM/METOD/KONTR_MATEM/Pay_2.files/image021.gif эллипса связаны соотношением http://edu.dvgups.ru/METDOC/ENF/VMATEM/WM/METOD/KONTR_MATEM/Pay_2.files/image023.gif

http://edu.dvgups.ru/METDOC/ENF/VMATEM/WM/METOD/KONTR_MATEM/Pay_2.files/image024.jpg

Рис. 3

 

Если центр эллипса находится в точке http://edu.dvgups.ru/METDOC/ENF/VMATEM/WM/METOD/KONTR_MATEM/Pay_2.files/image026.gif, то уравнение эллипса имеет вид:

  http://edu.dvgups.ru/METDOC/ENF/VMATEM/WM/METOD/KONTR_MATEM/Pay_2.files/image028.gif                                                               (5)

 

3.    Гипербола

Гиперболой называется множество точек плоскости, абсолютная величина разности расстояний которых до двух данных точек, называемых фокусами, есть величина постоянная, меньшая, чем расстояние между фокусами.

Уравнение гиперболы с центром в начале координат и с фокусами в точках http://edu.dvgups.ru/METDOC/ENF/VMATEM/WM/METOD/KONTR_MATEM/Pay_2.files/image015.gif и http://edu.dvgups.ru/METDOC/ENF/VMATEM/WM/METOD/KONTR_MATEM/Pay_2.files/image017.gif имеет вид:

  http://edu.dvgups.ru/METDOC/ENF/VMATEM/WM/METOD/KONTR_MATEM/Pay_2.files/image032.gif                                                                          (6)

где http://edu.dvgups.ru/METDOC/ENF/VMATEM/WM/METOD/KONTR_MATEM/Pay_2.files/image008.gif - действительная полуось,

       http://edu.dvgups.ru/METDOC/ENF/VMATEM/WM/METOD/KONTR_MATEM/Pay_2.files/image010.gif - мнимая полуось.

 Коэффициенты http://edu.dvgups.ru/METDOC/ENF/VMATEM/WM/METOD/KONTR_MATEM/Pay_2.files/image036.gif и http://edu.dvgups.ru/METDOC/ENF/VMATEM/WM/METOD/KONTR_MATEM/Pay_2.files/image038.gif гиперболы связаны соотношением  http://edu.dvgups.ru/METDOC/ENF/VMATEM/WM/METOD/KONTR_MATEM/Pay_2.files/image040.gif.

       Прямые http://edu.dvgups.ru/METDOC/ENF/VMATEM/WM/METOD/KONTR_MATEM/Pay_2.files/image042.gif- асимптоты гиперболы.

http://edu.dvgups.ru/METDOC/ENF/VMATEM/WM/METOD/KONTR_MATEM/Pay_2.files/image043.jpg

Рис. 4

 

Если центр гиперболы находится в точке http://edu.dvgups.ru/METDOC/ENF/VMATEM/WM/METOD/KONTR_MATEM/Pay_2.files/image026.gif, то уравнение имеет вид:

 http://edu.dvgups.ru/METDOC/ENF/VMATEM/WM/METOD/KONTR_MATEM/Pay_2.files/image046.gif                                                            (7)

4.    Парабола

Параболой называется множество точек плоскости, равноудаленных от точки, называемой фокусом и прямой, называемой директрисой.

Уравнение параболы с вершиной в начале координат имеет вид:

                                                        http://edu.dvgups.ru/METDOC/ENF/VMATEM/WM/METOD/KONTR_MATEM/Pay_2.files/image048.gif,                                                                                    (8)

где http://edu.dvgups.ru/METDOC/ENF/VMATEM/WM/METOD/KONTR_MATEM/Pay_2.files/image050.gif - расстояние между фокусом параболы и прямой линией, называемой директрисой. Фокус параболы имеет координаты http://edu.dvgups.ru/METDOC/ENF/VMATEM/WM/METOD/KONTR_MATEM/Pay_2.files/image052.gif.

http://edu.dvgups.ru/METDOC/ENF/VMATEM/WM/METOD/KONTR_MATEM/Pay_2.files/image053.jpg

Рис. 5

 

Если вершина параболы находится в точке http://edu.dvgups.ru/METDOC/ENF/VMATEM/WM/METOD/KONTR_MATEM/Pay_2.files/image026.gif, то уравнение имеет вид:

                                             http://edu.dvgups.ru/METDOC/ENF/VMATEM/WM/METOD/KONTR_MATEM/Pay_2.files/image056.gif                                                                      (9)

Задача 1.  Составить уравнение геометрического места точек, равноотстоящего от оси Оу и точки http://edu.dvgups.ru/METDOC/ENF/VMATEM/WM/METOD/KONTR_MATEM/Pay_2.files/image058.gif.

Решение:  Возьмем на искомой линии произвольную точку http://edu.dvgups.ru/METDOC/ENF/VMATEM/WM/METOD/KONTR_MATEM/Pay_2.files/image060.gif. Расстояние точки М от точки F определится по формуле расстояния между двумя точками:

http://edu.dvgups.ru/METDOC/ENF/VMATEM/WM/METOD/KONTR_MATEM/Pay_2.files/image062.gif

Расстояние точки М до оси Оу определится:

http://edu.dvgups.ru/METDOC/ENF/VMATEM/WM/METOD/KONTR_MATEM/Pay_2.files/image064.gif

Так как по условию http://edu.dvgups.ru/METDOC/ENF/VMATEM/WM/METOD/KONTR_MATEM/Pay_2.files/image066.gif, то искомая кривая имеет уравнение:

http://edu.dvgups.ru/METDOC/ENF/VMATEM/WM/METOD/KONTR_MATEM/Pay_2.files/image068.gif        http://edu.dvgups.ru/METDOC/ENF/VMATEM/WM/METOD/KONTR_MATEM/Pay_2.files/image070.gif

http://edu.dvgups.ru/METDOC/ENF/VMATEM/WM/METOD/KONTR_MATEM/Pay_2.files/image072.gif

Линия, определяемая полученным уравнением http://edu.dvgups.ru/METDOC/ENF/VMATEM/WM/METOD/KONTR_MATEM/Pay_2.files/image074.gif является параболой.

http://edu.dvgups.ru/METDOC/ENF/VMATEM/WM/METOD/KONTR_MATEM/Pay_2.files/image075.jpg

Задача 2.  Составить уравнение геометрического места точек, отношение  расстояний которых до точки F(-1; 0) и до прямой х = -9 равно 1/3.

Решение:  Возьмём на искомой кривой произвольную точку http://edu.dvgups.ru/METDOC/ENF/VMATEM/WM/METOD/KONTR_MATEM/Pay_2.files/image060.gif
Её расстояния от точки http://edu.dvgups.ru/METDOC/ENF/VMATEM/WM/METOD/KONTR_MATEM/Pay_2.files/image078.gif и прямой составляют   http://edu.dvgups.ru/METDOC/ENF/VMATEM/WM/METOD/KONTR_MATEM/Pay_2.files/image080.gif http://edu.dvgups.ru/METDOC/ENF/VMATEM/WM/METOD/KONTR_MATEM/Pay_2.files/image082.gif

Из условия задачи следует:

http://edu.dvgups.ru/METDOC/ENF/VMATEM/WM/METOD/KONTR_MATEM/Pay_2.files/image084.gif

Таким образом, искомая кривая имеет уравнение:

http://edu.dvgups.ru/METDOC/ENF/VMATEM/WM/METOD/KONTR_MATEM/Pay_2.files/image086.gif

Приведём это уравнение к каноническому виду:

http://edu.dvgups.ru/METDOC/ENF/VMATEM/WM/METOD/KONTR_MATEM/Pay_2.files/image088.gif - это уравнение эллипса с полуосями: http://edu.dvgups.ru/METDOC/ENF/VMATEM/WM/METOD/KONTR_MATEM/Pay_2.files/image090.gif

http://edu.dvgups.ru/METDOC/ENF/VMATEM/WM/METOD/KONTR_MATEM/Pay_2.files/image091.jpg

 

2.Самостоятельное выполнение задания.

1 вариант

№1.Построить эллипс x2+4y2 =16, найдя его фокусы и полуоси.

№2.Написать каноническое уравнение гиперболы, зная, что: 1) расстояние между фокусами равно 10, а между вершинами 2а=8; 2) действительная полуось а = 2, а эксцентриситет ε =.

 

2 вариант

№ 1.Построить гиперболу х2 – 4у2 = 16 и её асимптоты. Найти фокусы, эксцентриситет и угол  между асимптотами.

№ 2. Написать каноническое уравнение эллипса, зная, что 1) расстояние между фокусами равно 2с = 8, а малая полуось b = 3; 2) большая полуось а = 6, а эксцентриситет ε = 0,5.

 

 

3.Вывод.

 

 

 

 

 

 

 

 

 

 

 

 

 

 

 

 

Практическая работа №5  «Вычисление пределов числовых последовательностей»

Цель работы: закрепление практических навыков вычисления пределов числовых последовательностей.

Ход работы:

1)повторение теоретического материала;

2)выполнение заданий;

3)вывод.

1.Краткое содержание теоретического материала.

Последовательности.  Рассмотрим ряд натуральных чисел:

 

1,  2,  3, … ,  n –1,  n, … .

 

Если заменить каждое натуральное число   в этом ряду некоторым числом  un , следуя некоторому закону, то мы получим новый ряд чисел:          

 

u1 ,   u2 ,   u3 , …,   u- 1 ,   un  , …,  кратко обозначаемый { u}  

 

и называемый числовой последовательностью. Величина  un называется общим членом последовательности. Обычно числовая последовательностьзадаётся некоторой формулой  un n ), позволяющей найти любой член последовательности по его номеру  ; эта формула называется формулой общего члена. Заметим, что задать числовую последовательность формулой общего члена не всегда возможно; иногда последовательность задаётся путём описания её членов (см. ниже последний пример).

 

П р и м е р ы    числовых последовательностей:

 

                         1,  2,  3,  4,  5, … -  ряд натуральных чисел ;

 

                         2,  4,  6,  8,  10, … - ряд чётных чисел;

 

                         1.4,  1.41,  1.414,  1.4142, … - числовая последовательность

                                                                            приближённых  значений http://www.bymath.net/studyguide/ana/sec/ana1c.gif

                                                                            с увеличивающейся точностью.

В последнем примере невозможно дать формулу общего члена последовательности, тем не менее эта последовательность описана полностью.

Предел числовой последовательности. Рассмотрим числовую последовательность, общий член которой приближается к некоторому числу   приувеличении порядкового номера  n. В этом случае говорят, что числовая последовательность имеет предел. Это понятие имеет более строгоеопределение.

http://www.bymath.net/studyguide/ana/sec/ana1d.gif

Это определение означает, что  a  есть предел числовой последовательности, если её общий член неограниченно приближается к  a  при возрастании  n. Геометрически это значит, что для любого  http://www.bymath.net/studyguide/eps.gif > 0  можно найти такое число N,  что начиная с  n > N  все члены последовательности расположены внутри интервала ( a - http://www.bymath.net/studyguide/eps.gif , a + http://www.bymath.net/studyguide/eps.gif ). Последовательность, имеющая предел, называется сходящейся; в противном случае – расходящейся.

Последовательность называется ограниченной, если существует такое число M, что | un  | http://www.bymath.net/studyguide/leq.gif M  для всех  n . Возрастающая или убывающая последовательность называется монотонной.

Теорема Вейерштрасса. Всякая монотонная и ограниченная последовательность имеет предел (эта теорема даётся в средней школе без доказательства). 

Основные свойства пределов.  Нижеприведенные свойства пределов справедливы не только для числовых последовательностей, но и для функций.

Если { u} и { v}  - две сходящиеся последовательности, то:

http://www.bymath.net/studyguide/ana/sec/ana1e.gif

Если члены последовательностей { un }, { vn }, { wn } удовлетворяют неравенствам 

http://www.bymath.net/studyguide/ana/sec/ana1f.gif

При вычислении пределов зачастую появляются выражения, значение которых не определено. Такие выражения называют неопределенностями.

Основные виды неопределенностей: http://www.webmath.ru/poleznoe/images/limit/formules_1452.png , http://www.webmath.ru/poleznoe/images/limit/formules_1445.png , http://www.webmath.ru/poleznoe/images/limit/formules_1458.png , http://www.webmath.ru/poleznoe/images/limit/formules_1459.png , http://www.webmath.ru/poleznoe/images/limit/formules_1460.png , http://www.webmath.ru/poleznoe/images/limit/formules_1461.png , http://www.webmath.ru/poleznoe/images/limit/formules_1462.png

Все другие выражения не являются неопределенностями и принимают какое-то конкретное конечное или бесконечное значение.

Раскрытие неопределенностей

Для раскрытия неопределенностей используют следующее:

1.      упрощают выражение функции: раскладывают на множители, преобразовывают функцию с помощью формул сокращенного умножения, тригонометрических формул, домножают на сопряженное, что позволяет в дальнейшем сократить и т.д., и т.п.;

2.      замечательные пределы

3.      эквивалентные бесконечно малые функции


 

Некоторые замечательные пределы. 

http://www.bymath.net/studyguide/ana/sec/ana1a.gif

                                                 

Пример. Вычислить предел http://www.webmath.ru/poleznoe/images/limit/formules_1497.png

Решение. Получим неопределенность, разложим на множители числитель и знаменатель, сократим одинаковые элементы.

http://www.webmath.ru/poleznoe/images/limit/formules_1499.png

http://www.webmath.ru/poleznoe/images/limit/formules_1500.png

Ответ. http://www.webmath.ru/poleznoe/images/limit/formules_1498.png

 

Пример . Вычислить http://abc.vvsu.ru/Books/p_chisl_pos/obj.files/image1067.gif

Очевидно, числитель и знаменатель дроби стремятся к бесконечности, то есть имеется неопределенность вида http://abc.vvsu.ru/Books/p_chisl_pos/obj.files/image1069.gif. В таком случае можно вычислить предел, разделив числитель и знаменатель дроби на старшую степень n.

http://abc.vvsu.ru/Books/p_chisl_pos/obj.files/image1071.gif

 

Ответ. 3

 

Пример. Вычислить предел http://www.webmath.ru/poleznoe/images/limit/formules_1485.png

Решение. Получим неопределенность и домножим числитель и знаменатель на выражение, сопряженное к иррациональности.

http://www.webmath.ru/poleznoe/images/limit/formules_1487.png

http://www.webmath.ru/poleznoe/images/limit/formules_1488.png

http://www.webmath.ru/poleznoe/images/limit/formules_1489.png

http://www.webmath.ru/poleznoe/images/limit/formules_1490.png

Ответ. http://www.webmath.ru/poleznoe/images/limit/formules_1486.png

 

 

2.Самостоятельное выполнение задания.

1. Вычислить пределы числовых последовательностей.

1 вариант

 

2 вариант

 

 

 

 

 

 

 

 

2.      Записать первые пять членов числовой последовательности, заданной формулой n-го члена

1        Вариант      аn=

 

2        Вариант      аn=

 

3.      Вывод.

 

 

 

 

 

 

Практическая работа №6  «Исследование функции на непрерывность. Вычисление пределов»

Цель работы: закрепление практических навыков вычисления пределов функций.

Ход работы:

1)повторение теоретического материала;

2)выполнение заданий;

3)вывод.

1.Краткое содержание теоретического материала.

 

1.1. Понятие предела функции в точке.
           Основные теоремы о пределах.

 

       Пусть функция у = f(x) определена на некотором промежутке Х и пусть точка х0 є Х. Составим из множества Х последовательность точек: х1, х2,…,хn,…сходящихся к х0. Значения функции в этих точках также образуют последовательность: f(x1), f(x2),…,f(xn).

 

       Число А называется  пределом  функции f () в точке  =, если при  любых значениях , сколь угодно близких к числу (), значение функции f ()

становится сколь угодно близким к числу А.

 

       Математическое выражение предела даётся в формуле (1.)

 

                      f () = f  ().                                                       (1)    

                                       

         1.2. Основные теоремы о пределах.

 

Пусть  существует f (), g (), тогда:

   Предел аргумента в точке равен значению аргумента в этой точке

                                   =                                                                           (2)                                                                                                            

   Если с – постоянная величина, то предел постоянной равен самой постоянной

                         c= c, c – const                                                                         (3)

   Если с – постоянная величина, то постоянный множитель выносится за знак предела

                        cx = cx                                                                             (4)

 

   Предел суммы двух функций равен сумме пределов этих функций

                                                                  (5)

 

   Предел произведения равен произведению пределов:

                                                                                  (6)

   Предел отношения равен отношению пределов, если предел знаменателя отличен от нуля:

                                                      (7)

 

   Предел степени равен степени предела

                              = ()                                                                                 (8)

                    

1.3.   Понятие бесконечно малой и бесконечно большой функции.

Предел функции на бесконечности.

 

        Функция- называется бесконечно малой  при , если .

 

       Функция- называется бесконечно большой  при , если .

       Если функция бесконечно большая, то функция- бесконечно малая и наоборот.

 

       Число А называется пределом функции на бесконечности, если при  всех достаточно больших значений х  разность  есть бесконечно малая функция

 

1.4.  Правила раскрытия неопределённостей.

 

      Часто встречаются случаи, когда непосредственно применить теоремы о пределах нельзя.

В этих случаях необходимо сначала раскрыть неопределенности и потом только вычислять пределы.

 

Ø  В ситуации, когда числитель и знаменатель дроби стремится к нулю, говорят, что имеет место

неопределенность вида . Для раскрытия неопределенности такого вида необходимо:

 

          а) числитель и знаменатель дроби разложить на множители, а затем сократить на множитель, приведший к неопределенности, при этом можно использовать:

 

  • формулы сокращенного умножения,

 

  • вынесение общего множителя за скобки,

 

  • группировку,

 

  • преобразование квадратного трехчлена с помощью дискриминанта или теоремы Виета;

      т.к. ax2 + bx + c = a (x-x1)(x-x2), x1,x2 - корни уравнения ax2+bx+c=0,

 

  • преобразование многочлена с помощью деления многочлена на (x-x0),

 

  • умножение на сопряженное выражение, т.е. если предел содержит выражение  то

      путем умножения на  избавляемся от корней, т.к. 

 

        б)  использовать первый замечательный предел.

 

Первый замечательный предел

               (9)

                                                                                          (10)

 

                   

Ø  Если числитель и знаменатель неограниченно возрастают при х→∞, то в таком случае имеет место неопределенность вида. Для ее раскрытия надо разделить числитель и знаменатель дроби на старшую степень переменной х.

 

Ø  Если имеет место неопределённость  и , то в этих случаях применяют второй замечательный предел.

 

Второй замечательный предел:

                                                               

                                                                       

Ø  Если  имеют место неопределённости  ∞-∞, 0-0, то в этих случаях необходимо заданную функцию привести к дробно-линейному виду, а затем использовать предыдущие правила

                                                              

1.5.  Понятие непрерывности функции в точке и на промежутке.

 

      Функция называется непрерывной в точке, если существует предел функции  в этой точке, и он равен значению функции в этой точке:

                                       С =f(x) = f(x0)                                                          (13)                                                                                                                                   

      Функция непрерывна на промежутке, если она непрерывна в каждой точке промежутка.

 

       Все основные элементарные функции – постоянная, показательная, логарифмическая, степенная, тригонометрическая, обратные тригонометрические непрерывные на своих областях определения.

 

        Функция f(x) непрерывна на интервале (a, b), если она непрерывна в каждой точке этого интервала.

 

       Теорема. Пусть функции f1(x) и f2(x) непрерывны в точке х0. Тогда функции f1(x) + f2(x), f1(x) · f2(x) и f1(x)/f2(x) будут также непрерывны в точке х0 (для дроби при условии, что f2(x0) ≠ 0).

Решение задач

 

Пример 1. Вычислить предел функции 

Решение.

     Используя теоремы о пределах и формулы  (2)-(5) получим

 

Пример 2. Вычислить предел функции 

Решение.

       Для того, чтобы вычислить предел функции в точке подставим значение аргумента функции в этой точке, т.е. вместо х подставим единицу:

 

Пример 3. Вычислить предел функции 

Решение.

      Имеем неопределённость вида. Используя  правило раскрытия неопределённостей (а) воспользуемся формулами сокращённого умножения: .   

 

 

 

Пример 4. Вычислить предел функции 

.

 

Решение

        Имеем неопределённость вида. Используя  правило раскрытия неопределённостей, разделим каждое слагаемое почленно на :      

 

Пример 5 Вычислить предел функции 

Решение.

      Неопределённость вида. Решим уравнения числителя и знаменателя и разложим трёхчлены на множители: = 

             

 

Пример 6.  Вычислить предел функции   

Решение

     Неопределённость вида. Домножим и числитель, и знаменатель на сопряжённый множитель:

 

Пример 7  Вычислить предел функции 

Решение

      Имеем неопределённость вида. Произведём деление многочленов столбиков на (х-2):

 =

       x3 - 5x2 + 8x – 4    x –2                                             x3 – 3x2 + 4      x – 2

       x3 – 2x2                x2 – 3x +2                                    x3 – 2x2            x2x –2

      -3x2 +8x                                                                           - x2 + 4 

      -3x2 +6x                                                                          - x2 + 2x

                2x – 4                                                                       -2x +4

                2x – 4                                                                       -2x +4

                        0                                                                                0

                          

 

Пример 8.  Вычислить предел функции   

Решение.

      Имеем неопределённость вида. Применим первый замечательный предел,

формулы (9), (10) получим:

 

Пример 9.  Вычислить предел функции 

Решение.

     Имеем неопределённость вида. Применим второй замечательный предел  формулу (12): = ==  = =

= = = =  =

=  =  =

2.Самостоятельное выполнение задания.

1.Установить являются ли функции непрерывными или разрывными. Найти точки разрыва для функций имеющих разрыв.

Вариант 1                                                   Вариант 2

 

1)y = ;                                             1)у = ;

 

2)у =                                                    2)у =

 

2.Вычислить пределы

Вариант 1

 

Вариант 2

 

 

 

 

 

 

 

 

 

 

 

 

3.Сделать вывод.

 

 

 

Практическая работа №7  «Нахождение производных функций»

Цель работы: закрепление практических навыков нахождения производных функций.

Ход работы:

1)повторение теоретического материала;

2)выполнение заданий;

3)вывод.

1.Краткое содержание теоретического материала.

 

Определение: Производной функции f(x) (f'(x)) в точке x  называется предел отношения приращения функции к приращению аргумента при приращении аргумента стремящемся к нулю:

=f′(x)

Производные элементарных функций.

2

 

Правила дифференцирования.

Если у функций  f(x) и  g(x) существуют производные, то

1.С′=0

2. (u+v)′=u′+v′

3. (uv)′=u′v+v′u

4. (С·u)′=С·u′, где С=соnst

5. ()′=

6. Производная сложной функции:

f′(g(x))=f′(g)·g′(x)

2. Примеры.

1.      Найти значение производной функции:y=3-4x+8.

Решение:

По правилу нахождения производной алгебраической суммы функций (формула 2):

y′= (3-4x+8)′= (3-(4x)′+(8)′=3·2x+·-4·1+0=6x+·

2.   Найти значение производной функции: y =+4).

Решение:

Функция представляет собой произведение двух множителей: u=+4. По формуле 3:

y′ =+4))′ = +4)+ +4)′=6+4)+·.

3.      Найти значение производной функции: y=.

Решение:

Функция представляет собой частное двух выражений: u=4. По формуле 5: y′===

4.     8

Решение. Найдем производную данной функции по правилу дифференцирования сложной функции (формула 6):

9

5.      Если l05image040, то l05image042

6.      y = x3 – 3x2 + 5x + 2. Найдем y '(–1).

y ' = 3x2 – 6x+ 5. Следовательно, y '(–1) = 14.

7.Если y = ln x · cos x, то y ' = (ln x) ' cos x + ln x (cos x) ' =1/x∙cos x – ln x · sin x.

8. l05image044

 

 

 

2. Выполнить самостоятельно:

варианта

Найти производную функции y:

варианта

Найти производную функции y:

1

 

 

1.    y=6-

2.    y=(5x+7)

3.    y=

4.     y=

5.   y=

2

 

 

1.    y=-6-

2.    y=(x+17)

3.    y=

4.     y=

5.   y=

 

3.Сделать вывод.

 

 

 

 

 

 

Практическая работа №8  «Нахождение производных  второго порядка»

Цель работы: закрепление практических навыков нахождения производных второго порядка функций.

Ход работы:

1)повторение теоретического материала;

2)выполнение заданий;

3)вывод.

1.Краткое содержание теоретического материала.

 Понятие производных высших порядков

Рассмотрим дифференцируемую функцию http://edu.dvgups.ru/METDOC/ENF/VMATEM/WM/METOD/KULIK/WEBUMK/frame/6.files/image002.gif. Найдем её производную http://edu.dvgups.ru/METDOC/ENF/VMATEM/WM/METOD/KULIK/WEBUMK/frame/6.files/image004.gif. Рассматривая http://edu.dvgups.ru/METDOC/ENF/VMATEM/WM/METOD/KULIK/WEBUMK/frame/6.files/image006.gif как новую функцию, продифференцируем её:

http://edu.dvgups.ru/METDOC/ENF/VMATEM/WM/METOD/KULIK/WEBUMK/frame/6.files/image008.gif

Полученную новую производную называют второй производной от функции http://edu.dvgups.ru/METDOC/ENF/VMATEM/WM/METOD/KULIK/WEBUMK/frame/6.files/image002.gif. Вторую производную обозначают так:

http://edu.dvgups.ru/METDOC/ENF/VMATEM/WM/METOD/KULIK/WEBUMK/frame/6.files/image011.gif или http://edu.dvgups.ru/METDOC/ENF/VMATEM/WM/METOD/KULIK/WEBUMK/frame/6.files/image013.gif.

Аналогично находится производная третьего, четвертого, и т.д. n-го порядка. Третья производная обозначается так:

http://edu.dvgups.ru/METDOC/ENF/VMATEM/WM/METOD/KULIK/WEBUMK/frame/6.files/image015.gif

Четвертая:

http://edu.dvgups.ru/METDOC/ENF/VMATEM/WM/METOD/KULIK/WEBUMK/frame/6.files/image017.gif.

Производной n – го порядка от функции http://edu.dvgups.ru/METDOC/ENF/VMATEM/WM/METOD/KULIK/WEBUMK/frame/6.files/image002.gif называется производная от производной http://edu.dvgups.ru/METDOC/ENF/VMATEM/WM/METOD/KULIK/WEBUMK/frame/6.files/image020.gif-го порядка:

http://edu.dvgups.ru/METDOC/ENF/VMATEM/WM/METOD/KULIK/WEBUMK/frame/6.files/image022.gif.

Производные высших порядков вычисляются последовательным дифференцированием данной функции.

 Решение примеров

1. Найти производную второго порядка:

http://edu.dvgups.ru/METDOC/ENF/VMATEM/WM/METOD/KULIK/WEBUMK/frame/6.files/image024.gif.

Решение. Сначала найдем производную первого порядка:

http://edu.dvgups.ru/METDOC/ENF/VMATEM/WM/METOD/KULIK/WEBUMK/frame/6.files/image026.gif.

Теперь найдем производную от производной первого порядка. Это и будет производная второго порядка:

http://edu.dvgups.ru/METDOC/ENF/VMATEM/WM/METOD/KULIK/WEBUMK/frame/6.files/image028.gif.

Ответ. У=6х + 84х2

 

2. Найти производную второго порядка от функции http://www.webmath.ru/primeri_reshenii/images/derivative/primeri_1120.png

Решение. Находим первую производную как производную сложной функции:

http://www.webmath.ru/primeri_reshenii/images/derivative/primeri_1121.png

Вторую производную находим как от произведения, предварительно вынеся по правилам дифференцирования коэффициент 3 за знак производной. Также будем учитывать, что первый множитель - http://www.webmath.ru/primeri_reshenii/images/derivative/primeri_1175.png - есть сложной функцией:

http://www.webmath.ru/primeri_reshenii/images/derivative/primeri_1122.png

http://www.webmath.ru/primeri_reshenii/images/derivative/primeri_1123.png

http://www.webmath.ru/primeri_reshenii/images/derivative/primeri_1124.png

http://www.webmath.ru/primeri_reshenii/images/derivative/primeri_1125.png

Ответ. http://www.webmath.ru/primeri_reshenii/images/derivative/primeri_1126.png

2. Выполнить самостоятельно:

Найти производную второго порядка заданных функций

 

Вариант 1

Вариант 2

1)у =  х3

1) у = х4

2)у =  cos2х

2) у = sinx

3) у = ln(3x2  - 2х + 5)

3) у = (5x + 2)4

4) у =

4) у = 105-3х

5) у = 2х

5) у =

 

3.Вывод.

 

 

 

 

 

 

 

Практическая работа №9  «Геометрический смысл производной»

Цель работы: закрепление практических навыков решения задач с применением знаний геометрического смысла производной.

Ход работы:

1)повторение теоретического материала;

2)выполнение заданий;

3)вывод.

1.Краткое содержание теоретического материала.

Геометрический смысл производной. 

Касательная к графику функции f, дифференцируемой в точке xо, - это прямая, проходящая через точку (xо; f(xо)) и имеющая угловой коэффициент ′(xо). 


Угловой коэффициент имеет прямая вида
 y = kx + b.  Коэффициент k и является угловым коэффициентом этой прямой.

Угловой коэффициент равен тангенсу острого угла, образуемого этой прямой с осью абсцисс:

 
k = tg α

 
Здесь угол α – это угол между прямой
 y = kx + b и положительным (то есть против часовой стрелки) направлением оси абсцисс. Он называется углом наклона прямой (рис.1 и 2).
 http://raal100.narod.ru/olderfiles/4/Kasatelnaya.png Если угол наклона прямой
 y = kx + b острый, то угловой коэффициент является положительным числом. График возрастает (рис.1).

Если угол наклона прямой y = kx + b тупой, то угловой коэффициент является отрицательным числом. График убывает (рис.2).

Если прямая параллельна оси абсцисс, то угол наклона прямой равен нулю. В этом случае угловой коэффициент прямой тоже равен нулю (так как тангенс нуля есть ноль). Уравнение прямой будет иметь вид y = b (рис.3).

Если угол наклона прямой равен 90º (π/2), то есть она перпендикулярна оси абсцисс, то прямая задается равенством x = c, где c – некоторое действительное число (рис.4).

 

 

Уравнение касательной к графику функции y = f(x) в точке xо:


 y = f(xо) + ′(xо) (x – xо)

 

Алгоритм решения уравнения касательной к графику функции y = f(x):

1. Вычислить f(xо).

2. Вычислить  производные ′(x) и ′(xо).

3. Внести найденные числа xо,  f(xо),  ′(xо) в уравнение касательной и решить его.

 
Пример
: Найдем уравнение касательной к графику функции
 f(x) = x3 – 2x2 + 1 в точке с абсциссой 2.

Решение.

Следуем алгоритму.

1) Точка касания xо равна 2. Вычислим f(xо):

 f(xо) = f(2) = 23 – 2 ∙ 22 + 1 = 8 – 8 + 1 = 1

2) Находим ′(x). Для этого применяем формулы дифференцирования, изложенные в предыдущем разделе. Согласно этим формулам, х2 = 2х, а х3 = 3х2. Значит:

′(x) = 3х2 – 2 ∙ 2х = 3х2 – 4х.

Теперь, используя полученное значение ′(x), вычислим ′(xо):

′(xо) = ′(2) = 3 ∙ 22 – 4 ∙ 2 = 12 – 8 = 4.

3) Итак, у нас есть все необходимые данные: xо = 2, f(xо) = 1, ′(xо) = 4. Подставляем эти числа в уравнение касательной и находим окончательное решение:

у = f(xо) + ′(xо) (x – xо) = 1 + 4 ∙ (х – 2) = 1 + 4х – 8 = –7 + 4х = 4х – 7.

Ответ: у = 4х – 7.

 

Производная в точке x 0 равна угловому коэффициенту касательной к графику функции y = f(x) в этой точке.

ghjbpd

Рассмотрим график функции y = f ( x ):

ana3b

Из рис.1 видно, что для любых двух точек A и B графика функции: http://uztest.ru/jsmath/jsMath/fonts/cmr10/alpha/85/char01.pngxf(x0+http://uztest.ru/jsmath/jsMath/fonts/cmr10/alpha/85/char01.pngx)f(x0)=tghttp://uztest.ru/jsmath/jsMath/fonts/cmmi10/alpha/120/char0B.png, где http://www.bymath.net/studyguide/alfa.gif - угол наклона секущей AB. 
Таким образом, разностное отношение равно угловому коэффициенту секущей. 
Если зафиксировать точку A и двигать по направлению к ней точку B, то http://uztest.ru/jsmath/jsMath/fonts/cmr10/alpha/120/char01.pngx неограниченно уменьшается и приближается к 0, а секущая АВ приближается к касательной АС. 
Следовательно, предел разностного отношения равен угловому коэффициенту касательной в точке A.
Отсюда следует:

производная функции в точке есть угловой коэффициент касательной к графику этой функции в этой точке.

В этом и состоит геометрический смысл производной.

 

Признаки возрастания и убывания функции:

  • Если производная функции положительна  в каждой точке некоторого интервала, то функция возрастает на этом интервале.
  • Если производная функции отрицательна в каждой  точке некоторого интервала, то функция убывает на этом интервале.

Точки экстремума.

Точки экстремума — это точки, в которых возрастание функции сменяется убыванием или наоборот. На графике они выглядят, как точки перегиба функции. Пик — это максимум, впадина — это минимум.

•    Если точка  является точкой экстремума функции f(x) и в этой точке существует производная , то она равна нулю.
•    Если в точке   производная меняет знак с плюса на минус, то точка   является точкой максимума.
•    Если в точке   производная меняет знак с минуса на плюс, то точка   является точкой минимума.

Рассмотрим несколько примеров исследования функции на возрастание и убывание.

 Найти промежутки монотонности функций:

1) http://festival.1september.ru/articles/564728/f_clip_image020.gif

а) область определения http://festival.1september.ru/articles/564728/f_clip_image022.gif,

б) найдем первую производную:http://festival.1september.ru/articles/564728/f_clip_image024.gif,

в)http://festival.1september.ru/articles/564728/f_clip_image026.gifнайдем критические точки: http://festival.1september.ru/articles/564728/f_clip_image028.gifhttp://festival.1september.ru/articles/564728/f_clip_image030.gifhttp://festival.1september.ru/articles/564728/f_clip_image032.gif и http://festival.1september.ru/articles/564728/f_clip_image034.gif

Исследуем знак производной в полученных промежутках, решение представим в виде таблицы.

http://festival.1september.ru/articles/564728/f_clip_image036.gif

http://festival.1september.ru/articles/564728/f_clip_image038.gif

0

http://festival.1september.ru/articles/564728/f_clip_image040.gif

2

http://festival.1september.ru/articles/564728/f_clip_image042.gif

http://festival.1september.ru/articles/564728/f_clip_image044.gif

+http://festival.1september.ru/articles/564728/f_clip_image026_0000.gif

0

-

0

+

http://festival.1september.ru/articles/564728/f_clip_image046.gif

 

 

Итак, в промежутках http://festival.1september.ru/articles/564728/f_clip_image048.gif функция http://festival.1september.ru/articles/564728/f_clip_image020_0000.gif возрастает, в промежутке http://festival.1september.ru/articles/564728/f_clip_image050.gif убывает.

2) http://festival.1september.ru/articles/564728/f_clip_image052.gif

а) областьопределения http://festival.1september.ru/articles/564728/f_clip_image022_0000.gif,

б) найдем первую производную: http://festival.1september.ru/articles/564728/f_clip_image055.gif,

в) найдем критические точки:http://festival.1september.ru/articles/564728/f_clip_image028_0000.gifhttp://festival.1september.ru/articles/564728/f_clip_image057.gifhttp://festival.1september.ru/articles/564728/f_clip_image032_0000.gif

Исследуем знак производной в полученных промежутках, решение представим в виде таблицы.

http://festival.1september.ru/articles/564728/f_clip_image036_0000.gif

http://festival.1september.ru/articles/564728/f_clip_image038_0000.gif

0

http://festival.1september.ru/articles/564728/f_clip_image062.gif

http://festival.1september.ru/articles/564728/f_clip_image044_0000.gif

-

0

-

http://festival.1september.ru/articles/564728/f_clip_image046_0000.gif

 

Функция http://festival.1september.ru/articles/564728/f_clip_image052_0000.gif убывает на всей области определения.

3) http://festival.1september.ru/articles/564728/f_clip_image066.gif.

а) область определения http://festival.1september.ru/articles/564728/f_clip_image022_0001.gif,

б) найдем первую производную: http://festival.1september.ru/articles/564728/f_clip_image068.gif,

в) найдем критические точки:http://festival.1september.ru/articles/564728/f_clip_image028_0001.gifhttp://festival.1september.ru/articles/564728/f_clip_image070.gifhttp://festival.1september.ru/articles/564728/f_clip_image072.gif

Исследуем знак производной в полученных промежутках, решение представим в виде таблицы.

http://festival.1september.ru/articles/564728/f_clip_image036_0001.gif

http://festival.1september.ru/articles/564728/f_clip_image074.gif

2,5

http://festival.1september.ru/articles/564728/f_clip_image076.gif

http://festival.1september.ru/articles/564728/f_clip_image044_0001.gif

-

0

+

http://festival.1september.ru/articles/564728/f_clip_image046_0001.gif

 

Функция http://festival.1september.ru/articles/564728/f_clip_image066_0000.gif возрастает на промежутке http://festival.1september.ru/articles/564728/f_clip_image078.gif, убывает на промежутке http://festival.1september.ru/articles/564728/f_clip_image080.gif

Самостоятельно найти промежутки монотонности функции http://festival.1september.ru/articles/564728/f_clip_image082.gif.

 

2. Выполнить самостоятельно:

1 . Исследовать функцию на монотонность и экстремум:

 

 

Вариант №1

1.    y=

2.       y=

Вариант №2

1.       y=

2.       y=

 

2.Составить уравнение касательной к параболе в точке с абсциссой х=2

 

1 вариант

2 вариант

у = х2 – 2х

у =  + 2х

 

3.Найти угол наклона к оси Ох касательной, проведённой к кривой:

 

1 вариант     у = sin x    в точке  х = 2π/3

 

2 вариант      y = tg x     в точке  х =  π/3

 

3.Вывод.

 

 

 

 

 

 

 

 

 

 

 

 

Практическая работа №10  «Исследование функций и построение графиков»

Цель работы: закрепление практических навыков построения графиков функций с помощью производной.

Ход работы:

1)повторение теоретического материала;

2)выполнение заданий;

3)вывод.

1.Краткое содержание теоретического материала.


Общая схема исследования функции

·         Найти область определения функции. Выделить особые точки (точки разрыва).

·         Проверить наличие вертикальных асимптот в точках разрыва и на границах области определения.

·         Найти точки пересечения с осями координат.

·         Установить, является ли функция чётной или нечётной.

·         Определить, является ли функция периодической или нет (только для тригонометрических функций, остальные непериодические, пункт пропускается).

·         Найти точки экстремума и интервалы монотонности (возрастания и убывания) функции.

·         Найти точки перегиба и интервалы выпуклости-вогнутости.

·         Найти наклонные асимптоты функции.

·         Построить график функции.

Асимптота – это прямая, к которой бесконечно близко приближается график функции, и график при этом бесконечно удаляется от начала координат. Знание уравнения асимптоты функции может быть полезно при анализе функции и построении ее графика.
В зависимости от поведения аргумента асимптоты разделяются на вертикальные, горизонтальные и наклонные. Вертикальная асимптота – это вертикальная линия вида x=α, если http://allcalc.ru/images/asimp1.png.

    

Точки разрыва функции и границы области определения являются основанием для нахождения вертикальных асимптот. Горизонтальная асимптота – горизонтальная прямая линия вида x=α, если http://allcalc.ru/images/asimp2.png. Наклонная асимптота – прямая вида y=kx+b; для существования наклонных асимптот, необходимо одновременное существование пределов http://allcalc.ru/images/asimp3.png.

Пример 1.Построить график функции 
http://pics.semestr.ru/images/math/math/math-image050.gif. 
Решение. 
1. Область определения функции D(y) = (-∞;0)U(0;∞). 
2. Функция не является четной или нечетной. 
3. Найдем точки пересечения графика с осью ОХ; имеем 
http://pics.semestr.ru/images/math/math/math-image051.gif; http://pics.semestr.ru/images/math/math/math-image052.gif. 
4. Точки разрыва x=0, причем http://pics.semestr.ru/images/math/math/math-image053.gif; следовательно, x=0 является вертикальной асимптотой графика. 
Найдем наклонные асимптоты: 
http://pics.semestr.ru/images/math/math/math-image054.gif; 
http://pics.semestr.ru/images/math/math/math-image055.gif. 
Наклонная асимптота имеет уравнение http://pics.semestr.ru/images/math/math/math-image056.gif. 
5. Найдем экстремум функции и интервалы возрастания и убывания. Имеем http://pics.semestr.ru/images/math/math/math-image057.gif. Существует единственная критическая точка x=2. В промежутках http://pics.semestr.ru/images/math/math/math-image058.gif, следовательно, функция возрастает; в промежутке http://pics.semestr.ru/images/math/math/math-image059.gif, функция убывает. Далее, находим http://pics.semestr.ru/images/math/math/math-image060.gif; http://pics.semestr.ru/images/math/math/math-image061.gif, следовательно, x=2 – точка минимума ymin=3. 
6. Найдем интервалы выпуклости и вогнутости кривой и точки ее перегиба. Так как y’’>0 (x≠0), то график функции всюду вогнут. Точек перегиба кривая не имеет. 
Строим график функции. 
http://pics.semestr.ru/images/math/math/math-image062.gif

2. Выполнить самостоятельно:

Вариант№1

Исследовать функцию и построить график:

1.y=

2.y=

3.y=

Вариант№2

Исследовать функцию и построить график:

1.y=

2.y=

3.y=

 

3.Вывод.

 

 

 

 

 

 

 

Практическая работа №11"Интегрирование простейших функций"

 

Цель работы: закрепление практических навыков нахождения неопределённых интегралов.

Ход работы:

1)повторение теоретического материала;

2)выполнение заданий;

3)вывод.

1.Краткое содержание теоретического материала.

Неопределённый интеграл и непосредственное интегрирование.

Определение первообразной и неопределенного интеграла

Функция F(x) называется первообразной функции f(x), если

http://www.math24.ru/images/1int1.gif

Множество всех первообразных некоторой функции f(x) называется неопределенным

интегралом функцииf(x) и обозначается как

http://www.math24.ru/images/1int2.gif

Таким образом, если F - некоторая частная первообразная, то справедливо выражение

http://www.math24.ru/images/1int3.gif

где С - произвольная постоянная. 

Свойства неопределенного интеграла

В приведенных ниже формулах f и g - функции переменной xF - первообразная функции f
а, k, C - постоянные величины.

http://www.math24.ru/images/1int4.gif

  http://www.math24.ru/images/1int5.gif

 http://www.math24.ru/images/1int6.gif

  http://www.math24.ru/images/1int7.gif

 

Непосредственное интегрирование – это нахождение неопределенных интегралов с использованием таблицы интегралов и свойств неопределенного интеграла.

Таблица интегралов

1

Image171

11

Image172

2

Image173

12

Image174

3

Image175(Image176).

13

Image177.

4

Image178

14

Image179

5

Image180; Image181

15

Image182

6

Image183

16

Image184

7

Image185

17

Image186

8

Image187

18

Image188

9

Image189

19

Image190

10

Image191

20

Image192; Image193

 

   Пример 1

Вычислить http://www.math24.ru/images/1int39.gif.


Решение.

      http://www.math24.ru/images/1int40.gif

   Пример 2

Вычислить интеграл http://www.math24.ru/images/1int41.gif.


Решение.

Преобразуя выражение и применяя формулу для интеграла степенной функции, получаем

      http://www.math24.ru/images/1int42.gif

   Пример 3

Вычислить http://www.math24.ru/images/1int43.gif.


Решение.

Используем табличный интеграл http://www.math24.ru/images/1int44.gif. Тогда

      http://www.math24.ru/images/1int45.gif

   Пример 4

Вычислить http://www.math24.ru/images/1int46.gif.


Решение.

Воспользовавшись табличным интегралом http://www.math24.ru/images/1int47.gif, находим

      http://www.math24.ru/images/1int48.gif

   Пример 5

Вычислить интеграл http://www.math24.ru/images/1int49.gif без использования замены переменной.


Решение.

Используя формулу двойного угла sin 2x = 2 sin x cos x и тождество sin2x + cos2x = 1, получаем

      http://www.math24.ru/images/1int52.gif

 

Основные приемы интегрирования

Для вычисления неопределенных интегралов нет такого четкого алгоритма, как для вычисления производных. Кроме того, следует иметь в виду, что бесконечно много интегралов от элементарных функций не выражаются через эти элементарные функции, а представляют собой так называемые специальные функции. Поэтому, вычисление неопределенных интегралов скорее искусство, чем работа по алгоритму. Однако два общих приема все-таки имеются.

Замена переменных.

Пусть надо вычислить http://www.allmath.ru/highermath/mathanalis/matan/matan/matan2/clip_image268.gif. Сделаем замену переменных http://www.allmath.ru/highermath/mathanalis/matan/matan/matan2/clip_image270.gif, так что http://www.allmath.ru/highermath/mathanalis/matan/matan/matan2/clip_image272.gif. Пусть нам каким-то образом удалось вычислить http://www.allmath.ru/highermath/mathanalis/matan/matan/matan2/clip_image274.gif. Тогда имеет место формула

http://www.allmath.ru/highermath/mathanalis/matan/matan/matan2/clip_image276.gif.

*      Пример 6.

*       Image245имеет смысл перейти к переменной (сделать подстановку) t = sin x. Выражаем все множители подынтегрального выражения через переменную t: Image246; в результате Image247Image248(возвращаемся к исходной переменной) Image249. Другие примеры:
Image250. Подынтегральная функция содержит два множителя, ни один из которых не является производной другого, поэтому подводить их под знак дифференциала бесполезно. Попытаемся ввести новую переменную, такую, чтобы корни извлеклись: Image251= Image252Image253

*      Пример 7.

*       Image0 Image254. Здесь подынтегральная функция состоит из единственного множителя; можно опять попытаться сделать такую замену переменной, чтобы корень извлёкся. Структура подкоренного выражения подсказывает эту замену: Image255(или Image256, Image257): Image258. Интеграл свёлся к интегралу от квадрата косинуса. При интегрировании чётных степеней синуса и косинуса часто применяются формулы, выражающие Image259и Image260через косинус двойного угла: Image261. Image0Поэтому Image262Image263
Image264.
Пример 8.

dx==dt=dt=+С=

 

         Интегрирование по частям.

         Пусть http://www.allmath.ru/highermath/mathanalis/matan/matan/matan2/clip_image278.gif и http://www.allmath.ru/highermath/mathanalis/matan/matan/matan2/clip_image280.gif - две функции. Тогда имеет место формула

http://www.allmath.ru/highermath/mathanalis/matan/matan/matan2/clip_image282.gif.

*      Пример 9.

==x· =x·

*       

*      Пример 10.

*       

== =

*      Пример 11.

==.

 

 

   Полиномы и дробно-рациональная функция

Выражение

http://www.allmath.ru/highermath/mathanalis/matan/matan/matan2/clip_image284.gif

называется полиномом или многочленом степени n от переменной х. Число b (вещественное или комплексное) называется корнем полинома http://www.allmath.ru/highermath/mathanalis/matan/matan/matan2/clip_image286.gif, если http://www.allmath.ru/highermath/mathanalis/matan/matan/matan2/clip_image288.gif.

         Всякий полином может быть представлен в виде

http://www.allmath.ru/highermath/mathanalis/matan/matan/matan2/clip_image290.gif,

где сомножитель вида http://www.allmath.ru/highermath/mathanalis/matan/matan/matan2/clip_image292.gif соответствует вещественному корню b кратности k, а сомножитель вида http://www.allmath.ru/highermath/mathanalis/matan/matan/matan2/clip_image294.gif - паре комплексно-сопряженных корней кратности l. Отметим, что http://www.allmath.ru/highermath/mathanalis/matan/matan/matan2/clip_image296.gif.

         Функция вида http://www.allmath.ru/highermath/mathanalis/matan/matan/matan2/clip_image298.gif, где http://www.allmath.ru/highermath/mathanalis/matan/matan/matan2/clip_image300.gif и http://www.allmath.ru/highermath/mathanalis/matan/matan/matan2/clip_image286.gif - полиномы соответствующих степеней, называется дробно-рациональной функцией или дробью. Если m < n, то дробно-рациональная функция называется правильной.

         Теорема 1.  Пусть http://www.allmath.ru/highermath/mathanalis/matan/matan/matan2/clip_image303.gif есть правильная рациональная дробь и b есть вещественный корень P(x) кратности k, то есть http://www.allmath.ru/highermath/mathanalis/matan/matan/matan2/clip_image305.gif. Тогда

http://www.allmath.ru/highermath/mathanalis/matan/matan/matan2/clip_image307.gif,

где http://www.allmath.ru/highermath/mathanalis/matan/matan/matan2/clip_image309.gif, http://www.allmath.ru/highermath/mathanalis/matan/matan/matan2/clip_image311.gif, а http://www.allmath.ru/highermath/mathanalis/matan/matan/matan2/clip_image313.gif - полином такой степени, что второе слагаемое есть правильная рациональная дробь.

         Теорема 2.  Пусть http://www.allmath.ru/highermath/mathanalis/matan/matan/matan2/clip_image303.gif есть правильная рациональная дробь и b есть комплексный корень P(x) кратности l, то есть http://www.allmath.ru/highermath/mathanalis/matan/matan/matan2/clip_image315.gif. Тогда

http://www.allmath.ru/highermath/mathanalis/matan/matan/matan2/clip_image317.gif,

где http://www.allmath.ru/highermath/mathanalis/matan/matan/matan2/clip_image311.gif, а http://www.allmath.ru/highermath/mathanalis/matan/matan/matan2/clip_image313.gif - полином такой степени, что второе слагаемое есть правильная рациональная дробь.

   Интегрирование дробно-рациональных функций

Пусть http://www.allmath.ru/highermath/mathanalis/matan/matan/matan2/clip_image303.gif есть правильная рациональная дробь, у которой http://www.allmath.ru/highermath/mathanalis/matan/matan/matan2/clip_image319.gif. Тогда, согласно предыдущим теоремам, ее можно представить в виде

http://www.allmath.ru/highermath/mathanalis/matan/matan/matan2/clip_image321.gif,

которое называется разложением правильной рациональной дроби на простейшие.

         Для нахождения коэффициентов разложения стандартным является следующий алгоритм:

1. Написать разложение рациональной дроби на простейшие с неопределенными коэффициентами.

2. Привести правую часть получившегося выражения к общему знаменателю, раскрыть скобки и собрать члены с одинаковыми степенями х.

3. Приравнять коэффициенты при одинаковых степенях х в знаменателях получившейся и исходной дроби.

4. Решить получившуюся систему линейных алгебраических уравнений и найти все неопределенные коэффициенты.

В результате интеграл http://www.allmath.ru/highermath/mathanalis/matan/matan/matan2/clip_image323.gif распадется на сумму интегралов следующих типов:

http://www.allmath.ru/highermath/mathanalis/matan/matan/matan2/clip_image325.gif,   http://www.allmath.ru/highermath/mathanalis/matan/matan/matan2/clip_image327.gif http://www.allmath.ru/highermath/mathanalis/matan/matan/matan2/clip_image329.gif,    http://www.allmath.ru/highermath/mathanalis/matan/matan/matan2/clip_image331.gif и http://www.allmath.ru/highermath/mathanalis/matan/matan/matan2/clip_image333.gif, http://www.allmath.ru/highermath/mathanalis/matan/matan/matan2/clip_image335.gif.

Все они вычисляются в явном виде. Имеем

http://www.allmath.ru/highermath/mathanalis/matan/matan/matan2/clip_image337.gif,    http://www.allmath.ru/highermath/mathanalis/matan/matan/matan2/clip_image339.gif.

Запоминать явные выражения для интегралов двух последних типов не надо.

 

2.Самостоятельное выполнение заданий

Найти неопределенный интеграл:

*       

№1

1.   

2.  

3.  

4.  

5.  

                     №2                                                                      

1.  

2.  

3.  

4.   

5.  

 

 

 

 

 

№3

1.  

2.  

3.  

4.  

5.    dx

 

3.Вывод.

 

 

 

 

 

 

 

 

 

 

Практическая работа №12 "Вычисление простейших определённых интегралов"

 

Цель работы: закрепление практических навыков вычисления определённых интегралов.

Ход работы:

1)повторение теоретического материала;

2)выполнение заданий;

3)вывод.

1.Краткое содержание теоретического материала.

Определенный интеграл, его свойства и вычисление

    Определенный интеграл вычисляется по формуле Ньютона-Лейбница:

= F(a)-F(b)

 - соответственно верхний и нижний пределы интегрирования, они пишутся и читаются снизу вверх, а в формулу подставляются сверху вниз!)

Основные свойства определенного интеграла:

1.      При перестановке пределов интегрирования изменяется знак интеграла:

 

2.      Отрезок интегрирования можно разбивать на части:

3.      Определенный интеграл от алгебраической суммы  функций равен алгебраической сумме их определенных интегралов.

4.      Постоянный множитель можно выносить за знак интеграла.

Пример 1.

==27-8=19.

 Вычисления определённого интеграла методом введения новой переменной.

Пример 2.

====

Пример 3.

= - =-()=-

 Вычисление определенного интеграла по частям:

Используем формулу:

-

Пример 4.

=-+=()+-1-1=-2;

Пример 5.

=-6xctgx +=-6·-6·+ln|sinx|+ ln|sin|- ln|sin|= π+ ln1- ln= π+ 0+ln2= π+ln2

2.Выполнить самостоятельно:

Вычислить определенный интеграл.

Вариант 1

Вариант 2

1.  

 

 

3

 

4

1.  

 

2

 

3

 

 

4

 

 

3.Вывод.

 

 

 

Практическая работа №13 "Решение прикладных задач с помощью определённого интеграла"

 

Цель работы: закрепление практических навыков решения задач с помощью определённого интеграла.

Ход работы:

1)повторение теоретического материала;

2)выполнение заданий;

3)вывод.

1.Краткое содержание теоретического материала.

 Задача о вычислении пути

Согласно физическому смыслу первой производной, производная функции в точке есть мгновенная скорость точки, т.е. http://festival.1september.ru/articles/641283/Image957.gif. Отсюда, http://festival.1september.ru/articles/641283/Image958.gif. Интегрируя полученное равенство в пределах от t1 до t2 получаем

http://festival.1september.ru/articles/641283/Image959.gif

Тогда путь, пройденный точкой при неравномерном движении по прямой с переменной скоростью http://festival.1september.ru/articles/641283/img1.jpg(е) за отрезок времени [http://festival.1september.ru/articles/641283/Image960.gif]выражается интегралом

http://festival.1september.ru/articles/641283/img2.jpg                          (1)

Пример 1. Скорость прямолинейного движения тела выражается формулой http://festival.1september.ru/articles/641283/img1.jpg = 2t+3t2(м/с). Найти путь, пройденный телом за 5 секунд от начала движения.

Решение.

http://festival.1september.ru/articles/641283/img3.jpg

Пример 2. Два тела начали двигаться одновременно из одной точки в одном направлении по прямой. Первое тело движется со скоростью vhttp://festival.1september.ru/articles/641283/Image962.gif=(6t2+2t) м/с, второе – со скоростью v2=(4t+5) м/с. На каком расстояния друг от друга они окажутся через 5 с?

Решение. Искомая величина есть разность расстояний, пройденных телами за 5 с.

http://festival.1september.ru/articles/641283/img4.jpg

Таким образом, S=S1-S2= 275-75=200 (м).

2. Задача о вычислении работы переменной силы

Пусть материальная точка под действием силы F движется по прямой. Если действующая сила постояна, а пройденный путь равен s, то как известно из курса физики, работа А этой F вычисляется по формуле:

А= F*s

Работу переменной силы f(x) при перемещении по оси Оx материальной точки от x=a до x=b, находим по формуле (3):

A=http://festival.1september.ru/articles/641283/Image967.gif (2)

Решении задач на вычисление работы силы упругости, связанных с растяжением и сжатием пружин, основывается на законе Гука. По закону Гука сила F, растягивающая или сжимающая пружину, пропорциональная этому растяжению или сжатию, т.е. F=kx, где x – величина растяжения или сжатия, k – коэффициент пропорциональности.

Пример 1. Сила упругости F пружины, растянутой на 11 = 0,05 м, равна 3H. Какую работу надо произвести, чтобы растянуть пружину на 12 =0,1 м?

Решение. Подставив данные в формулу закона Гука, получим: 3=k*0.05, т.е. k=60, следовательно, сила упругости выражается соотношением F=60x. Найдем работу переменной силы по формуле (2), полагая, что а=0; b=0,1:

A=http://festival.1september.ru/articles/641283/Image969.gif=0,3Дж

3. Задача о силе давления жидкости

Согласно закону Паскаля величина P давления жидкости на горизонтальную площадку вычисляется по формуле P=gphS, (4)

Где g – ускорение свободного падения в м/с2;

p– плотность жидкости в кг/м3;

h – глубина погружения площадки в м;

S – площадь площадки в м2.

По этой формуле нельзя искать давление жидкости на вертикально погруженную пластинку, так как ее разные точки лежат на разных глубинах.

Пусть в жидкость погружена вертикально пластина, ограниченная линиями х = а, х = b, у1 = f1(x) и у2=f2(х).

Для решения задачи разобьем пластину на n частей (малых горизонт альных полосок) прямыми, параллельными поверхности жидкости (т.е. параллельными оси OY). На глубине х выделим одну из них и обозначим через f(x) ее длину, а через http://festival.1september.ru/articles/641283/Image970.gif ее ширину. Приняв полоску за прямоугольник, находим ее площадь http://festival.1september.ru/articles/641283/Image971.gif.

http://festival.1september.ru/articles/641283/Image972.gif

Найдем дифференциал dp этой функции.

http://festival.1september.ru/articles/641283/Image973.gifhttp://festival.1september.ru/articles/641283/Image974.gif

Тогда по закону Паскаля интегрируя полученное равенство в пределах от х = а до х = b, получим

P=ghttp://festival.1september.ru/articles/641283/Image975.gif                        (3)

Пример

Аквариум имеет форму прямоугольного параллелепипеда. Найдем силу давления воды (плотность воды 1000 кг/м3), наполняющей аквариум, на одну из его вертикальных стенок, размеры которой 0,4 м x 0,7 м.

Решение. Выберем систему координат так, чтобы оси Оy и Оx соответственно содержали верхнее основание и боковую сторону вертикальной стенки аквариума. Для нахождения силы давления воды на стенку воспользуемся формулой (3). Стенка имеет форму прямоугольника, поэтому http://festival.1september.ru/articles/641283/img5.jpg Так как пределы интегрирования а=0 и b=0,4, то получим:

http://festival.1september.ru/articles/641283/img6.jpg

 

           2. Выполнить самостоятельно:

Вариант 1.

1.Вычислить площадь фигуры, ограниченной линиями у = 4 – х2  и  у = - 5. Сделать чертёж.

 

2.Скорость движения точки v = 18t – 3t2  м/с. Найдите путь, пройденный точкой от начала движения до её остановки.

 

3.Пружина в спокойном состоянии имеет длину 0,1 м. Сила в 20 Н растягивает её на 0,01 м. Какую работу надо совершить, чтобы растянуть её от 0,12 до 0,14 м?

 

                    Вариант 2.

1.Вычислить площадь фигуры, ограниченной линиями у = х2  - 4х  и  у = 0. Сделать чертёж.

 

2.Тело брошено с поверхности земли  вертикально вверх со скоростью v = 29,4 – 9,8t  м/с. Найдите наибольшую высоту подъёма тела.

 

3.При сжатии пружины на 0,05 м совершается работа 30 Дж. Какую работу необходимо совершить, чтобы сжать пружину на 0,08 м?

 

 

 

3.Вывод.

 

 

 

 

 

 

Практическая работа №14 "Исследование функций двух переменных"

 

Цель работы: закрепление практических навыков исследования функций двух переменных.

Ход работы:

1)повторение теоретического материала;

2)выполнение заданий;

3)вывод.

1.Краткое содержание теоретического материала.

 

 

Определение функции двух переменных

 

Если каждой паре ( x;y) значений двух независимых друг от друга переменных величин х и у из некоторого множества D соответствует единственное значение величины, то говорят, что z есть функция двух независимых переменных x и y, определенная на множестве D.

 

Обозначается: z=f(x;y) или z=z(x;y).

 

Например,  S=ab, S=S(a;b)- функции двух переменных; V=abc, V=V(a,b.c) – функция трех переменных;

 

Способы задания функций нескольких переменных

 

Чтобы задать функцию двух (трех) переменных, нужно указать способ, с помощью которого для каждой пары (тройки) значений аргументов можно найти соответствующее значение функции. Наиболее часто функция задается аналитически - это явное задание функции или неявное задание

Частные производные первого порядка

Пусть функция двух переменных z = f(x, у) определена в некоторой окрестности точки М(x, у) евклидова пространства Е2. Частная производная функции z = f(x, у) по аргументу x является обыкновенной производной функции одной переменной х при фиксированном значении переменной у и обозначается как

http://fislac.ru/lin_prog/ris4/image567.gif

Аналогичным образом определяется частная производная функции f(x, у) по переменной у в точке М, обозначаемая как

http://fislac.ru/lin_prog/ris4/image568.gif

Функция, имеющая частные производные, называется дифференцируемой.

Совершенно аналогично определяются частные производные функций трех и более переменных. Частная производная функции нескольких переменных характеризует скорость ее изменения по данной координате при фиксированных значениях других координат.

Пример: http://mathprofi.ru/f/chastnye_proizvodnye_primery_clip_image008.gif – функция двух переменных.

Иногда используют запись http://mathprofi.ru/f/chastnye_proizvodnye_primery_clip_image010.gif. Также встречаются задания, где вместо буквы http://mathprofi.ru/f/chastnye_proizvodnye_primery_clip_image012.gif используется буква http://mathprofi.ru/f/chastnye_proizvodnye_primery_clip_image014.gif.

Найти частные производные первого и второго порядка функции http://mathprofi.ru/f/chastnye_proizvodnye_primery_clip_image020.gif

Сначала найдем частные производные первого порядка. Их две.

Обозначения:
http://mathprofi.ru/f/chastnye_proizvodnye_primery_clip_image022.gif или http://mathprofi.ru/f/chastnye_proizvodnye_primery_clip_image024.gif – частная производная по «икс»
http://mathprofi.ru/f/chastnye_proizvodnye_primery_clip_image026.gif или http://mathprofi.ru/f/chastnye_proizvodnye_primery_clip_image028.gif – частная производная по «игрек»

Начнем с http://mathprofi.ru/f/chastnye_proizvodnye_primery_clip_image022_0000.gif. Когда мы находим частную производную по «икс», то переменная http://mathprofi.ru/f/chastnye_proizvodnye_primery_clip_image006_0000.gifсчитается константой (постоянным числом).

http://mathprofi.ru/f/chastnye_proizvodnye_primery_clip_image031.gif

Комментарии к выполненным действиям:

(1) Первое, что мы делаем при нахождении частной производной – заключаем всю функцию в скобки под штрих с подстрочным индексом.

 (2) Используем правила дифференцирования http://mathprofi.ru/f/chastnye_proizvodnye_primery_clip_image037.gif, http://mathprofi.ru/f/chastnye_proizvodnye_primery_clip_image039.gif. Для простого примера, как этот, оба правила вполне можно применить на одном шаге. Обратите внимание на первое слагаемое: так как http://mathprofi.ru/f/chastnye_proizvodnye_primery_clip_image006_0001.gif считается константой, а любую константу можно вынести за знак производной, то http://mathprofi.ru/f/chastnye_proizvodnye_primery_clip_image041.gif мы выносим за скобки. То есть в данной ситуацииhttp://mathprofi.ru/f/chastnye_proizvodnye_primery_clip_image041_0000.gif ничем не лучше обычного числа. Теперь посмотрим на третье слагаемое http://mathprofi.ru/f/chastnye_proizvodnye_primery_clip_image043.gif: здесь, наоборот, выносить нечего. Так как http://mathprofi.ru/f/chastnye_proizvodnye_primery_clip_image006_0002.gif константа, то http://mathprofi.ru/f/chastnye_proizvodnye_primery_clip_image043_0000.gif – тоже константа, и в этом смысле она ничем не лучше последнего слагаемого – «семерки».

(3) Используем табличные производные http://mathprofi.ru/f/chastnye_proizvodnye_primery_clip_image045.gif и http://mathprofi.ru/f/chastnye_proizvodnye_primery_clip_image047.gif.

(4) Теперь http://mathprofi.ru/f/chastnye_proizvodnye_primery_clip_image026_0000.gif. Когда мы находим частную производную по «игрек», то переменная http://mathprofi.ru/f/chastnye_proizvodnye_primery_clip_image004_0000.gif считается константой (постоянным числом).

http://mathprofi.ru/f/chastnye_proizvodnye_primery_clip_image051.gif

(1) Используем те же правила дифференцирования http://mathprofi.ru/f/chastnye_proizvodnye_primery_clip_image037_0000.gif, http://mathprofi.ru/f/chastnye_proizvodnye_primery_clip_image039_0000.gif. В первом слагаемом выносим константу http://mathprofi.ru/f/chastnye_proizvodnye_primery_clip_image053.gif за знак производной, во втором слагаемом ничего вынести нельзя поскольку http://mathprofi.ru/f/chastnye_proizvodnye_primery_clip_image055.gif – уже константа.

(2) Используем таблицу производным элементарных функций. Мысленно поменяем в таблице все «иксы» на «игреки». В частности, используемые нами формулы выглядят так: http://mathprofi.ru/f/chastnye_proizvodnye_primery_clip_image045_0000.gif и http://mathprofi.ru/f/chastnye_proizvodnye_primery_clip_image058.gif.

Итак, частные производные первого порядка найдены

Подведем итог, чем же отличается нахождение частных производных от нахождения «обычных» производных функции одной переменной:

1) Когда мы находим частную производную http://mathprofi.ru/f/chastnye_proizvodnye_primery_clip_image022_0001.gif, переменная http://mathprofi.ru/f/chastnye_proizvodnye_primery_clip_image006_0004.gif считается константой.

2) Когда мы находим частную производную http://mathprofi.ru/f/chastnye_proizvodnye_primery_clip_image026_0001.gif, переменная http://mathprofi.ru/f/chastnye_proizvodnye_primery_clip_image004_0001.gif считается константой.

3) Правила и таблица производных элементарных функций справедливы и применимы для любой переменной (http://mathprofi.ru/f/chastnye_proizvodnye_primery_clip_image004_0002.gif, http://mathprofi.ru/f/chastnye_proizvodnye_primery_clip_image006_0005.gif либо какой-нибудь другой), по которой ведется дифференцирование.

Шаг второй. Находим частные производные второго порядка. Их четыре.

Обозначения:
http://mathprofi.ru/f/chastnye_proizvodnye_primery_clip_image063.gif или http://mathprofi.ru/f/chastnye_proizvodnye_primery_clip_image065.gif – вторая производная по «икс»
http://mathprofi.ru/f/chastnye_proizvodnye_primery_clip_image067.gif или http://mathprofi.ru/f/chastnye_proizvodnye_primery_clip_image069.gif – вторая производная по «игрек»
http://mathprofi.ru/f/chastnye_proizvodnye_primery_clip_image071.gif или http://mathprofi.ru/f/chastnye_proizvodnye_primery_clip_image073.gif – смешанная производная «икс по игрек»
http://mathprofi.ru/f/chastnye_proizvodnye_primery_clip_image075.gif или http://mathprofi.ru/f/chastnye_proizvodnye_primery_clip_image077.gif – смешанная производная «игрек по икс»

В понятии второй производной нет ничего сложного. Говоря простым языком, вторая производная – это производная от первой производной.

Частные производные первого порядка:
http://mathprofi.ru/f/chastnye_proizvodnye_primery_clip_image079.gif
http://mathprofi.ru/f/chastnye_proizvodnye_primery_clip_image081.gif

Сначала найдем смешанные производные:
http://mathprofi.ru/f/chastnye_proizvodnye_primery_clip_image083.gif

Как видите, всё просто: берем частную производную http://mathprofi.ru/f/chastnye_proizvodnye_primery_clip_image022_0002.gif и дифференцируем ее еще раз, но в данном случае – уже по «игрек».

Аналогично:
http://mathprofi.ru/f/chastnye_proizvodnye_primery_clip_image086.gif

В практических примерах можно ориентироваться на следующее равенство:
http://mathprofi.ru/f/chastnye_proizvodnye_primery_clip_image088.gif

Таким образом, через смешанные производные второго порядка очень удобно проверить, а правильно ли мы нашли частные производные первого порядка.

Находим вторую производную по «икс».
Никаких изобретений, берем http://mathprofi.ru/f/chastnye_proizvodnye_primery_clip_image079_0000.gif и дифференцируем её по «икс» еще раз:
http://mathprofi.ru/f/chastnye_proizvodnye_primery_clip_image091.gif

Аналогично:
http://mathprofi.ru/f/chastnye_proizvodnye_primery_clip_image093.gif

Следует отметить, что при нахождении http://mathprofi.ru/f/chastnye_proizvodnye_primery_clip_image063_0000.gif, http://mathprofi.ru/f/chastnye_proizvodnye_primery_clip_image067_0000.gif нужно проявить повышенное внимание, так как никаких чудесных равенств для их проверки не существует.

 

Примеры нахождения частных производных первого порядка.

http://fislac.ru/lin_prog/ris4/image569.gif

Решение. Дифференцируем функцию z = f(x, y) сначала по х, полагая у фиксированной величиной, потом повторяем эту же процедуру, меняя роли x и у. Получаем

http://fislac.ru/lin_prog/ris4/image570.gif

http://fislac.ru/lin_prog/ris4/image571.gif

Решение. Частные производные этой функции трех переменных выражаются следующими формулами:

http://fislac.ru/lin_prog/ris4/image572.gif

 

Алгоритм исследования функции двух переменных на экстремум

Функция z = f(x,y) имеет максимум в точке M0(x0;y0), если f(x0;y0) > f(x;y) для всех точек (x;y), достаточно близких к точке (x0;y0) и отличных от неё. Функция z = f(x,y) имеет минимум в точке M0(x0;y0), если f(x0;y0) < f(x;y) для всех точек (x;y), достаточно близких к точке (x0;y0) и отличных от неё. Максимум и минимум функции называются экстремумами функции.
Исследование функции двух переменных на экстремум проводят по следующей схеме. 
1. Находят 
частные производные dz/dx и dz/dy. 
2. Решают систему уравнений:

http://math.semestr.ru/math/images/extremum_image001.gif


и таким образом находят критические точки функции. 
3. Находят частные производные второго порядка:

http://math.semestr.ru/math/images/extremum_image002.gif


4. Вычисляют значения этих частных производных второго порядка в каждой из найденных в п.2 критических точках M(x0;y0).

http://math.semestr.ru/math/images/extremum_image003.gif


5. Делают вывод о наличии экстремумов: 
а) если AC – B2 > 0 и A < 0 , то в точке M имеется максимум; 
б) если AC – B2 > 0 и A > 0 , то в точке M имеется минимум; 
в) если AC – B2 < 0, то экстремума нет; 
г) если AC – B2 = 0, то вопрос о наличии экстремума остается открытым;

 

Примеры исследования функций двух переменных на экстремум.
Пример №1
. Исследовать на экстремум функцию 
http://pics.semestr.ru/images/math/math/math-image130.gif 
Решение. 
1) Найдем частные производные: 
http://pics.semestr.ru/images/math/math/math-image131.gif 
http://pics.semestr.ru/images/math/math/math-image132.gif 
2) тогда система для отыскания стационарных точек имеет вид 
http://pics.semestr.ru/images/math/math/math-image133.gif. 
Решив систему, получим четыре стационарные точки: 
P1(1,2), P2(2,1), P3(-1,-2), P4(-2,-1). Найдем производные 2-го порядка 
http://pics.semestr.ru/images/math/math/math-image134.gif 
и составим дискриминант ∆AB-C2 для каждой стационарной точки. 
1) Для точки P1(1,2) Δ=36-144<0, в P1(1,2) экстремума нет. 
2) Для точки P2(-1,-2), Δ =144 – 36 >0, A>0, в P2(2,1)функция имеет минимум, zmin = -28 
3) Для точки P3(-1,-2), Δ = 36-144 <0, в P3(-1,-2) экстремума нет. 
4) Для точки P4(-2,-1), Δ=144 – 36 >0, A<0 в P4(-2,-1) функция имеет максимум zmax=28

Пример №2. Найти наибольшее и наименьшее значения функции z = x2 + 4xy – y2 – 6x – 2y в треугольнике, ограниченном осями координат и прямой 2x + 3y – 6 = 0. 
Решение. 
Найдем критические точки локального экстремума внутри указанной области и значения данной функции z = f(x,y) в этих точках. Так как http://pics.semestr.ru/images/math/math/math-image097.gif,http://pics.semestr.ru/images/math/math/math-image098.gif, то система для отыскания критических точек имеет вид 
http://pics.semestr.ru/images/math/math/math-image099.gifhttp://pics.semestr.ru/images/math/math/math-image100.gifhttp://pics.semestr.ru/images/math/math/math-image101.gif. 
Точка P0(1;1) находится внутри области, причем z(P0) = -4. Исследуем функцию z на границе области. На отрезке ОА имеем : y=0, z = f(x;0) или g1(x)=x2 – 6x, гдеhttp://pics.semestr.ru/images/math/math/math-image102.gif; g’1=2x-6; g’1(3)=0; g1(3)=f(3;0)=-9. На отрезке ОВ имеем: x=0. z=f(0;y) или z=g2(y)=-y2-2y, где http://pics.semestr.ru/images/math/math/math-image103.gif, http://pics.semestr.ru/images/math/math/math-image104.gif. На отрезке АВ имеем http://pics.semestr.ru/images/math/math/math-image105.gif http://pics.semestr.ru/images/math/math/math-image106.gif или 
http://pics.semestr.ru/images/math/math/math-image107.gifгдеhttp://pics.semestr.ru/images/math/math/math-image102.gif; http://pics.semestr.ru/images/math/math/math-image108.gif; http://pics.semestr.ru/images/math/math/math-image109.gif, 
http://pics.semestr.ru/images/math/math/math-image110.gif. 
Найдем значения функции z в точках О, А и В. z(0) = f(0,0) = 0; z(A) = f(3;0) = -9, z(B) = f(0;2) = -8. 
Сравнивая значения http://pics.semestr.ru/images/math/math/math-image111.gif, приходим к выводу: 
наибольшее значение zmax=0 в т. O(0,0); 
наименьшее значение zmin=-9 в т. A(3,0).

2.Самостоятельное выполнение заданий.

Вариант 1

Вариант 2

1.Найти частные производные первого порядка функции:

z = x3 + 3x2y – y3

z = x+ xy + y2

2Исследовать на экстремум функцию:

1)z = 2x2 + 3y2 + 2xy – 10x + 16y – 7

1)z = - 2x2  - y2 + 3xy – 2x + 7y + 6

2)z = - 5x2 – 3y2 + 2xy – 18x – 10y + 4

2)z = 5 – 7x2 – 5y2 + 2xy – 34x + 34y

 

3.Сделать вывод.

 

 

 

 

 

 

 

 

 

 

 

Практическая работа №15 "Решение дифференциальных уравнений первого порядка"

 

Цель работы: закрепление практических навыков решения дифференциальных уравнений первого порядка.

Ход работы:

1)повторение теоретического материала;

2)выполнение заданий;

3)вывод.

1.Краткое содержание теоретического материала.

           Дифференциальное уравнение – равенство, содержащее производные или дифференциалы неизвестной функции.

 

          Общий вид дифференциального уравнения:

F(x,y,y,y ,,…)=0

 где x – независимая переменная, y – неизвестная функция,  y, - её производная первого порядка и т.д.

 

          Решение дифференциального уравнения – функция, подстановка которой в это уравнение обращает его тождество.

 

           Общее решение – решение дифференциального уравнения, содержащее столько произвольных постоянных, каков порядок уравнения.

 

           Частное решение – это решение, получающееся из общего решения при конкретных определенных значениях произвольных постоянных C

 

           Для нахождения частных решений задают начальные условия.

 

           Порядок дифференциального уравнения – наивысший порядок производных или дифференциалов, входящих в это уравнение.

 

           Интегральная кривая -  график y=F(x), построенный на плоскости xOy,являющийся решением дифференциального уравнения.

 

           Общему решению y=F(x,C) соответствует семейство интегральных кривых, зависящих от постоянной С.

 

           Теорема Коши: Если функция f(x,y) непрерывна и имеет непрерывную производную то решение дифференциального уравнения y’=f(x,y) при начальном условии f(x0)=y0 существует и единственно т.е. через точку (x00) проходит единственная интегральная кривая данного уравнения.

 

1.1. Виды дифференциальных уравнений

 

Виды дифференциальных уравнений:

 

   Обыкновенные дифференциальные уравнения - уравнения, в которых одна независимая переменная

 

   Дифференциальные уравнения в частных производных - уравнения, в которых независимых переменных две и более

 

Виды дифференциальных уравнений представлены в таблице 1.

 

Таблица 1.

Обыкновенные дифференциальные уравнения первого порядка

Название

Вид

Способ решения

С разделяющимися переменными

P(x,y)dx+Q(x,y)dy=0

если P(x,y) и Q(x,y) разлагаются на множители, зависящие каждый только от одной переменной.

Т.е.

f(x)g(y)dx+(x)q(y)dy=0

или

y’= f(x)g(y)

1.разделить переменные

2.проинтегрировать

3.привести к стандартному виду

y=(x)+c – общее решение

Однородные

P(x,y)dx+ Q(x,y)dy=0

где P(x,y), Q(x,y) – однородные функции одного измерения

или

y’=

(если в функции заменить x=tx, y=ty и преобразовать вернемся к исходному уравнению)

1. замена y=tx,  тогда  

2. привести к уравнению с разделяющимися переменными и решить (см. выше).

3. вернуться к замене, подставить

4. привести к стандартному виду y=

Линейные

y’+P(x)y=Q(x)

(y’ и у’ входят в первых степенях не перемножаясь между собой)

а) линейное однородное

y’+P(x)y=0

б) линейное неоднородное

y’+P(x)y=Q(x)

в) уравнение Бернулли

y’+P(x)y=Q(x)y’’

1. замена y=uv,тогда y’=uv+vu

2. u’v+v’u+ P(x) uv= Q(x)

    v(u’+P(x)u)+v’u= Q(x)  (*)

3. в уравнении (*) приравнять скобку к нулю

u’+P(x)u=0 – c разделенными переменными

найти u

u=P(x)

4. значение u подставить в уравнение (*)

vP(x)=Q(x) - c разделенными переменными

найти v

v=F(x)+c

5. вернуться к замене

y=P(x)(F(x)+c) – общее решение

 

 

Пример 1

Решить дифференциальное уравнение http://mathprofi.ru/g/differencialnye_uravnenija_primery_reshenii_clip_image022.gif

В первую очередь нужно переписать производную немного в другом виде. Вспоминаем обозначение http://mathprofi.ru/g/differencialnye_uravnenija_primery_reshenii_clip_image024.gif. Итак:
http://mathprofi.ru/g/differencialnye_uravnenija_primery_reshenii_clip_image026.gif

На втором шаге смотрим, нельзя ли разделить переменные? Это значит, что в левой части нам нужно оставить только «игреки», а в правую часть перенести все  «иксы». Разделение переменных выполняется с помощью «школьных» действий: вынесение за скобки, перенос слагаемых из части в часть со сменой знака, перенос множителей из части в часть по правилу пропорции и т.п.

Дифференциалы http://mathprofi.ru/g/differencialnye_uravnenija_primery_reshenii_clip_image028.gif и http://mathprofi.ru/g/differencialnye_uravnenija_primery_reshenii_clip_image030.gif – это полноправные множители и активные участники действий. В рассматриваемом примере переменные легко разделяются перекидыванием множителей по правилу пропорции:
http://mathprofi.ru/g/differencialnye_uravnenija_primery_reshenii_clip_image032.gif

Переменные разделены. В левой части – только «игреки», в правой части – только «иксы».

Следующий этап – интегрирование дифференциального уравнения. Всё просто, навешиваем интегралы на обе части:
http://mathprofi.ru/g/differencialnye_uravnenija_primery_reshenii_clip_image034.gif

Разумеется, интегралы нужно взять. В данном случае они табличные:
http://mathprofi.ru/g/differencialnye_uravnenija_primery_reshenii_clip_image036.gif
Как мы помним, к любой 
первообразной приписывается константа. Здесь два интеграла, но константу http://mathprofi.ru/g/differencialnye_uravnenija_primery_reshenii_clip_image038.gif достаточно записать один раз (т.к. константа + константа всё равно равна другой константе). В большинстве случаев её помещают в правую часть.

Строго говоря, после того, как взяты интегралы, дифференциальное уравнение считается решённым. Единственное, у нас «игрек» не выражен через «икс», то есть решение представлено в неявном виде.  Решение дифференциального уравнения в неявном виде называется общим интегралом дифференциального уравнения. То есть, http://mathprofi.ru/g/differencialnye_uravnenija_primery_reshenii_clip_image036_0000.gif – это общий интеграл.

Ответ в такой форме вполне приемлем, но нет ли варианта получше? Давайте попытаемся получить общее решение.

Пожалуйста, запомните первый технический приём, он очень распространен и часто применяется в практических заданиях: если в правой части после интегрирования появляется логарифм, то константу во многих случаях (но далеко не всегда!) тоже целесообразно записать под логарифмом.

То есть, ВМЕСТО записи http://mathprofi.ru/g/differencialnye_uravnenija_primery_reshenii_clip_image036_0001.gif обычно пишут http://mathprofi.ru/g/differencialnye_uravnenija_primery_reshenii_clip_image041.gif.

Зачем это нужно? А для того, чтобы легче было выразить «игрек». Используем свойство логарифмов http://mathprofi.ru/g/differencialnye_uravnenija_primery_reshenii_clip_image046.gif. В данном случае:
http://mathprofi.ru/g/differencialnye_uravnenija_primery_reshenii_clip_image048.gif

Теперь логарифмы и модули можно с чистой совестью убрать:
http://mathprofi.ru/g/differencialnye_uravnenija_primery_reshenii_clip_image050.gif

Функция представлена в явном виде. Это и есть общее решение.

Ответ: общее решение: http://mathprofi.ru/g/differencialnye_uravnenija_primery_reshenii_clip_image052.gif

Ответы многих дифференциальных уравнений довольно легко проверить. В нашем случае это делается совсем просто, берём найденное решение http://mathprofi.ru/g/differencialnye_uravnenija_primery_reshenii_clip_image050_0000.gif и дифференцируем его:
http://mathprofi.ru/g/differencialnye_uravnenija_primery_reshenii_clip_image062.gif

После чего подставляем http://mathprofi.ru/g/differencialnye_uravnenija_primery_reshenii_clip_image050_0001.gif и производную http://mathprofi.ru/g/differencialnye_uravnenija_primery_reshenii_clip_image064.gif в исходное уравнение http://mathprofi.ru/g/differencialnye_uravnenija_primery_reshenii_clip_image022_0002.gif:
http://mathprofi.ru/g/differencialnye_uravnenija_primery_reshenii_clip_image066.gif
http://mathprofi.ru/g/differencialnye_uravnenija_primery_reshenii_clip_image068.gif – получено верное равенство, значит, общее решение http://mathprofi.ru/g/differencialnye_uravnenija_primery_reshenii_clip_image050_0002.gif удовлетворяет уравнению http://mathprofi.ru/g/differencialnye_uravnenija_primery_reshenii_clip_image022_0003.gif, что и требовалось проверить.

Придавая константе http://mathprofi.ru/g/differencialnye_uravnenija_primery_reshenii_clip_image038_0001.gif различные значения, можно получить бесконечно много частных решений дифференциального уравнения. Ясно, что любая из функций http://mathprofi.ru/g/differencialnye_uravnenija_primery_reshenii_clip_image055.gif, http://mathprofi.ru/g/differencialnye_uravnenija_primery_reshenii_clip_image057.gif, http://mathprofi.ru/g/differencialnye_uravnenija_primery_reshenii_clip_image059.gif и т.д. удовлетворяет дифференциальному уравнению http://mathprofi.ru/g/differencialnye_uravnenija_primery_reshenii_clip_image022_0001.gif.

Иногда общее решение называют семейством функций. В данном примере общее решение  http://mathprofi.ru/g/differencialnye_uravnenija_primery_reshenii_clip_image052_0000.gif – это семейство линейных функций, а точнее, семейство прямых пропорциональностей.

После обстоятельного разжевывания первого примера уместно ответить на несколько наивных вопросов о дифференциальных уравнениях:

1) В этом примере нам удалось разделить переменные. Всегда ли это можно сделать? Нет, не всегда. И даже чаще переменные разделить нельзя. Например, в однородных уравнениях первого порядка, необходимо сначала провести замену. В других типах уравнений, например, в линейном неоднородном уравнении первого порядка, нужно использовать различные приёмы и методы для нахождения общего решения. Уравнения с разделяющимися переменными, которые мы рассматриваем в первом примере – простейший тип дифференциальных уравнений.

2) Всегда ли можно проинтегрировать дифференциальное уравнение? Нет, не всегда. Очень легко придумать «навороченное» уравнение, которое не проинтегрировать,  кроме того, существуют неберущиеся  интегралы. Но подобные ДУ можно решить приближенно с помощью специальных методов.

3) В данном примере мы получили решение в виде общего интеграла http://mathprofi.ru/g/differencialnye_uravnenija_primery_reshenii_clip_image041_0000.gif. Всегда ли можно из общего интеграла найти общее решение, то есть, выразить «игрек» в явном виде? Нет не всегда. Например: http://mathprofi.ru/g/differencialnye_uravnenija_primery_reshenii_clip_image071.gif. Ну и как тут выразить «игрек»?! В таких случаях ответ следует записать в виде общего интеграла. Кроме того, иногда общее решение найти можно, но оно записывается настолько громоздко и коряво, что уж лучше оставить ответ в виде общего интеграла

Пример 2

Найти частное решение дифференциального уравнения http://mathprofi.ru/g/differencialnye_uravnenija_primery_reshenii_clip_image073.gif, удовлетворяющее начальному условию http://mathprofi.ru/g/differencialnye_uravnenija_primery_reshenii_clip_image075.gif

Решение: по условию требуется найти частное решение ДУ, удовлетворяющее заданному начальному условию. Такая постановка вопроса также называется задачей Коши.

Сначала находим общее решение. В уравнении нет переменной «икс», но это не должно смущать, главное, в нём есть первая производная.

Переписываем производную в нужном виде:
http://mathprofi.ru/g/differencialnye_uravnenija_primery_reshenii_clip_image077.gif

Очевидно, что переменные можно разделить:
http://mathprofi.ru/g/differencialnye_uravnenija_primery_reshenii_clip_image079.gif

Интегрируем уравнение:
http://mathprofi.ru/g/differencialnye_uravnenija_primery_reshenii_clip_image081.gif
http://mathprofi.ru/g/differencialnye_uravnenija_primery_reshenii_clip_image083.gif

Общий интеграл получен. Здесь константа с надстрочной звездочкой, дело в том, что очень скоро она превратится в другую константу.

Теперь пробуем общий интеграл преобразовать в общее решение (выразить «игрек» в явном виде). Вспоминаем старое, доброе, школьное: http://mathprofi.ru/g/differencialnye_uravnenija_primery_reshenii_clip_image085.gif. В данном случае:
http://mathprofi.ru/g/differencialnye_uravnenija_primery_reshenii_clip_image087.gif

 Используя свойство степеней, перепишем функцию следующим образом:
http://mathprofi.ru/g/differencialnye_uravnenija_primery_reshenii_clip_image089.gif

Если http://mathprofi.ru/g/differencialnye_uravnenija_primery_reshenii_clip_image091.gif – это константа, то http://mathprofi.ru/g/differencialnye_uravnenija_primery_reshenii_clip_image093.gif – тоже некоторая константа, переообозначим её буквой http://mathprofi.ru/g/differencialnye_uravnenija_primery_reshenii_clip_image038_0002.gif:
http://mathprofi.ru/g/differencialnye_uravnenija_primery_reshenii_clip_image096.gif
Запомните «снос» константы – это второй технический приём, который часто используют в ходе решения дифференциальных уравнений.

Итак, общее решение: http://mathprofi.ru/g/differencialnye_uravnenija_primery_reshenii_clip_image098.gif.

На завершающем этапе нужно найти частное решение, удовлетворяющее заданному начальному условию http://mathprofi.ru/g/differencialnye_uravnenija_primery_reshenii_clip_image075_0000.gif.

В чём состоит задача? Необходимо подобрать такое значение константы http://mathprofi.ru/g/differencialnye_uravnenija_primery_reshenii_clip_image038_0002.gif, чтобы выполнялось условие http://mathprofi.ru/g/differencialnye_uravnenija_primery_reshenii_clip_image075_0000.gif.

Оформить можно по-разному, но понятнее всего, пожалуй, будет так. В общее решение вместо «икса» подставляем ноль, а вместо «игрека» двойку:
http://mathprofi.ru/g/differencialnye_uravnenija_primery_reshenii_clip_image101.gif
http://mathprofi.ru/g/differencialnye_uravnenija_primery_reshenii_clip_image103.gif
http://mathprofi.ru/g/differencialnye_uravnenija_primery_reshenii_clip_image105.gif
То есть, http://mathprofi.ru/g/differencialnye_uravnenija_primery_reshenii_clip_image107.gif

Стандартная версия оформления:
http://mathprofi.ru/g/differencialnye_uravnenija_primery_reshenii_clip_image109.gif

Теперь в общее решение http://mathprofi.ru/g/differencialnye_uravnenija_primery_reshenii_clip_image096_0000.gif подставляем найденное значение константы http://mathprofi.ru/g/differencialnye_uravnenija_primery_reshenii_clip_image107_0000.gif:
http://mathprofi.ru/g/differencialnye_uravnenija_primery_reshenii_clip_image112.gif – это и есть нужное нам частное решение.

Ответ: частное решение: http://mathprofi.ru/g/differencialnye_uravnenija_primery_reshenii_clip_image112.gif

Выполним проверку. Проверка частного решение включает в себя два этапа:

Сначала необходимо проверить, а действительно ли найденное частное решение http://mathprofi.ru/g/differencialnye_uravnenija_primery_reshenii_clip_image112_0000.gifудовлетворяет начальному условию http://mathprofi.ru/g/differencialnye_uravnenija_primery_reshenii_clip_image075_0001.gif? Вместо «икса» подставляем ноль и смотрим, что получится:
http://mathprofi.ru/g/differencialnye_uravnenija_primery_reshenii_clip_image114.gif – да, действительно получена двойка, значит, начальное условие выполняется.

Второй этап уже знаком. Берём полученное частное решение http://mathprofi.ru/g/differencialnye_uravnenija_primery_reshenii_clip_image112_0001.gif и находим производную:
http://mathprofi.ru/g/differencialnye_uravnenija_primery_reshenii_clip_image116.gif

Подставляем http://mathprofi.ru/g/differencialnye_uravnenija_primery_reshenii_clip_image118.gif и http://mathprofi.ru/g/differencialnye_uravnenija_primery_reshenii_clip_image120.gif в исходное уравнение http://mathprofi.ru/g/differencialnye_uravnenija_primery_reshenii_clip_image073_0000.gif:

http://mathprofi.ru/g/differencialnye_uravnenija_primery_reshenii_clip_image122.gif
http://mathprofi.ru/g/differencialnye_uravnenija_primery_reshenii_clip_image124.gif – получено верное равенство.

Вывод: частное решение найдено правильно.

Пример 3

Решить дифференциальное уравнение http://mathprofi.ru/g/lineinye_differencialnye_uravnenija_clip_image031.gif

Решение: Данное уравнение является линейным и имеет простейший вид: http://mathprofi.ru/g/lineinye_differencialnye_uravnenija_clip_image025_0000.gif.

Как решить линейное уравнение?

Существуют два способа решения. Первый способ – это так называемый метод вариации произвольной постоянной, но мы им не воспользуемся. Второй способ связан с заменой переменной и подстановкой, иногда его называют методом Бернулли. В данном примере будет рассматриваться метод подстановки, он алгоритмически прост и понятен, и решение уравнения принимает чёткий трафаретный характер.

Сделаем замену:

http://mathprofi.ru/g/lineinye_differencialnye_uravnenija_clip_image034.gif, где http://mathprofi.ru/g/lineinye_differencialnye_uravnenija_clip_image036.gif и http://mathprofi.ru/g/lineinye_differencialnye_uravnenija_clip_image038.gif – некоторые, пока ещё неизвестные функции, зависящие от «икс».

Коль скоро проводится замена http://mathprofi.ru/g/lineinye_differencialnye_uravnenija_clip_image034_0000.gif, то нужно выяснить, чему равна производная. По правилу дифференцирования произведения:http://mathprofi.ru/g/lineinye_differencialnye_uravnenija_clip_image040.gif

Подставляем http://mathprofi.ru/g/lineinye_differencialnye_uravnenija_clip_image034_0001.gif и http://mathprofi.ru/g/lineinye_differencialnye_uravnenija_clip_image040_0000.gif в наше уравнение http://mathprofi.ru/g/lineinye_differencialnye_uravnenija_clip_image031_0000.gif:
http://mathprofi.ru/g/lineinye_differencialnye_uravnenija_clip_image043.gif

В чём состоит задача? Необходимо найти неизвестные функции «у» и «v», которые зависят от «икс». И как раз этому будут посвящены все последующие действия.

После подстановки смотрим на два слагаемых, которые располагаются вот на этих местах:
http://mathprofi.ru/g/lineinye_differencialnye_uravnenija_clip_image045.jpg
У них нужно вынести за скобки всё, что можно вынести. В данном случае:
http://mathprofi.ru/g/lineinye_differencialnye_uravnenija_clip_image047.gif

Теперь нужно составить систему уравнений. Система составляется стандартно:

Приравниваем к нулю то, что находится в скобках: http://mathprofi.ru/g/lineinye_differencialnye_uravnenija_clip_image049.gif.

Если http://mathprofi.ru/g/lineinye_differencialnye_uravnenija_clip_image049_0000.gif, тогда из нашего уравнения http://mathprofi.ru/g/lineinye_differencialnye_uravnenija_clip_image047_0000.gif получаем: http://mathprofi.ru/g/lineinye_differencialnye_uravnenija_clip_image052.gif или просто http://mathprofi.ru/g/lineinye_differencialnye_uravnenija_clip_image054.gif.

Уравнения записываем в систему:
http://mathprofi.ru/g/lineinye_differencialnye_uravnenija_clip_image056.gif.

Именно в таком порядке.

Система опять же решается стандартно.

Сначала из первого уравнения находим функцию http://mathprofi.ru/g/lineinye_differencialnye_uravnenija_clip_image038_0000.gif. Это простейшее уравнение с разделяющимися переменными.

http://mathprofi.ru/g/lineinye_differencialnye_uravnenija_clip_image059.gif

Функция http://mathprofi.ru/g/lineinye_differencialnye_uravnenija_clip_image038_0001.gif найдена. Обратите внимание, что константу http://mathprofi.ru/g/lineinye_differencialnye_uravnenija_clip_image062.gif на данном этапе мы не приписываем

Далее подставляем найденную функцию http://mathprofi.ru/g/lineinye_differencialnye_uravnenija_clip_image064.gif во второе уравнение системы http://mathprofi.ru/g/lineinye_differencialnye_uravnenija_clip_image054_0000.gif:
http://mathprofi.ru/g/lineinye_differencialnye_uravnenija_clip_image067.gif

Из второго уравнения находим функцию http://mathprofi.ru/g/lineinye_differencialnye_uravnenija_clip_image036_0000.gif.
http://mathprofi.ru/g/lineinye_differencialnye_uravnenija_clip_image070.gif
http://mathprofi.ru/g/lineinye_differencialnye_uravnenija_clip_image072.gif
Функция http://mathprofi.ru/g/lineinye_differencialnye_uravnenija_clip_image036_0001.gif найдена. А вот здесь уже добавляем константу http://mathprofi.ru/g/lineinye_differencialnye_uravnenija_clip_image062_0000.gif.

Обе функции найдены: 
http://mathprofi.ru/g/lineinye_differencialnye_uravnenija_clip_image064_0000.gif 
http://mathprofi.ru/g/lineinye_differencialnye_uravnenija_clip_image076.gif

Записываем общее решение:
http://mathprofi.ru/g/lineinye_differencialnye_uravnenija_clip_image078.gif

В ответе можно раскрыть скобки, это дело вкуса:

Ответ: общее решение http://mathprofi.ru/g/lineinye_differencialnye_uravnenija_clip_image080.gif

Проверка :

Берём полученный ответ http://mathprofi.ru/g/lineinye_differencialnye_uravnenija_clip_image082.gif и находим производную:
http://mathprofi.ru/g/lineinye_differencialnye_uravnenija_clip_image084.gif

Подставим http://mathprofi.ru/g/lineinye_differencialnye_uravnenija_clip_image086.gif и http://mathprofi.ru/g/lineinye_differencialnye_uravnenija_clip_image088.gif в исходное уравнение http://mathprofi.ru/g/lineinye_differencialnye_uravnenija_clip_image031_0001.gif:
http://mathprofi.ru/g/lineinye_differencialnye_uravnenija_clip_image090.gif
Получено верное равенство, таким образом, общее решение найдено правильно.

Пример 2

Найти общее решение дифференциального уравнения http://mathprofi.ru/g/lineinye_differencialnye_uravnenija_clip_image092.gif

Решение: Данное уравнение имеет «классический» вид http://mathprofi.ru/g/lineinye_differencialnye_uravnenija_clip_image002_0000.gif линейного уравнения. Проведем замену: http://mathprofi.ru/g/lineinye_differencialnye_uravnenija_clip_image094.gif и подставим http://mathprofi.ru/g/lineinye_differencialnye_uravnenija_clip_image034_0003.gif и http://mathprofi.ru/g/lineinye_differencialnye_uravnenija_clip_image097.gif в исходное уравнение http://mathprofi.ru/g/lineinye_differencialnye_uravnenija_clip_image092_0000.gif:
http://mathprofi.ru/g/lineinye_differencialnye_uravnenija_clip_image099.gif

После подстановки проведем вынесение множителя за скобки.

  http://mathprofi.ru/g/lineinye_differencialnye_uravnenija_clip_image101.gif

Составляем систему. Для этого приравниванием к нулю то, что находится в скобках: http://mathprofi.ru/g/lineinye_differencialnye_uravnenija_clip_image103.gif, автоматически получая и второе уравнение системы:
http://mathprofi.ru/g/lineinye_differencialnye_uravnenija_clip_image105.gif

В результате:
http://mathprofi.ru/g/lineinye_differencialnye_uravnenija_clip_image107.gif.

Из первого уравнения найдем функцию http://mathprofi.ru/g/lineinye_differencialnye_uravnenija_clip_image038_0002.gif:
http://mathprofi.ru/g/lineinye_differencialnye_uravnenija_clip_image110.gif
http://mathprofi.ru/g/lineinye_differencialnye_uravnenija_clip_image112.gif – найденную функцию  http://mathprofi.ru/g/lineinye_differencialnye_uravnenija_clip_image038_0003.gif подставим во второе уравнение системы http://mathprofi.ru/g/lineinye_differencialnye_uravnenija_clip_image114.gif:
http://mathprofi.ru/g/lineinye_differencialnye_uravnenija_clip_image116.gif
Теперь находим функцию http://mathprofi.ru/g/lineinye_differencialnye_uravnenija_clip_image036_0002.gif. Уравнение опять получилось простенькое:
http://mathprofi.ru/g/lineinye_differencialnye_uravnenija_clip_image119.gif
http://mathprofi.ru/g/lineinye_differencialnye_uravnenija_clip_image121.gif

Обе функции найдены: 
http://mathprofi.ru/g/lineinye_differencialnye_uravnenija_clip_image112_0000.gif
http://mathprofi.ru/g/lineinye_differencialnye_uravnenija_clip_image123.gif
Таким образом:
Общее решение: http://mathprofi.ru/g/lineinye_differencialnye_uravnenija_clip_image125.gif

Ответ: общее решение: http://mathprofi.ru/g/lineinye_differencialnye_uravnenija_clip_image127.gif

 

 

 

2.Самостоятельное выполнение заданий.

 

Вариант 1

Вариант 2

1. Найти общее решение дифференциального уравнения с разделяющимися переменными.

 

2. Найти частное решение дифференциального уравнения с разделяющимися переменными.

 у(4) = 0

(1 + y)dx = (1 – x)dy;  у(3) = - 2

3. Найти решение линейного дифференциального уравнения первого порядка.

 

 

 

4. Найти частное решение линейного дифференциального уравнения.

 =,  если у(1) = 3

, если у(1) = 1

 

3.Вывод.

 

 

 

 

 

 

 

 

 

 

 

 

Практическая работа №16 "Решение дифференциальных уравнений второго порядка"

 

Цель работы: закрепление практических навыков решения дифференциальных уравнений второго порядка.

Ход работы:

1)повторение теоретического материала;

2)выполнение заданий;

3)вывод.

1.Краткое содержание теоретического материала.

В дифференциальное уравнение второго порядка обязательно входит вторая производная http://mathprofi.ru/h/differencialnye_uravnenija_vtorogo_poryadka_clip_image002.gif и не входят производные более высоких порядков: 
http://mathprofi.ru/h/differencialnye_uravnenija_vtorogo_poryadka_clip_image004.gif

В теории  и практике различают два типа таких уравнений – однородное уравнение и неоднородное уравнение.

Однородное ДУ второго порядка с постоянными коэффициентами имеет следующий вид:
http://mathprofi.ru/h/differencialnye_uravnenija_vtorogo_poryadka_clip_image017.gif, где http://mathprofi.ru/h/differencialnye_uravnenija_vtorogo_poryadka_clip_image019.gif и http://mathprofi.ru/h/differencialnye_uravnenija_vtorogo_poryadka_clip_image021.gif – константы (числа), а в правой части – строго ноль.

Неоднородное ДУ второго порядка с постоянными коэффициентами имеет вид:
http://mathprofi.ru/h/differencialnye_uravnenija_vtorogo_poryadka_clip_image023.gif, где http://mathprofi.ru/h/differencialnye_uravnenija_vtorogo_poryadka_clip_image019_0000.gif и http://mathprofi.ru/h/differencialnye_uravnenija_vtorogo_poryadka_clip_image021_0000.gif – константы, а http://mathprofi.ru/h/differencialnye_uravnenija_vtorogo_poryadka_clip_image025.gif – функция, зависящая только от «икс». В простейшем случае функция http://mathprofi.ru/h/differencialnye_uravnenija_vtorogo_poryadka_clip_image025_0000.gif может быть числом, отличным от нуля.

 

Обыкновенные дифференциальные уравнения второго порядка.

Допускающие понижения порядка

y’’=f(x)

Решаются двойным интегрированием

Линейные однородные второго порядка с постоянными коэффициентами

y’’+py+qy=0

где p, qзаданные числа

 

Всякое Л.О.У. второго порядка имеет систему двух линейно независимых частных решений.

которая называется фундаментальной системой решений.

 

Общее решение есть линейная комбинация частных решений его фундаментальной системы

 

1.Составить характеристическое уравнение

2.в зависимости от вида корней, фундаментальная система решений имеет вид:

корни

характеристического уравнения

фундаментальная система частных решений

общее решение

действительные

Различные

Действительные

 или

Равные

 k1=k2=kR

Комплексные

(мнимые)

Комплексные

 

Пример 1

Решить дифференциальное уравнение http://mathprofi.ru/h/differencialnye_uravnenija_vtorogo_poryadka_clip_image050.gif

Решение: составим и решим характеристическое уравнение:
http://mathprofi.ru/h/differencialnye_uravnenija_vtorogo_poryadka_clip_image052.gif
http://mathprofi.ru/h/differencialnye_uravnenija_vtorogo_poryadka_clip_image054.gif

http://mathprofi.ru/h/differencialnye_uravnenija_vtorogo_poryadka_clip_image056.gif, http://mathprofi.ru/h/differencialnye_uravnenija_vtorogo_poryadka_clip_image058.gif
Получены два различных действительных корня .
Всё, что осталось сделать – записать ответ, руководствуясь формулой http://mathprofi.ru/h/differencialnye_uravnenija_vtorogo_poryadka_clip_image042_0000.gif

Ответ: общее решение: http://mathprofi.ru/h/differencialnye_uravnenija_vtorogo_poryadka_clip_image060.gif

Общее решение линейного неоднородного уравнения второго порядка с постоянными коэффициентами, т.е. уравнения вида http://abc.vvsu.ru/Books/u_vyssh_m1/obj.files/image4665.gif, записывается в виде http://abc.vvsu.ru/Books/u_vyssh_m1/obj.files/image4667.gif, где http://abc.vvsu.ru/Books/u_vyssh_m1/obj.files/image4669.gif- общее решение соответствующего однородного уравнения, а http://abc.vvsu.ru/Books/u_vyssh_m1/obj.files/image4671.gif- какое-либо частное решение неоднородного уравнения.

Укажем способ, позволяющий найти частное решение неоднородного уравнения по виду правой части. Заметим, что это возможно лишь в случаях, когда правая часть уравнения является функцией определенного вида.

1.                   Пусть http://abc.vvsu.ru/Books/u_vyssh_m1/obj.files/image4673.gif, где http://abc.vvsu.ru/Books/u_vyssh_m1/obj.files/image4674.gif- некоторое число, не равное нулю. Тогда

http://abc.vvsu.ru/Books/u_vyssh_m1/obj.files/image4676.gif если, http://abc.vvsu.ru/Books/u_vyssh_m1/obj.files/image4678.gifhttp://abc.vvsu.ru/Books/u_vyssh_m1/obj.files/image4680.gif, то частное решение уравнения ищут в виде http://abc.vvsu.ru/Books/u_vyssh_m1/obj.files/image4682.gif, где http://abc.vvsu.ru/Books/u_vyssh_m1/obj.files/image4684.gif- неизвестное число, которое находят, подставляя http://abc.vvsu.ru/Books/u_vyssh_m1/obj.files/image4686.gif в неоднородное уравнение;

http://abc.vvsu.ru/Books/u_vyssh_m1/obj.files/image4688.gif если http://abc.vvsu.ru/Books/u_vyssh_m1/obj.files/image4690.gif, а http://abc.vvsu.ru/Books/u_vyssh_m1/obj.files/image4692.gif, то в этом случае частное решение ищут в виде http://abc.vvsu.ru/Books/u_vyssh_m1/obj.files/image4694.gif;

http://abc.vvsu.ru/Books/u_vyssh_m1/obj.files/image4696.gif наконец, если и http://abc.vvsu.ru/Books/u_vyssh_m1/obj.files/image4698.gif и http://abc.vvsu.ru/Books/u_vyssh_m1/obj.files/image4700.gif, т.е. http://abc.vvsu.ru/Books/u_vyssh_m1/obj.files/image4702.gif, то http://abc.vvsu.ru/Books/u_vyssh_m1/obj.files/image4704.gif.

2. Если           http://abc.vvsu.ru/Books/u_vyssh_m1/obj.files/image4706.gif, где http://abc.vvsu.ru/Books/u_vyssh_m1/obj.files/image4708.gif – многочлен степениhttp://abc.vvsu.ru/Books/u_vyssh_m1/obj.files/image4709.gifhttp://abc.vvsu.ru/Books/u_vyssh_m1/obj.files/image4711.gif, то

http://abc.vvsu.ru/Books/u_vyssh_m1/obj.files/image4713.gif при http://abc.vvsu.ru/Books/u_vyssh_m1/obj.files/image4715.gifhttp://abc.vvsu.ru/Books/u_vyssh_m1/obj.files/image4717.gif решение ищут, просто «передразнивая» правую часть, т.е. http://abc.vvsu.ru/Books/u_vyssh_m1/obj.files/image4719.gif, как и правая часть, должна представлять собой произведение многочлена той же степени, что и в правой части уравнения, но с неопределенными коэффициентами, и http://abc.vvsu.ru/Books/u_vyssh_m1/obj.files/image4721.gif,т.е. http://abc.vvsu.ru/Books/u_vyssh_m1/obj.files/image4723.gif. В частности, если http://abc.vvsu.ru/Books/u_vyssh_m1/obj.files/image4725.gif, тоhttp://abc.vvsu.ru/Books/u_vyssh_m1/obj.files/image4727.gif;

http://abc.vvsu.ru/Books/u_vyssh_m1/obj.files/image4729.gifпри http://abc.vvsu.ru/Books/u_vyssh_m1/obj.files/image4731.gifhttp://abc.vvsu.ru/Books/u_vyssh_m1/obj.files/image4733.gif частное решение http://abc.vvsu.ru/Books/u_vyssh_m1/obj.files/image4735.gif ищут в виде http://abc.vvsu.ru/Books/u_vyssh_m1/obj.files/image4737.gif;

http://abc.vvsu.ru/Books/u_vyssh_m1/obj.files/image4739.gif при http://abc.vvsu.ru/Books/u_vyssh_m1/obj.files/image4741.gif находим http://abc.vvsu.ru/Books/u_vyssh_m1/obj.files/image4743.gif по формуле http://abc.vvsu.ru/Books/u_vyssh_m1/obj.files/image4745.gif.

2.                  Пусть теперь http://abc.vvsu.ru/Books/u_vyssh_m1/obj.files/image4747.gif, т.е. в правой части уравнения находится многочлен некоторой степени или некоторое число (если степень многочлена нулевая). Тогда мы можем воспользоваться формулами, рассмотренными выше, полагая в них http://abc.vvsu.ru/Books/u_vyssh_m1/obj.files/image4749.gif. (Действительно http://abc.vvsu.ru/Books/u_vyssh_m1/obj.files/image4751.gif и, очевидно, http://abc.vvsu.ru/Books/u_vyssh_m1/obj.files/image4753.gif).

Таким образом, имеем:

http://abc.vvsu.ru/Books/u_vyssh_m1/obj.files/image4755.gif если http://abc.vvsu.ru/Books/u_vyssh_m1/obj.files/image4756.gifhttp://abc.vvsu.ru/Books/u_vyssh_m1/obj.files/image4758.gif, то http://abc.vvsu.ru/Books/u_vyssh_m1/obj.files/image4760.gif;

http://abc.vvsu.ru/Books/u_vyssh_m1/obj.files/image4762.gif если http://abc.vvsu.ru/Books/u_vyssh_m1/obj.files/image4764.gifhttp://abc.vvsu.ru/Books/u_vyssh_m1/obj.files/image4766.gif, то http://abc.vvsu.ru/Books/u_vyssh_m1/obj.files/image4768.gif;

http://abc.vvsu.ru/Books/u_vyssh_m1/obj.files/image4770.gif если http://abc.vvsu.ru/Books/u_vyssh_m1/obj.files/image4772.gif, то http://abc.vvsu.ru/Books/u_vyssh_m1/obj.files/image4774.gif.

 

Пример 2

Найти общее решение дифференциального уравнения.

http://mathprofi.ru/h/kak_reshit_neodnorodnoe_uravnenie_vtorogo_poryadka_clip_image016.gif

Решение: 
1) Сначала найдем общее решение соответствующего однородного уравнения. http://mathprofi.ru/h/kak_reshit_neodnorodnoe_uravnenie_vtorogo_poryadka_clip_image019.gif

Составим и решим характеристическое уравнение:
http://mathprofi.ru/h/kak_reshit_neodnorodnoe_uravnenie_vtorogo_poryadka_clip_image021.gif
http://mathprofi.ru/h/kak_reshit_neodnorodnoe_uravnenie_vtorogo_poryadka_clip_image023.gif – получены различные действительные корни, поэтому общее решение: http://mathprofi.ru/h/kak_reshit_neodnorodnoe_uravnenie_vtorogo_poryadka_clip_image025.gif

2) Теперь нужно найти какое-либо частное решение http://mathprofi.ru/h/kak_reshit_neodnorodnoe_uravnenie_vtorogo_poryadka_clip_image009_0002.gif неоднородного уравнения http://mathprofi.ru/h/kak_reshit_neodnorodnoe_uravnenie_vtorogo_poryadka_clip_image016_0001.gif

И вопрос, который вызывает затруднения чаще всего: В каком виде нужно искать частное решение http://mathprofi.ru/h/kak_reshit_neodnorodnoe_uravnenie_vtorogo_poryadka_clip_image009_0003.gif?

Прежде всего, смотрим на нашу правую часть: http://mathprofi.ru/h/kak_reshit_neodnorodnoe_uravnenie_vtorogo_poryadka_clip_image028.gif. Тут у нас многочлен третьей степени. По идее, частное решение тоже следует искать в виде многочлена третьей степени: http://mathprofi.ru/h/kak_reshit_neodnorodnoe_uravnenie_vtorogo_poryadka_clip_image030.gif, где http://mathprofi.ru/h/kak_reshit_neodnorodnoe_uravnenie_vtorogo_poryadka_clip_image032.gif – пока ещё неизвестные коэффициенты (числа).

После предварительного анализа смотрим на корни характеристического уравнения http://mathprofi.ru/h/kak_reshit_neodnorodnoe_uravnenie_vtorogo_poryadka_clip_image023_0000.gif, найденные на предыдущем этапе: это различные действительные корни, отличные от нуля.

Используем метод неопределенных коэффициентов.

Найдём первую и вторую производную:

http://mathprofi.ru/h/kak_reshit_neodnorodnoe_uravnenie_vtorogo_poryadka_clip_image035.gif
http://mathprofi.ru/h/kak_reshit_neodnorodnoe_uravnenie_vtorogo_poryadka_clip_image037.gif

Подставим http://mathprofi.ru/h/kak_reshit_neodnorodnoe_uravnenie_vtorogo_poryadka_clip_image009_0004.gif и http://mathprofi.ru/h/kak_reshit_neodnorodnoe_uravnenie_vtorogo_poryadka_clip_image040.gif в левую часть неоднородного уравнения:
http://mathprofi.ru/h/kak_reshit_neodnorodnoe_uravnenie_vtorogo_poryadka_clip_image042.gif

(1) Выполняем подстановку http://mathprofi.ru/h/kak_reshit_neodnorodnoe_uravnenie_vtorogo_poryadka_clip_image030_0001.gif и http://mathprofi.ru/h/kak_reshit_neodnorodnoe_uravnenie_vtorogo_poryadka_clip_image045.gif.
(2) Раскрываем скобки.
(3) После максимальных упрощений ставим знак равенства и приписываем нашу правую часть http://mathprofi.ru/h/kak_reshit_neodnorodnoe_uravnenie_vtorogo_poryadka_clip_image047.gif.

Далее работаем с последним равенством – необходимо приравнять коэффициенты при соответствующих степенях и составить систему линейных уравнений. В картинках процесс выглядит так:
http://mathprofi.ru/h/kak_reshit_neodnorodnoe_uravnenie_vtorogo_poryadka_clip_image049.jpg

http://mathprofi.ru/h/kak_reshit_neodnorodnoe_uravnenie_vtorogo_poryadka_clip_image051.gif

Чтобы было еще проще, новичкам рекомендую предварительно сгруппировать подобные слагаемые:
http://mathprofi.ru/h/kak_reshit_neodnorodnoe_uravnenie_vtorogo_poryadka_clip_image053.gif, и только потом составлять систему.

В данном случае система получилась очень простой, и многие из читателей справятся с ней даже устно.

Подставляем найденные значения http://mathprofi.ru/h/kak_reshit_neodnorodnoe_uravnenie_vtorogo_poryadka_clip_image032_0000.gif в наш исходный подбор частного решения http://mathprofi.ru/h/kak_reshit_neodnorodnoe_uravnenie_vtorogo_poryadka_clip_image030_0002.gif:
http://mathprofi.ru/h/kak_reshit_neodnorodnoe_uravnenie_vtorogo_poryadka_clip_image056.gif

Таким образом, подобранное частное решение неоднородного уравнения:
http://mathprofi.ru/h/kak_reshit_neodnorodnoe_uravnenie_vtorogo_poryadka_clip_image058.gif

3) Запишем общее решение неоднородного уравнения: 
http://mathprofi.ru/h/kak_reshit_neodnorodnoe_uravnenie_vtorogo_poryadka_clip_image060.gif

Ответ: общее решение: http://mathprofi.ru/h/kak_reshit_neodnorodnoe_uravnenie_vtorogo_poryadka_clip_image062.gif

Пример 3

Найти общее решение неоднородного дифференциального уравнения.

http://mathprofi.ru/h/kak_reshit_neodnorodnoe_uravnenie_vtorogo_poryadka_clip_image080.gif

Решение: 
1) Найдем общее решение соответствующего однородного уравнения:
http://mathprofi.ru/h/kak_reshit_neodnorodnoe_uravnenie_vtorogo_poryadka_clip_image082.gif

Составим и решим характеристическое уравнение:
http://mathprofi.ru/h/kak_reshit_neodnorodnoe_uravnenie_vtorogo_poryadka_clip_image084.gif
http://mathprofi.ru/h/kak_reshit_neodnorodnoe_uravnenie_vtorogo_poryadka_clip_image086.gif
http://mathprofi.ru/h/kak_reshit_neodnorodnoe_uravnenie_vtorogo_poryadka_clip_image088.gif 
http://mathprofi.ru/h/kak_reshit_neodnorodnoe_uravnenie_vtorogo_poryadka_clip_image090.gif, http://mathprofi.ru/h/kak_reshit_neodnorodnoe_uravnenie_vtorogo_poryadka_clip_image092.gif – получены различные действительные корни, среди которых нет нуля, поэтому общее решение: http://mathprofi.ru/h/kak_reshit_neodnorodnoe_uravnenie_vtorogo_poryadka_clip_image094.gif.

2) Выясняем, в каком виде нужно искать частное решение http://mathprofi.ru/h/kak_reshit_neodnorodnoe_uravnenie_vtorogo_poryadka_clip_image009_0007.gif?

Сначала смотрим на правую часть и выдвигаем первую гипотезу: раз в правой части находится экспонента, умноженная на константу http://mathprofi.ru/h/kak_reshit_neodnorodnoe_uravnenie_vtorogo_poryadka_clip_image097.gif, то частное решение, по идее, нужно искать в виде http://mathprofi.ru/h/kak_reshit_neodnorodnoe_uravnenie_vtorogo_poryadka_clip_image099.gif

Далее смотрим на корни характеристического уравнения http://mathprofi.ru/h/kak_reshit_neodnorodnoe_uravnenie_vtorogo_poryadka_clip_image090_0000.gif, http://mathprofi.ru/h/kak_reshit_neodnorodnoe_uravnenie_vtorogo_poryadka_clip_image092_0000.gif, найденные в предыдущем пункте. Это два действительных корня, среди которых нет нуля. То есть, частное решение дифференциального уравнения следует искать в виде:
http://mathprofi.ru/h/kak_reshit_neodnorodnoe_uravnenie_vtorogo_poryadka_clip_image101.gif, где http://mathprofi.ru/h/kak_reshit_neodnorodnoe_uravnenie_vtorogo_poryadka_clip_image103.gif – пока еще неизвестный коэффициент, который предстоит найти.

Используем правило дифференцирования произведения http://mathprofi.ru/h/kak_reshit_neodnorodnoe_uravnenie_vtorogo_poryadka_clip_image105.gif. Найдем первую и вторую производную:
http://mathprofi.ru/h/kak_reshit_neodnorodnoe_uravnenie_vtorogo_poryadka_clip_image107.gif
http://mathprofi.ru/h/kak_reshit_neodnorodnoe_uravnenie_vtorogo_poryadka_clip_image109.gif

Подставим http://mathprofi.ru/h/kak_reshit_neodnorodnoe_uravnenie_vtorogo_poryadka_clip_image111.gif, http://mathprofi.ru/h/kak_reshit_neodnorodnoe_uravnenie_vtorogo_poryadka_clip_image113.gif и http://mathprofi.ru/h/kak_reshit_neodnorodnoe_uravnenie_vtorogo_poryadka_clip_image115.gif в левую часть неоднородного уравнения:
http://mathprofi.ru/h/kak_reshit_neodnorodnoe_uravnenie_vtorogo_poryadka_clip_image117.gif

Что сделано? Подстановка, упрощение, сокращение, и в конце – приравнивание к исходной правой части http://mathprofi.ru/h/kak_reshit_neodnorodnoe_uravnenie_vtorogo_poryadka_clip_image119.gif.

Из последнего равенства http://mathprofi.ru/h/kak_reshit_neodnorodnoe_uravnenie_vtorogo_poryadka_clip_image121.gif автоматически получаем http://mathprofi.ru/h/kak_reshit_neodnorodnoe_uravnenie_vtorogo_poryadka_clip_image123.gif.
Найденное значение http://mathprofi.ru/h/kak_reshit_neodnorodnoe_uravnenie_vtorogo_poryadka_clip_image123_0000.gif подставляем в наш исходный подбор http://mathprofi.ru/h/kak_reshit_neodnorodnoe_uravnenie_vtorogo_poryadka_clip_image111_0000.gif.

Таким образом, частное решение: http://mathprofi.ru/h/kak_reshit_neodnorodnoe_uravnenie_vtorogo_poryadka_clip_image126.gif

3) Составляем общее решение неоднородного уравнения:
http://mathprofi.ru/h/kak_reshit_neodnorodnoe_uravnenie_vtorogo_poryadka_clip_image128.gif

Ответ: общее решение: http://mathprofi.ru/h/kak_reshit_neodnorodnoe_uravnenie_vtorogo_poryadka_clip_image130.gif

Пример 4

Найти частное решение неоднородного уравнения, соответствующее заданным начальным условиям.
http://mathprofi.ru/h/kak_reshit_neodnorodnoe_uravnenie_vtorogo_poryadka_clip_image151.gif, http://mathprofi.ru/h/kak_reshit_neodnorodnoe_uravnenie_vtorogo_poryadka_clip_image153.gif, http://mathprofi.ru/h/kak_reshit_neodnorodnoe_uravnenie_vtorogo_poryadka_clip_image155.gif

Алгоритм решения полностью сохраняется, но в конце добавляется дополнительный пункт.

Решение: 
1) Найдем общее решение соответствующего однородного уравнения:
http://mathprofi.ru/h/kak_reshit_neodnorodnoe_uravnenie_vtorogo_poryadka_clip_image157.gif
Характеристическое уравнение:
http://mathprofi.ru/h/kak_reshit_neodnorodnoe_uravnenie_vtorogo_poryadka_clip_image159.gif
http://mathprofi.ru/h/kak_reshit_neodnorodnoe_uravnenie_vtorogo_poryadka_clip_image161.gif
http://mathprofi.ru/h/kak_reshit_neodnorodnoe_uravnenie_vtorogo_poryadka_clip_image163.gif – получены кратные действительные корни, поэтому общее решение:
http://mathprofi.ru/h/kak_reshit_neodnorodnoe_uravnenie_vtorogo_poryadka_clip_image165.gif

2) Выясняем, в каком виде нужно искать частное решение http://mathprofi.ru/h/kak_reshit_neodnorodnoe_uravnenie_vtorogo_poryadka_clip_image009_0008.gif. Смотрим на правую часть неоднородного уравнения http://mathprofi.ru/h/kak_reshit_neodnorodnoe_uravnenie_vtorogo_poryadka_clip_image168.gif, и сразу появляется первая версия подбора: http://mathprofi.ru/h/kak_reshit_neodnorodnoe_uravnenie_vtorogo_poryadka_clip_image170.gif.

Далее смотрим на корни характеристического уравнения: http://mathprofi.ru/h/kak_reshit_neodnorodnoe_uravnenie_vtorogo_poryadka_clip_image163_0000.gif – действительные кратные корни.  Частное решение следует искать в виде: 
http://mathprofi.ru/h/kak_reshit_neodnorodnoe_uravnenie_vtorogo_poryadka_clip_image174.gif
Ищем неизвестный коэффициент http://mathprofi.ru/h/kak_reshit_neodnorodnoe_uravnenie_vtorogo_poryadka_clip_image103_0000.gif.

Найдем первую и вторую производную:
http://mathprofi.ru/h/kak_reshit_neodnorodnoe_uravnenie_vtorogo_poryadka_clip_image177.gif
http://mathprofi.ru/h/kak_reshit_neodnorodnoe_uravnenie_vtorogo_poryadka_clip_image179.gif

Подставим http://mathprofi.ru/h/kak_reshit_neodnorodnoe_uravnenie_vtorogo_poryadka_clip_image009_0009.gif, http://mathprofi.ru/h/kak_reshit_neodnorodnoe_uravnenie_vtorogo_poryadka_clip_image181.gif и http://mathprofi.ru/h/kak_reshit_neodnorodnoe_uravnenie_vtorogo_poryadka_clip_image040_0000.gif в левую часть неоднородного уравнения и максимально упростим выражение:
http://mathprofi.ru/h/kak_reshit_neodnorodnoe_uravnenie_vtorogo_poryadka_clip_image183.gif
В самом конце после упрощений приписываем исходную правую часть http://mathprofi.ru/h/kak_reshit_neodnorodnoe_uravnenie_vtorogo_poryadka_clip_image185.gif.

Из последнего равенства http://mathprofi.ru/h/kak_reshit_neodnorodnoe_uravnenie_vtorogo_poryadka_clip_image187.gif следует:
http://mathprofi.ru/h/kak_reshit_neodnorodnoe_uravnenie_vtorogo_poryadka_clip_image189.gif 
Таким образом: http://mathprofi.ru/h/kak_reshit_neodnorodnoe_uravnenie_vtorogo_poryadka_clip_image191.gif.

3) Составим общее решение неоднородного уравнения: 
http://mathprofi.ru/h/kak_reshit_neodnorodnoe_uravnenie_vtorogo_poryadka_clip_image193.gif

4) Найдем частное решение, удовлетворяющее заданным начальным условиям http://mathprofi.ru/h/kak_reshit_neodnorodnoe_uravnenie_vtorogo_poryadka_clip_image153_0000.gif, http://mathprofi.ru/h/kak_reshit_neodnorodnoe_uravnenie_vtorogo_poryadka_clip_image155_0000.gif

Сначала берём найденное общее решение http://mathprofi.ru/h/kak_reshit_neodnorodnoe_uravnenie_vtorogo_poryadka_clip_image195.gif и применяем к нему первое начальное условие http://mathprofi.ru/h/kak_reshit_neodnorodnoe_uravnenie_vtorogo_poryadka_clip_image153_0001.gif:
http://mathprofi.ru/h/kak_reshit_neodnorodnoe_uravnenie_vtorogo_poryadka_clip_image197.gif
Согласно начальному условию: http://mathprofi.ru/h/kak_reshit_neodnorodnoe_uravnenie_vtorogo_poryadka_clip_image199.gif – получаем первое уравнение.

Далее находим производную от общего решения:
http://mathprofi.ru/h/kak_reshit_neodnorodnoe_uravnenie_vtorogo_poryadka_clip_image201.gif и применяем к найденной производной второе начальное уравнение http://mathprofi.ru/h/kak_reshit_neodnorodnoe_uravnenie_vtorogo_poryadka_clip_image155_0001.gif:
http://mathprofi.ru/h/kak_reshit_neodnorodnoe_uravnenie_vtorogo_poryadka_clip_image203.gif
Согласно второму начальному условию: http://mathprofi.ru/h/kak_reshit_neodnorodnoe_uravnenie_vtorogo_poryadka_clip_image205.gif – получаем второе уравнение.

Составим и решим систему:
http://mathprofi.ru/h/kak_reshit_neodnorodnoe_uravnenie_vtorogo_poryadka_clip_image207.gif

Подставим найденные значения констант http://mathprofi.ru/h/kak_reshit_neodnorodnoe_uravnenie_vtorogo_poryadka_clip_image209.gif,  в общее решение http://mathprofi.ru/h/kak_reshit_neodnorodnoe_uravnenie_vtorogo_poryadka_clip_image211.gif

Ответ: частное решение: http://mathprofi.ru/h/kak_reshit_neodnorodnoe_uravnenie_vtorogo_poryadka_clip_image213.gif

Пример 5

Найти общее решение неоднородного уравнения
http://mathprofi.ru/h/kak_reshit_neodnorodnoe_uravnenie_vtorogo_poryadka_clip_image008.gif

Решение: 
1) Найдем общее решение соответствующего однородного уравнения:
http://mathprofi.ru/h/kak_reshit_neodnorodnoe_uravnenie_vtorogo_poryadka_clip_image010.gif
Характеристическое уравнение:
http://mathprofi.ru/h/kak_reshit_neodnorodnoe_uravnenie_vtorogo_poryadka_clip_image012.gif
http://mathprofi.ru/h/kak_reshit_neodnorodnoe_uravnenie_vtorogo_poryadka_clip_image014.gif
http://mathprofi.ru/h/kak_reshit_neodnorodnoe_uravnenie_vtorogo_poryadka_clip_image016_0002.gif
http://mathprofi.ru/h/kak_reshit_neodnorodnoe_uravnenie_vtorogo_poryadka_clip_image018.gif – получены сопряженные комплексные корни, поэтому общее решение:
http://mathprofi.ru/h/kak_reshit_neodnorodnoe_uravnenie_vtorogo_poryadka_clip_image020.gif.

2) Частное решение неоднородного уравнения ищем в «обычном» виде: http://mathprofi.ru/h/kak_reshit_neodnorodnoe_uravnenie_vtorogo_poryadka_clip_image022.gif 
Выясним, чему равны коэффициенты http://mathprofi.ru/h/kak_reshit_neodnorodnoe_uravnenie_vtorogo_poryadka_clip_image024.gif.

Найдем производные:
http://mathprofi.ru/h/kak_reshit_neodnorodnoe_uravnenie_vtorogo_poryadka_clip_image026.gif
http://mathprofi.ru/h/kak_reshit_neodnorodnoe_uravnenie_vtorogo_poryadka_clip_image028_0000.gif

Подставим http://mathprofi.ru/h/kak_reshit_neodnorodnoe_uravnenie_vtorogo_poryadka_clip_image030_0003.gif и http://mathprofi.ru/h/kak_reshit_neodnorodnoe_uravnenie_vtorogo_poryadka_clip_image032_0001.gif в левую часть неоднородного уравнения:
http://mathprofi.ru/h/kak_reshit_neodnorodnoe_uravnenie_vtorogo_poryadka_clip_image034.gif
(После подстановки и максимальных упрощений приписываем правую часть: http://mathprofi.ru/h/kak_reshit_neodnorodnoe_uravnenie_vtorogo_poryadka_clip_image036.gif)

Из последнего равенства http://mathprofi.ru/h/kak_reshit_neodnorodnoe_uravnenie_vtorogo_poryadka_clip_image038.gif составим и решим систему:
http://mathprofi.ru/h/kak_reshit_neodnorodnoe_uravnenie_vtorogo_poryadka_clip_image040_0001.gif

Таким образом, подобранное частное решение: http://mathprofi.ru/h/kak_reshit_neodnorodnoe_uravnenie_vtorogo_poryadka_clip_image042_0000.gif.

3) Составим общее решение неоднородного уравнения: 
http://mathprofi.ru/h/kak_reshit_neodnorodnoe_uravnenie_vtorogo_poryadka_clip_image044.gif

Ответ: общее решение: http://mathprofi.ru/h/kak_reshit_neodnorodnoe_uravnenie_vtorogo_poryadka_clip_image046.gif

 

 

2.Самостоятельное выполнение заданий.

 

Вариант 1

Вариант 2

Найти общее решение дифференциальных уравнений

1)y+ y= x2 + 4

1)y- 2y= 2x + 1

2)y- 5y+ 6y = e4x

2)y- y- 6y = e3x

3)y- 8y+ 15y = 3sin5x

3)y- 5y- 24y = cos3x

Найти частное решение дифференциального уравнения

y+ 2y = 3x – 4, у(0) = 1, y(0) = 2

y+ 7y = 2x2 + x, y(0) = 0, y(0) = 0

 

3.Вывод.

 

 

 

 

Практическая работа №17 "Определение сходимости числовых рядов"

 

Цель работы: закрепление практических навыков определения сходимости числовых рядов.

 Ход работы:

1)повторение теоретического материала;

2)выполнение заданий;

3)вывод.

 

1.Краткое содержание теоретического материала.

          Выражение

                                                                                          (1)

называется числовым рядом, числа - членами ряда,  - общим членом ряда.

    Сумма n первых членов ряда    называется  частичной суммой этого ряда.

     Ряд называется сходящимся, если последовательность его частичных сумм имеет конечный предел:  Значение S называется суммой ряда.

   Если ряд не сходиться, то он называется расходящимся.

  Часто интерес представляет не сама сумма того или иного ряда, а лишь факт сходимости или расходимости этого ряда. Установление того факта нередко основывается на сравнении исследуемых рядов с рядами, о сходимости или расходимости которых заранее известно.

    В качестве рядов сравнения часто выбирают:

а) бесконечно убывающую геометрическую прогрессию:

             которая сходится и имеет сумму

б) гармонический ряд , являющийся расходящимся;

в) обобщенно-гармоничный ряд

Левая фигурная скобка:      при p>1

- сходится

    при p1

- расходится

                                                              

 

                                    Основные свойства рядов

 

        Пусть дан ряд (1). Ряд

                                                                                                               (2)

называется  остатком ряда (1).

 

 

  • Если сходится ряд (1), то сходится и n-й остаток этого ряда (2) и наоборот.

 

  • Если сходится ряд (1), то сходится и ряд

,

причем сумма последнего ряда равна aS.

 

  • Если сходятся ряды

,       ,

имеющие соответственно суммы S и t, то сходится и ряд

,

причем сумма последнего ряда равна S + t.

 

 Признаки сходимости рядов

 

            Необходимый  признак  сходимости ряда.

 

Если ряд (1) сходится, то

                                                  .                                                                   (3)

 

Этот признак сходимости является необходимым, но не достаточным.

Так, для гармонического ряда    , но тем не менее ряд расходиться.

 

            Достаточный  признак  расходимости.

 

Если для ряда (1) предел  или не существует, то ряд расходится.

 

 

ü  Ряды с положительными членами

 

²  Признак сравнения рядов (первый признак сравнения).

 

            Пусть даны два ряда

,

и     

,

причем 

Тогда:

 

а) если сходится ряд , то будет сходиться и ряд  ;

 

б) если расходится ряд  , то будет расходиться и ряд  .

 

²  Признак  сравнения в предельной форме (второй признак сравнения).

 

          Если существует конечный и отличный от нуля предел

, то оба ряда  и   одновременно сходятся или одновременно расходятся.

 

²  Признак  Даламбера.

 

          Если для ряда (1)существует

                                                                                                   (4)

то при D>1 ряд расходится, при D<1 ряд сходится, при D=1 вопрос остается нерешенным.

 

 

²  Признак  Коши.

 

           Если для ряда (1) существует

                                                                                                                              (5)

то  при C < 1 ряд сходится, при  С > 1 ряд расходится, при C = 1 Вопрос остается нерешенным.

 

 

²  Интегральный признак Коши. 

 

            Если  F(x) при  - непрерывная, положительная и монотонно убывающая функция,

 то ряд , где , сходиться или расходится в зависимости от того, сходиться или расходится

                                                    .                                                                                   (6)

В случае сходимости ряда остаток  удовлетворяет неравенству: .

Пример1    Исследовать сходимость ряда

 

 Решение:

         Сравним ряд с рядом, у которого  (т.е. с бесконечно убывающей геометрической прогрессией).Применим первый признак сравнения рядов.

          (n = 1, 2, 3,

Так как ряд  сходиться, то сходиться и данный ряд.

 

Пример2 Исследовать на сходимость ряд

Решение:

Сравним ряд с гармоническим рядом, у которого .

Применим второй признак сравнения рядов:

.

Так как предел конечен и отличен то нуля, ряд  расходится, то расходится и данный ряд.

 

Пример3   Исследовать на сходимость ряд .

 

Решение:

      Применим признак Даламбера; имеем  ,

Так как D<1, то ряд сходиться.

Пример4 Исследовать на сходимость ряд

Решение:

 

              Признак Коши для этого ряда дает:

.

Так как C < 1, ряд сходиться.

 

ü  Знакопеременные ряды

 

        Знакопеременным  рядом называется ряд, членами которого являются действительные числа произвольного знака.

 

Знакопеременный ряд  сходиться, если сходится ряд

                                                                                                                (7)

В этом случае исходный ряд называется абсолютно-сходящимся.

Операции над абсолютно-сходящимися рядами:

 

  • Если в абсолютно-сходящемся ряде произвольным образом переставить члены, то полученный

 ряд так же будет абсолютно сходиться, а сумма его будет равна сумме исходного.

 

  • Абсолютно-сходящиеся ряды можно перемножать.

 Пусть даны два абсолютно-сходящихся ряда:

и     

           

Соответственно с суммами S и t.

Тогда ряд, членами которого являются все произведения любого члена первого ряда на любой член второго ряда, также сходится абсолютно и сумма его равна произведению St.

 

ü  Знакочередующиеся  ряды

 

        Знакопеременный ряд называется знакочередующимся, если соседние его члены имеют различные знаки.

                                                                                             (8)

где     (n = 1, 2, 3, …)

 

²  Признак сходимости Лейбница

 

            Знакочередующийся ряд (8) сходится, если выполняются следующие два условия:

-  абсолютные величины его членов монотонно убывают: ;

 

-   .

 

            Для знакочередующегося ряд (8), удовлетворяющего признаку сходимости Лейбница, остаток   не превосходит абсолютной величины своего первого члена, т.е.

 

           Признак Лейбница является не только достаточным, но  и необходимым признаком сходимости для знакочередующихся рядов с монотонно убывающими членами.

 

Алгоритм исследования на сходимость знакопеременных рядов

 

°         Исследовать на сходимость ряд, составленный из модулей членов данного ряда, используя

какой-либо признак сравнения.

 

°         Сделать вывод об абсолютной или условной сходимости этого ряда.

 

 

°         Выяснить, сходится ли данный знакочередующийся ряд, применяя признак Лейбница.

 Для этого:

 

-  Проверить, выполняется ли равенство для абсолютных величин членов ряда

 

-  Найти предел общего члена ряда

 

-  Сделать вывод о сходимости данного исходного ряда

 

Пример5  Исследовать на сходимость ряд

Решение:

 

            Этот ряд знакочередующийся.

Первое условие Лейбница не выполняется: 1,1 > 1,02 > 1,003 > …

Проверим выполнение второго условия:

Так как ряд расходится.

Пример6 Исследовать на сходимость ряд

Решение:

 

           Этот ряд знакочередующийся. Так как

       … ,

то

  … .

Первое условие признак Лейбница выполняется.

Далее, 

т.е.  выполняется и второе условие. Значит, данный ряд сходится

 

 

 

2.Самостоятельное выполнение заданий

Вариант 1

Вариант 1

1.Исследовать сходимость рядов

+

+

2.Исследовать ряд на сходимость, применяя признаки сравнения

3.Исследовать ряды на сходимость, применяя признак Даламбера

 

3.Вывод.

 

 

 

Практическая работа №18 "Исследование на сходимость степенных рядов"

 

Цель работы: закрепление практических навыков определения сходимости степенных рядов.

 Ход работы:

1)повторение теоретического материала;

2)выполнение заданий;

3)вывод.

 

1.Краткое содержание теоретического материала.

 

 Функциональные ряды

 

           Выражение

                                                                                               (1)

 

называется функциональным рядом относительно переменной x.

 

          Совокупность всех значений переменной x, при которой функции  определены  и ряд    сходится, называется областью  сходимости функционального ряда.

Если значение  принадлежит области сходимости ряда , то говорят о

сумме этого функционального ряда в точке : .

 

           Сумма функционального ряда сама является функцией переменной x.

 

 

 Степенные ряды.

 

          Функциональный ряд вида

                                              (2)

где - действительные числа, называется степенным относительно переменной x рядом.

 

          Для любого степенного ряда существует такое неотрицательное число R, что этот ряд сходится абсолютно при , расходится при . Поведение ряда при  подлежит дальнейшему анализу.

 

          Число R называется радиусом сходимости данного степенного ряда.

 

          Область значений переменной  x: -R<x<Rинтервалом сходимости.

 

          Если R = 0, то ряд сходится лишь при x = 0, если же R=, то ряд сходится при любом действительном x.

           Интервал сходимости степного ряда находят, применяя признак Даламбера к ряду, составленному из абсолютных величин членов исходного ряда, т.е. находят  ,

по признаку Даламбера ряд сходится, если <1. Степенной ряд сходится равномерно в любом замкнутом интервале, содержащемся в его интервале сходимости.

 

         При  сумма степенного ряда есть непрерывная и сколько угодно раз дифференцируемая функция от x. Степенной ряд можно почленно дифференцировать при  и интегрировать на любом замкнутом интервале, содержащемся в интервале (-R, R). Получающиеся в результате почленного интегрирования от 0 до x ряды имеют тот же радиус сходимости R.

Пример1 Исследовать на сходимость ряд   

Решение:

 

         Ряд  сходится при x > 1, и расходиться при x ≤ 1. Следовательно, область сходимости этого ряда описывается неравенством x > 1.

 

Пример2 Исследовать на сходимость ряд

Решение:

 

         Члены функционального ряда  при любом x меньше

соответствующих членов ряда “обратных квадратов“:.

Так как последующий ряд сходится, будет сходиться и исходный ряд при любом x; его

областью сходимости является множество всех действительных чисел.

 

Пример3 Исследовать на сходимость ряд рассмотрим ряд 

Решение:

 

            Поскольку sin x ≤ 1, члены этого ряда не меньше соответствующих членов гармоничного ряда, начиная с третьего:

Следовательно, исходный ряд не сходится ни при каком значении x. Область сходимости – Ø.

 

Пример4 Исследовать на сходимость ряд

Решение:

 

Применим признак Даламбера:

Так как предел < 1 и не зависит от x, то ряд сходится при всех значениях x.

 

Приме5   Исследовать на сходимость ряд 

Решение:

 

        Здесь

Стало быть, ряд расходится при всех значениях x, кроме x = 0.

 

Пример6 Исследовать на сходимость ряд 

Решение:

 

        Применим признак Даламбера

.

Ряд сходится, если ,   т.е. если . Исследуем сходимость ряда на концах промежутка. При  получим знакочередующийся ряд  Применим признак Лейбница:

1)    

2)    

Оба условия признака Лейбница выполняются, следовательно, ряд сходится.

При х = -  имеем

Этот ряд расходится, так как членами его являются члены гармонического ряда, умноженные на -1. таким образом, исходный степенной ряд сходится при  .

 

2.Самостоятельное выполнение заданий

Вариант 1

Вариант 2

Найти интервал сходимости ряда и исследовать его сходимость на концах интервала

1)

 

1)

 

 

2)

2)

 

3)               

 

3)        

 

4)

4)         

 

3.Вывод.

 

 

 

 

Практическая работа №19 "Разложение функций в степенные ряды"

 

Цель работы: закрепление практических навыков разложения функций в степенные ряды.

 Ход работы:

1)повторение теоретического материала;

2)выполнение заданий;

3)вывод.

 

1.Краткое содержание теоретического материала.

Ø  Ряды Тейлора и Маклорена


Если функция (x) имеет непрерывные производные вплоть до (n+1)-го порядка,

 то ее можно разложить в степенной ряд по формуле Тейлора:

http://www.math24.ru/images/10ser1.gif

где Rn − остаточный член в форме Лагранжа определяется выражением

http://www.math24.ru/images/10ser2.gif

Если приведенное разложение сходится в некотором интервале x, т.е. http://www.math24.ru/images/10ser3.gif,

 то оно называется рядом Тейлора, представляющим разложение функции (x) в точке a

Если a = 0, то такое разложение называется рядом Маклорена:

http://www.math24.ru/images/10ser4.gif

Разложение некоторых функций в ряд Маклорена

  http://www.math24.ru/images/10ser5.gif

 http://www.math24.ru/images/10ser6.gif

  http://www.math24.ru/images/10ser7.gif

  http://www.math24.ru/images/10ser8.gif

  http://www.math24.ru/images/10ser9.gif

   Пример 1

Найти ряд Маклорена для функции http://www.math24.ru/images/10ser10.gif.


Решение.

Воспользуемся тригонометрическим равенством http://www.math24.ru/images/10ser11.gif
Поскольку ряд Маклорена для cos x имеет вид http://www.math24.ru/images/10ser12.gif, то можно записать

      http://www.math24.ru/images/10ser13.gif

Отсюда следует:

      http://www.math24.ru/images/10ser14.gif

   Пример 2

Разложить в ряд Тейлора функцию http://www.math24.ru/images/10ser15.gif в точке x = 1.


Решение.

Вычислим производные:

      http://www.math24.ru/images/10ser16.gif

Видно, что http://www.math24.ru/images/10ser17.gif для всех n ≥ 3. Для x = 1 получаем значения:

      http://www.math24.ru/images/10ser18.gif

Следовательно, разложение в ряд Тейлора имеет вид

      http://www.math24.ru/images/10ser19.gif

   Пример 3

Найти разложение в ряд Маклорена функции e kxk − действительное число.


Решение.

Вычислим производные:

      http://www.math24.ru/images/10ser20.gif

Тогда в точке x = 0 получаем

      http://www.math24.ru/images/10ser21.gif

Следовательно, разложение данной функции в ряд Маклорена выражается формулой

      http://www.math24.ru/images/10ser22.gif

   Пример 4

Найти разложение в ряд Тейлора кубической функции x3 в точке x = 2.


Решение.

Обозначим http://www.math24.ru/images/10ser23.gif. Тогда

      http://www.math24.ru/images/10ser24.gif

и далее http://www.math24.ru/images/10ser25.gif для всех x ≥ 4. 
В точке x = 2, соответственно, получаем

      http://www.math24.ru/images/10ser26.gif

Таким образом, разложение в ряд Тейлора имеет вид

    http://www.math24.ru/images/10ser27.gif

   Пример 5

Найти разложение в ряд Маклорена функции http://www.math24.ru/images/10ser28.gif.

Решение.

Пусть http://www.math24.ru/images/10ser29.gif, где μ − действительное число, и x ≠ −1. Производные будут равны

      http://www.math24.ru/images/10ser30.gif

При x = 0, соответственно, получаем

      http://www.math24.ru/images/10ser31.gif

Следовательно, разложение в ряд записывается в виде

      http://www.math24.ru/images/10ser32.gif

Полученное выражение называется биномиальным рядом

   Пример 6

Найти разложение в ряд Маклорена функции http://www.math24.ru/images/10ser33.gif.

Решение.

Используя формулу биномиального ряда, найденную в предыдущем примере,

и подставляя http://www.math24.ru/images/10ser34.gif, получаем

      http://www.math24.ru/images/10ser35.gif

Ограничиваясь первыми 3-мя членами, разложение можно записать в виде

      http://www.math24.ru/images/10ser36.gif

 

Приближённое вычисление определённых интегралов

Ряды применяются также для приближённого вычисления интегра-

лов в случаях, когда первообразная не выражается через элементарные

функции или нахождение первообразной сложно.

Пусть требуется вычислить  с точностью до ε > 0. Если по-

дынтегральную функцию f (x) можно разложить в ряд и интервал сходи-

мости включает в себя отрезок [a, b],  то для вычисления заданного инте-

грала можно воспользоваться свойством почленного интегрирования этого

ряда. Ошибку вычислений определяют так же, как и при вычислении зна-

чений функций.

Пример 7. Вычислить интеграл  с точностью до ε = 0,001.

Решение. Разложим подынтегральную функцию в ряд Маклорена,

заменяя x на (-x2) в формуле   (ряд Маклорена для функции ех)

  х (-).

Интегрируя обе части этого равенства на отрезке  , лежащем внутри интервала сходимости (− ∞;+∞), получим:

-

Получим знакочередующийся ряд. Так как

2.Самостоятельное выполнение заданий.

Вариант 1

Вариант 1

1.Разложить в ряд Тейлора по степеням х функци

у = sin3x

y = cos3x

2.Вычислить с точностью до 0,001путём разложения подинтегральной функции в ряд Маклорена

 

1)dx

1)

2)

2)

3)

3)

 

3.Вывод.

 

 

 

 

Практическая работа №20 "Решение задач с применением формул алгебры логики"

 

Цель работы: закрепление практических навыков решения задач с применением формул алгебры логики.

 Ход работы:

1)повторение теоретического материала;

2)выполнение заданий;

3)вывод.

 

1.Краткое содержание теоретического материала.

Элементы теории множеств

 

1. Логические символы

 

Квантор  - заменяет выражение "для любого", "для произвольного", "для какого бы ни было".

 

Квантор  - заменяет выражение "существует", "найдется".

 

Запись  (импликация) означает, что из справедливости высказывания A вытекает справедливость высказывания B. Если, кроме того, из справедливости высказывания B вытекает справедливость A, то записываем . Если  , то высказывание B является необходимым и достаточным условием для того, чтобы выполнялось высказывание A.

 

Если предложения A и B справедливы одновременно, то записываем  . Если же справедливо хотя бы одно из предложений A или B, то записываем  .

 

2. Операции над множествами

 

Математическое понятие множества элементов принимается в качестве интуитивного. Множество задается правилом или признаком, согласно которому определяем, принадлежит ли данный элемент множеству или не принадлежит.

 

Множество обозначают символом A = {x}, где x - общее наименование элементов множества A. Часто множество записывают в виде A = {a, b, c, ...}, где в фигурных скобках указаны элементы множества A. Будем пользоваться обозначениями:

 

                  N - множество всех натуральных чисел;

                  Z - множество всех целых чисел;

                  Q - множество всех рациональных чисел;

                  R - множество всех действительных чисел;

                  C - множество всех комплексных чисел;

                  Z0 - множество всех неотрицательных целых чисел.

 

Запись   означает, что элемент a принадлежит множеству A.

 

Запись    означает, что элемент a не принадлежит множеству A. Множество B, все элементы которого принадлежат множеству A, называется подмножеством множества A, и при этом записывают

(см. рис. 1).

 

 

 

Всегда , так как каждый элемент множества, естественно, принадлежит A. Пустое множество, т. е. множество, не содержащее ни одного элемента, обозначим символом  . Любое множество содержит пустое множество в качестве своего подмножества.

Если  , то A и B называются равными множествами, при этом записывают A = B.

Если  , то множество элементов множества А , не принадлежащих A, называется дополнением множества A к множеству  (см. рис. 2).

 

Дополнение множества A к множеству обозначают символом ; или просто CA, если известно, к какому множеству берется  дополнение. Таким образом,СА=

Если А⊂ℐ , то иногда дополнение множества B к множеству A называют разностью множеств A и B и обозначают A\B (см. рис. 3),      т. е.   

 

Пусть A и B - подмножества множества .

Объединением множеств A и B называется множество АВ(см. рис. 4)   

 

 

 

 

 

Аналогично, если   подмножества множества , то их объединением будет множество

   

Пересечением подмножеств A и B называется множество (см. рис. 5)

   

 

Рисунки 1-6 называются диаграммы Эйлера-Венна.

Аналогично, символом   обозначают пересечение подмножеств          множества ℐ,т.е.    

Два элемента a и b называются упорядоченной парой, если указано, какой из этих элементов первый, какой второй, при этом

    

Упорядоченную пару элементов a и b обозначают символом (a, b).

Аналогично определяется упорядоченная система из n элементов a1, a2, ..., an, которую обозначают символом (a1, a2, ..., an). Элементы a1, a2, ..., an называются координатами упорядоченной системы (a1, a2, ..., an).

Совокупность всевозможных упорядоченных пар (a, b), где, называется произведением множеств A и B и обозначается символом .

 

Аналогично, символом  обозначают произведение множеств , т. е. совокупность всевозможных упорядоченных систем (a1, a2, ..., an),

где   .

1.     Свойства операций над множествами

Пусть A, B и D - произвольные подмножества множества . Тогда непосредственно из определений объединения, пересечения и дополнения вытекают следующие предложения:

1.     Закон идемпотентности для объединения и пересечения множеств:

АА=А, АА=А

2.     Закон коммутативности:

АВ=ВА; АВ=ВА.

3.     Закон ассоциативности:

АD)= (АВ)D; АD)= (АВ)D

4.     Закон дистрибутивности:

АD)=(АВ)D); АD)=(АВ)D)

5.     Закон поглощения:

АD)=А, АD)=А

6.     Законы, описывающие свойства пустого и универсального множества по отношению к объединению и пересечению:

А⋃∅=А, А⋂∅=, А⋃ℐ=, А⋂ℐ

7.   Законы дополнения: А=ℐ, А⋂=∅

8.   Закон инволютивности дополнения:

9.   Закон Де Моргана: =, =

Истинность каждого тождества проще всего проверяется построением диаграмм Эйлера-Венна.

 

2.     Выполнить самостоятельно:

проверку следующих утверждений диаграммой Эйлера-Венна:

Вариант 1

Вариант 2

Вариант 3

 

3.Вывод.

 

 

 

Практическая работа №21 "Погрешности простейших арифметических действий"

 

Цель работы: закрепление практических навыков решения задач на нахождение погрешностей простейших арифметических действий.

 Ход работы:

1)повторение теоретического материала;

2)выполнение заданий;

3)вывод.

 

1.Краткое содержание теоретического материала.

Различают абсолютную и относительную погрешности. Пусть x - истинное значение величины, http://www.olga.uni-altai.ru/img/f1.jpg - её приближенное значение, принимаемое в расчетах. 
Величина http://www.olga.uni-altai.ru/img/f2.jpg называется 
абсолютной погрешностью числа http://www.olga.uni-altai.ru/img/f1.jpg. 
Точная верхняя грань множества значений http://www.olga.uni-altai.ru/img/f3.jpg, которое определяется найденным , http://www.olga.uni-altai.ru/img/f1.jpg и имеющейся информацией относительно x, называется 
предельной абсолютной погрешностью величины http://www.olga.uni-altai.ru/img/f1.jpg. 
Относительной погрешностью δ величины http://www.olga.uni-altai.ru/img/f1.jpg называется отношение её абсолютной погрешности к величине http://www.olga.uni-altai.ru/img/f1.jpg:   http://www.olga.uni-altai.ru/img/f4.jpg. 
Аналогично можно определить
 предельную относительную погрешность δ x числа http://www.olga.uni-altai.ru/img/f1.jpg:   http://www.olga.uni-altai.ru/img/f5.jpg 
Относительные погрешности чисел принято выражать в процентах, поэтому:   http://www.olga.uni-altai.ru/img/f6.jpg 
При записи приближённых чисел желательно указывать их точность, сообщая те границы, в которых это число может находиться: http://www.olga.uni-altai.ru/img/f1.jpg ± Δx.

Значащая цифра числа считается верной в узком смысле, если абсолютная погрешность (предельная) не превосходит половины единицы того разряда, в котором стоит данная цифра. 
В противном случае цифра считается 
сомнительной. 
Значащая цифра числа считается 
верной в широком смысле, если абсолютная погрешность (предельная) не превосходит единицы того разряда, в котором стоит данная цифра.

При записи чисел руководствуются следующим правилом: все цифры числа должны быть верными. 
Поэтому округление чисел, записанных в десятичной системе, производится 
по правилу первой отбрасываемой цифры:

  • Если первая из отбрасываемых цифр меньше 5, то оставляемые десятичные знаки сохраняются без изменения;
  • Если первая из отбрасываемых цифр больше 5, то последняя оставляемая цифра увеличивается на 1;
  • Если первая из отбрасываемых цифр равна 5, а за ней идут не нули, то последняя оставляемая цифра увеличивается на 1;
  • Если первая из отбрасываемых цифр равна 5, и все цифры, идущие за ней - нули, то последняя оставляемая цифра увеличивается на 1, если она нечетная, и остаётся без изменения, если - четная.

  Ошибки арифметических действий

Если f (x, y) = x + y, то Δx + y = Δx + Δy. 
Если f (x, y) = x - y, то Δx - y = Δx + Δy. 
Если f (x, y) = x · y, то Δxy = http://www.olga.uni-altai.ru/img/f7.jpg. 
Если f (x, y) = http://www.olga.uni-altai.ru/img/f8.jpg, то http://www.olga.uni-altai.ru/img/f9.jpg.

Из формул для абсолютных погрешностей суммы, разности, произведения и частного выводятся формулы для соответствующих относительных погрешностей. 
http://www.olga.uni-altai.ru/img/f10.jpg 
http://www.olga.uni-altai.ru/img/f11.jpg 
http://www.olga.uni-altai.ru/img/f12.jpg 
http://www.olga.uni-altai.ru/img/f13.jpg

Если f (x) = x n , то Δxn = http://www.olga.uni-altai.ru/img/f14.jpg. 
Если http://www.olga.uni-altai.ru/img/f15.jpg, то http://www.olga.uni-altai.ru/img/f16.jpg. 
δxn = n · δx. 
http://www.olga.uni-altai.ru/img/f17.jpg.

  Основные задачи теории приближённых вычислений

Прямая задача: указаны действия, которые нужно выполнить и заданы предльные погрешности. Требуется оценить погрешность результата.

Обратная задача: указаны действия, которые нужно выполнить, задана погрешность, которая допустима для результата. Требуется установить, какими должны быть погрешности исходных данных, чтобы полученный результат имел заданную точность.

 

2.Самостоятельное выполнение заданий.

Вариант 1

Вариант 2

1.Оцените относительную погрешность разности двух приближенных чисел a1 и a2 , если абсолютные погрешности этих чисел равны D1 и D2

a1 = 35,6,  a2= 35,7, D1=D2=0,05

a1 = 26,6,  a2= 26,7, D1=D2=0,05

2.Вычислите абсолютную погрешность в широком смысле произведения двух чисел a1  и  a2

a1 =3,3,  a2=35

a1 =3,5,  a2=23

3.Найти число верных знаков частного , если все цифры делимого и делителя верны

 

a1 =350a2=35

a1 =230a2=23

4.Со сколькими знаками нужно взять a , чтобы относительная погрешность была не более 1%. Указать наименьшее число знаков

 

a = 211/2

a = 231/2

 

3.Вывод.

 

 

 

 

Практическая работа №22 "Численное решение алгебраических уравнений"

 

Цель работы: закрепление практических навыков численного решения алгебраических уравнений.

 Ход работы:

1)повторение теоретического материала;

2)выполнение заданий;

3)вывод.

 

1.Краткое содержание теоретического материала.

 

Численные методы решения нелинейных уравнений с одним неизвестным

Для большинства уравнений вида f (x) = 0 (1), где f (x) - нелинейная функция одной переменной, не существует аналитических выражений (формул) для вычисления их корней. Поэтому приходится применять различные численные методы для отыскания корней уравнения f (x) = 0.         (1)

Задача нахождения корней уравнения (1) обычно состоит из двух этапов:

  1. отделение корня, т.е. установление промежутка, в котором находится корень, причём единственный.
  2. уточнение корня, т.е. вычисление приближённого значения корня с заданной точностью.


Для существования корня уравнения (1) на замкнутом промежутке достаточно, чтобы выполнялись условия теоремы Больцано-Коши.

Теорема Больцано-Коши:    если функция f(x) определена и непрерывна на замкнутом промежутке [a; b] и на концах его принимает значение разных знаков, f(a) · f(b) < 0, то между a и b найдётся по крайней мере один корень функции f(x), т.е. найдётся точка с, a < c < b, для f (с) = 0. 
Для единственности корня функции на замкнутом промежутке, если он существует, достаточно, чтобы функция f(x) была монотонной, т.е. f ' (x) сохраняет свой знак на промежутке.

Отделение корня можно произвести графически или аналитически. Таким образом, на 1 этапе нужно найти такой промежуток [a; b], чтобы f(a) · f(b) < 0 и f ' (x) сохраняла свой знак на [a; b].

Уточнение корня можно произвести одним из следующих методов.

1.      Метод половинного деления (метод дихотомии).

Пусть на отрезке [a; b] имеется только один корень уравнения (1). Найдем середину отрезка http://www.olga.uni-altai.ru/img/f18.jpg. Если f (с) = 0, то корень найден. В противном случае из двух отрезков [a; c] и [c; b] выбираем тот, в котором содержится корень.

http://www.olga.uni-altai.ru/img/f19.jpg

С выбранным промежутком делаем то же, что с исходным и т.д. 
Тогда, либо через конечное число делений отрезка пополам найдём точное значение корня, либо построим бесконечную последовательность вложенных отрезков:
[a; b] http://www.olga.uni-altai.ru/img/f20.jpg [a1; b1] http://www.olga.uni-altai.ru/img/f20.jpg ... http://www.olga.uni-altai.ru/img/f20.jpg [an; bn], длины которых стремятся к нулю.

Как только |bn - an| < E, где Е - заданная точность, то в качестве приближённого значения корня можно взять середину этого отрезка: http://www.olga.uni-altai.ru/img/f21.jpg.

  Метод хорд.

Пусть на отрезке [a; b] имеется единственный корень, т.е. f(a) · f(b) < 0; 
f ' (x) сохраняет свой знак на [a; b]; 
f '' (x) сохраняет свой знак на [a; b].

Заменим дугу кривой y = f (x) на отрезке [a; b] хордой, проходящей через точки (a; f (a)) и (b; f (b)). Абсцисса точки пересечения хорды с осью Ох есть приближение к корню уравнения (1). Обозначим её через x1.

Корень уравнения (1) будет находиться между x1 и одним из концов отрезка [a; b] в зависимости от свойств функции. 
Выбрав часть отрезка, содержащую корень, осуществим такое же построение, и получим точку х2, и т.д. В результате получим последовательность приближённых значений, монотонно сходящуюся к точному значению корня.

http://www.olga.uni-altai.ru/img/f22.jpg


Если f ' (x) · f '' (x) > 0 для любого xhttp://www.olga.uni-altai.ru/img/f23.jpg[a; b], то для вычисления х1, х2, ... , хi, ... используются следующие формулы: 
http://www.olga.uni-altai.ru/img/f24.jpg

Если f ' (x) · f '' (x) < 0 для любого xhttp://www.olga.uni-altai.ru/img/f23.jpg[a; b], то для вычисления х1, х2, ... , хi, ... используются следующие формулы: 
http://www.olga.uni-altai.ru/img/f25.jpg 
      или 
http://www.olga.uni-altai.ru/img/f25a.jpg

Заканчиваем процесс уточнения корня, когда расстояние между очередными приближениями хn и xn-1 станет меньше заданной погрешности E: |хn - xn-1| < E или когда значение функции |f (xn)| < E.

  Метод касательных (метод Ньютона)

Пусть на отрезке [a; b]:

  1. Уравнение f (x) = 0 имеет единственный корень, т.е. f (a) · f (b) < 0;
  2. f ' (х) и f '' (х) сохраняют свои знаки.

 

Заменим дугу кривой y = f (x) на [a; b] касательной, проведённой к графику функции y = f (x) в одной из точек (a; f (a)) и (b; f (b)). Эту точку следует выбирать так, чтобы точка пересечения касательной с осью Ox не вышла за пределы отрезка [a; b]. 
Абсцисса х1 точки пересечения касательной с осью Ox принимается за приближённое значение корня с. Выбрав часть отрезка, содержащую корень, осуществим такое же построение и получим точку х2 и т.д.

http://www.olga.uni-altai.ru/img/f26.jpg

В результате получим последовательность приближённых значений {xn}, монотонно сходящуюся к точному значению корня с. 
При этом корень уравнения f (x) = 0 находится между xi и одним из концов промежутка [a; b] в зависимости от свойств функции y = f (x).

Если f ' (x) · f '' (x) > 0 для любого xhttp://www.olga.uni-altai.ru/img/f23.jpg[a; b], то для вычисления х1, х2, ... , хi, ... используются следующие формулы: 
http://www.olga.uni-altai.ru/img/f27.jpg

Если f ' (x) · f '' (x) < 0 для любого xhttp://www.olga.uni-altai.ru/img/f23.jpg[a; b], то для вычисления х1, х2, ... , хi, ... используются следующие формулы: 
http://www.olga.uni-altai.ru/img/f227.jpg

Процесс уточнения корня заканчивается, когда выполняется условие |хn - xn-1| < E, где E - допустимая погрешность вычисления или когда |f (xn)| < E.

  Комбинированный метод хорд и касательных

Соединяя метод хорд с методом касательных, получаем метод, на каждом шаге которого находим приближённые значения корня с по недостатку и по избытку: xn < c < http://www.olga.uni-altai.ru/img/f28.jpg, причем каждое значение xn и http://www.olga.uni-altai.ru/img/f28.jpg стремится к с.

Если f ' (x) · f '' (x) < 0 для любого xhttp://www.olga.uni-altai.ru/img/f23.jpg[a; b], то для вычисления значений по недостатку и по избытку используются следующие формулы: 
http://www.olga.uni-altai.ru/img/f30.jpg

Если f ' (x) · f '' (x) > 0 для любого xhttp://www.olga.uni-altai.ru/img/f23.jpg[a; b], то для вычисления значений по недостатку и по избытку используются следующие формулы: 
http://www.olga.uni-altai.ru/img/f29.jpg

Процесс уточнения корня заканчивается, когда выполняется условие http://www.olga.uni-altai.ru/img/f31.jpg, где E - допустимая погрешность вычисления. При этом в качестве приближённого значения корня принимается середина промежутка [xn, http://www.olga.uni-altai.ru/img/f28.jpg]:   http://www.olga.uni-altai.ru/img/f32.jpg.

  Метод простой итерации

По функции f (x) строят функцию φ (x) такую, что уравнение x = φ(x) (2) эквивалентно уравнению f (x) = 0 (1). При этом корень c уравнения (1) является корнем уравнения (2).

Затем строят последовательность {xk} по формуле (3) xk = φ (xk-1), k = 1, 2, : начиная с некоторого приближения x0.

Сходимость последовательности {xk} обеспечивается выбором функции φ (x) и выбором начального значения x0. Выбирая различными способами функцию φ, будем получать различные итерационные методы.

Опишем один из способов получения уравнения (2): 
http://www.olga.uni-altai.ru/img/f33.jpg, где р - произвольное число. При этом число k имеет тот же знак, что и производная функции f на отрезке [a; b] и | φ ' (x)| http://www.olga.uni-altai.ru/img/f34.jpg q < 1. 

Пример 1. Методом половинного деления уточнить корень уравнения

f(x) = x4 + 2 x3 - x - 1 = 0

лежащий на отрезке [ 0, 1] .

Последовательно имеем:

f(0) = - 1; f(1) = 1; f(0,5) = 0,06 + 0,25 - 0,5 - 1 = - 1,19;

f(0,75) = 0,32 + 0,84 - 0,75 - 1 = - 0,59;

f(0,875) = 0,59 + 1,34 - 0,88 - 1 = + 0,05;

f(0,8125) = 0,436 + 1,072 - 0,812 - 1 = - 0,304;

f(0,8438) = 0,507 + 1,202 - 0,844 - 1 = - 0,135;

f(0,8594) = 0,546 + 1,270 - 0,859 - 1 = - 0,043 и т. д.

Можно принять

x = http://www.exponenta.ru/educat/systemat/hanova/equation/images/Image1098.gif(0,859 + 0,875) = 0,867

 

Метод хорд

 

В данном методе процесс итераций состоит в том, что в качестве приближений к корню уравнения (1) принимаются значения х1, х2, ..., хn точек пересечения хорды АВ с осью абсцисс (Рисунок 3). Сначала запишем уравнение хорды AB:

http://www.exponenta.ru/educat/systemat/hanova/equation/images/Image1099.gif.

Для точки пересечения хорды AB с осью абсцисс (х = х1, y = 0) получим уравнение:

http://www.exponenta.ru/educat/systemat/hanova/equation/images/Image1100.gif

Пусть для определенности f'' (x) > 0 при а less.gif (65 bytes)  х less.gif (65 bytes)  b (случай f'' (x) < 0 сводится к нашему, если записать уравнение в виде - f(x) = 0). Тогда кривая у = f(x) будет выпукла вниз и, следовательно, расположена ниже своей хорды АВ. Возможны два случая: 1) f(а) > 0 (Рисунок 3, а) и 2)f(b) < 0 (Рисунок 3, б).

http://www.exponenta.ru/educat/systemat/hanova/equation/images/Image1101.gif   image002.jpg (4453 bytes)

Рисунок 3, а, б.

 

В первом случае конец а неподвижен и последовательные приближения: x0 = b;

http://www.exponenta.ru/educat/systemat/hanova/equation/images/Image1102.gif http://www.exponenta.ru/educat/systemat/hanova/equation/images/Image1103.gif

(5)

образуют ограниченную монотонно убывающую последовательность, причем

http://www.exponenta.ru/educat/systemat/hanova/equation/images/Image1104.gif

Во втором случае неподвижен конец b, а последовательные приближения: x0 = а;

http://www.exponenta.ru/educat/systemat/hanova/equation/images/Image1105.gif

(6)

образуют ограниченную монотонно возрастающую последовательность, причем

http://www.exponenta.ru/educat/systemat/hanova/equation/images/Image1106.gif

Обобщая эти результаты, заключаем:

  1. неподвижен тот конец, для которого знак функции f (х) совпадает со знаком ее второй производной f'' (х);
  2. последовательные приближения xn лежат по ту сторону корня x , где функция f (х) имеет знак, противоположный знаку ее второй производной f'' (х).

Итерационный процесс продолжается до тех пор, пока не будет обнаружено, что

x- xi - 1|< e ,

где e - заданная предельная абсолютная погрешность.

Пример 2. Найти положительный корень уравнения

f(x) = x3 - 0,2 x- 0,2 х - 1,2 = 0

с точностью e = 0,01.

Прежде всего, отделяем корень. Так как

f (1) = -0,6 < 0 и f (2) = 5,6 > 0,

то искомый корень x лежит в интервале [1, 2]. Полученный интервал велик, поэтому разделим его пополам. Так как

f (1,5) = 1,425 > 0, то 1< x < 1,5.

Так как f'' (x) = 6 x - 0,4 > 0 при 1 < х < 1,5 и f (1,5) > 0, то воспользуемся формулой (5) для решения поставленной задачи:

http://www.exponenta.ru/educat/systemat/hanova/equation/images/Image1107.gif= 1,15;

|x1 - x0| = 0,15 > e ,

следовательно, продолжаем вычисления;

f (х1) = -0,173;

http://www.exponenta.ru/educat/systemat/hanova/equation/images/Image1108.gif= 1,190;

|x2 - x1| = 0,04 > e ,

f (х2) = -0,036;

http://www.exponenta.ru/educat/systemat/hanova/equation/images/Image1109.gif= 1,198;

|x3 - x2| = 0,008 < e .

Таким образом, можно принять x = 1,198 с точностью e = 0,01.

Заметим, что точный корень уравнения x = 1,2.

 

 

2.Самостоятельное выполнение заданий.

Вариант 1

Вариант 2

1.Методом половинного деления с точностью до 0,01 найти приближённое значение наибольшего действительного корня алгебраического уравнения

3+3х2-9х + 2 = 0

х3 – 3х2 – 7х + 4 = 0

2.Методом хорд с точностью до 0,01 найти приближённое значение наибольшего действительного корня алгебраического уравнения

х3 + 3х2 + 2х + 5 = 0

х3 - 2х2 + 3х – 2 = 0

3.Методом касательных с точностью до 0,01 найти приближённое значение наибольшего действительного корня алгебраического уравнения

х3 + х2  - 3 = 0

3 – 0,9х – 6 = 0

4.Методом итераций с точностью до 0,01 найти приближённое значение наибольшего действительного корня алгебраического уравнения

х3 – х + 1 = 0

х3 –2х - 3 = 0

 

3.Вывод.

 

 

 

 

Практическая работа №23 "Решение простейших задач на определение  вероятности с использованием теоремы сложения вероятностей "

 

Цель работы: закрепление практических навыков решения простейших задач на определение вероятности события.

 Ход работы:

1)повторение теоретического материала;

2)выполнение заданий;

3)вывод.

 

1.Краткое содержание теоретического материала.

           Теория вероятностей – это раздел математики изучающий закономерности массовых случайных событий.

 

           Испытанием называется совокупность условий, при котором может произойти  данное случайное событие.

 

           Событие – это факт, который при осуществлении определенных условий может произойти или нет. События обозначаются большими буквами латинского алфавита А, В,С...

 

           Случайным называется событие, наступление которого нельзя гарантировать.

 

           События бывают достоверные, невозможные и случайные.

 

           Достоверное событие – это событие, которое в результате испытания непременно должно произойти.

 

           Невозможное событие – это событие, которое в результате испытания не может произойти.

 

           Случайное событие – это событие, которое при испытаниях может произойти или не может произойти.

 

 

 

           События называются несовместными, если в результате данного испытания появление одного из них исключает появление другого.

 

           События называются совместными, если в результате данного испытания появление одного из них не исключает появление другого.

 

           События называются равновозможными, если нет оснований считать, что одно из них происходит чаще, чем другое.

 

           События образуют полную группу событий, если в результате испытания обязательно произойдет хотя бы одно из них и любые два из них несовместны.

 

           Два несовместных события А и Ā (читается «не А») называются противоположенными, если в результате испытания одно из них должно обязательно произойти.

 

           Операции над событиями

 

à Суммой нескольких событий называется событие, состоящее в наступлении хотя бы одного из них в результате испытания.

 

à Произведением нескольких событий называется событие, которое состоит в совместном наступлении всех этих событий в результате испытания.

 

           Вероятность события – это число, характеризующее степень возможности появления событий при многократном повторении событий.

 

           Вероятность обозначается буквой Р (probability (англ.) – вероятность).

 

           Классическое определение вероятности: Вероятностью Р(А) ( события А называется отношение числа благоприятствующих исходов m к общему числу равновозможных несовместных исходов n:

                                           Р(А)=m/n                                                                             (1)                                           

           Свойства вероятности:

 

   Вероятность случайного события А находится между 0 и 1.

                                                0<Р(А)<1                                                                          (2)                                                          

 

   Вероятность достоверного события равна 1.

                                  Р(А)=m/n=n/n=1                                                                      (3)

 

   Вероятность возможного события равна 0

                                     Р(А)=m/n=0/n=0                                                                   (4)

Суммой нескольких событий называется событие, состоящее в появлении хотя бы одного из этих событий.

Произведением нескольких событий называется событие, состоя­щее в совместном осуществлении всех этих событий

Теоремы сложения вероятностей

 

   Вероятность суммы двух несовместных событий А и В равна сумме вероятностей этих событий:

                                        Р(А+В)=Р(А)+Р(В).                                                          (5)

                                 

   Вероятность появления хотя бы одного из двух совместных событий равна сумме вероятностей этих событий без вероятности их совместного наступления, т.е.
                                            Р(А+В)=Р(А)+Р(В)-Р(А*В).                                              (6)

 

 

Теорема умножения вероятностей

 

 

   Вероятность произведения 2 независимых событий А и В равна произведению вероятностей этих событий:

                                                   Р(А*В)=Р(А)*Р(В)                                                       (7)

 

Решение задач

 

Пример1  Найти вероятность выпадения числа кратного 3 при одном бросании игрального кубика.


Решение:

          Событие А – выпадение числа кратного 3. Этому событию благоприятствуют два исхода: числа 3 и 6, т.е. m=2. Общее число исходов состоит в выпадении чисел: 1,2,3,4,5,6, т.е. n=6. Очевидно, что эти события равновозможны и образуют полную группу. Тогда искомая вероятность, по определению, равна отношению числа благоприятствующих исходов к числу всех исходов.
Р(А)=
m/n=2/6=1/3.

 

Пример2  В урне 10 белых, 5 красных и 5 зеленых шаров. Найти вероятность того, что вынутый наугад шар будет цветным (не белым).


Решение:
Число исходов, благоприятствующих событию А, равно сумме красных и зеленых шаров:
m=10. Общее число равновозможных несовместных исходов равно общему числу шаров в урне:n=20.Тогда:
Р(А)=
m/n=10/20=0,5

 

Пример 3  Найти вероятность выпадения цифры 2 или 3 при бросании игральной кости

 

 

           

Решение:

   Событие А – выпадение цифры 2, вероятность этого события Р(А)=1/6. Событие В – выпадение цифры 3,  вероятность этого события Р(В)=1/6. События несовместные, поэтому
Р(А+В)=Р(А)+Р(В)=1/6+1/6=2/6=1/3.

 

Пример 4  Получена партия одежды в количестве 40 штук. Из низ 20 комплектов мужской одежды, 6 – женской и14 – детской. Найти вероятность того,  что взятая наугад одежда окажется не женской.

Решение:

       Событие А – одежда мужская, вероятность Р(А)=20/40=1/2
Событие В – одежда женская, Р(В)=6/40=3/20
Событие С –одежда детская, Р(С)=14/40=7/20.
Тогда Р(А+С)=Р(А)+Р(С)=1/2+7/20=17/20.
В этом случае, если события А и В являются совместными, то справедлива следующая теорема.

Пример 5  Вероятность попадания в мишень одного стрелка равна 0,65, а второго 0,6. Определить вероятность поражения мишени при одновременных выстрелах двух стрелков.

Решение:

        Так как при стрельбе возможно попадание в мишень двумя стрелками, то эти события совместные, следовательно

Р(А+В)= Р(А)+Р(В)-Р(А*В)=0,65+0,6-0,39=0,86.

 

Пример6   В урне находятся 10 шаров: 3 белых и 7 черных. Первым был вынут черный шар, найти вероятность того, что второй шар будет черным.

Решение:

         Вероятность появления черного шара первый раз (событие В) равно Р(В)=3/10; а вероятность появления его второй раз (событие А), при условии, что событие В произошло, равно Р(А/В)=2/9, т.к. в урне осталось 9 шаров, из них 2 черных.

Рассмотрим закон умножения  вероятностей для независимых событий.

Произведением двух событий А и В называют событие С=А*В, состоящее в совместном осуществлении этих событий.

 

Пример 7  В билете 3 раздела. Из 40 вопросов первого раздела студент знает 30 вопросов, из 30 вопросов – 15, из 30 вопросов третьего – 10. Определить вероятность правильного ответа студента по билету.

Решение:

         Учитывая, что ответ на каждые разделы есть независимые события А1, А2, и А3,а их вероятности соответственно равны:

Р(А1)=30/40=3/4;       Р(А2)=15/30=1/2;       Р(А3)=10/30=1/3.

Тогда вероятность правильного ответа на билет Р(В), можно найти по формуле

Р(В)= Р(А1)*Р(А2)*Р(А3)=3/4*1/2*1/3=1/8=0,125.

 

2.Самостоятельнле выполнение заданий.

Вариант 1

Вариант 2

1). Из партии, в которой 4 стандартные и 7 бракованных деталей, случайным образом вынимают 4 детали. Найти вероятность, что среди них имеются:

а) 2 стандартные детали;

б) хотя бы 1 деталь стандартная.

1). Из корзины, в которой 8 красных и 5 синих и 3 белых шара, случайным образом вынимают 2 шара. Найти вероятность, что они:

а) оба красные;

б) хотя бы 1 красный.

2)Двум студентам предложена задача. Вероятность того, что её решит 1-й студент равна 0,72, что решит 2-й – 0,65. Найти вероятность того, что задачу решат оба студента; что решит только один?

 

2) Два стрелка независимо друг от друга производят выстрел по мишени. Вероятность попадания 1-м - 

0,8, 2-м – 0,9. Какова вероятность, что после одного выстрела в мишени будет только одна пробоина?

 

3) В пирамиде 7 винтовок, из них 3 с оптическим прицелом. Вероятность поражения цели простой винтовкой 0, 58, а с оптическим прицелом 0, 93. Найти вероятность того, что стрелок поразит цель, стреляя случайно взятой винтовкой.

3) В первом ящике содержится 20 деталей, из них 15 стандартных. Во втором ящике 30 деталей, из них 24 стандартные. А  в третьем – 10 деталей, из них 6 стандартные. Найти вероятность того, что наудачу извлеченная деталь из наудачу взятого ящика – стандартная.

3.Вывод.

 

Практическая работа №24 "Построение закона распределения дискретной случайной величины по заданному условию"

 

Цель работы: закрепление практических навыков решения задач на построение закона распределения дискретной случайной величины по заданному условию.

 Ход работы:

1)повторение теоретического материала;

2)выполнение заданий;

3)вывод.

 

1.Краткое содержание теоретического материала.

 Закон распределения дискретных и непрерывных случайных величин

 

          Случайная величина считается заданной, если известен закон распределения случайной величины.

 

           Законом распределения случайной величины называется всякое соотношение между возможными значениями случайной величины и соответствующими им вероятностями.

 

           Распределение дискретной случайной величины может быть задано в виде таблицы, в графическом и аналитическом виде.

 

           Пусть дискретная величина X принимает значения Х=х1, Х=х2,…, Х=хn. Обозначим вероятности этих событий соответственно: Р(Х = х1) = р1, Р(Х = х2) = р2,…, Р(Х = хn) = рn.

 

            Таблица, содержащая возможные значения случайной величины и соответствующие вероятности, является простейшей формой задания распределения дискретной случайной величины:

 

Значение случайной величины х1

х1

х2

хn

Вероятности значений р1

р1

р2

рn

 

Так как в результате испытания случайная величина Х всегда примет одно из своих возможных значений х1, х2, … хn, то эти случайные события образуют полную группу событий и

                           

                                  р1 + р2 + …+ рn =  = 1.                                                     (1)

                            

 

          Табличную формулу задания называют также рядом распределения. Для наглядности ряд распределения можно представить в графическом виде, где по оси абсцисс откладываются значения случайной величины, а по оси ординат вероятности этих значений.

           В ряде практических случаев вместо вероятности того, что случайная величина Х принимает некоторое определённое значение хi ,необходимо знать, что случайная величина Х меньше хi. Эта вероятность задаётся интегральной функцией распределения.

 

           Функция распределения определяет вероятность того, что случайная величина Х принимает значение, меньшее фиксированного действительного числа х, т.е.

F(x) = P(X < x).

           Функцию распределения F(x) иногда называют интегральной функцией распределения или интегральным законом распределения.

            Функцию F(x) можно получить, суммируя значения вероятностей по тем значениям случайной величины, которые меньше xi, т. е.

 F(xi) = P (X < xi) = ∑ P(xi)              x<xi
где неравенство
x < xi под знаком суммы показывает, что суммирование распространяется на все значения х меньше xi.

             Для дискретной случайной величины график функции распределения представляет собой разрывную ступенчатую функцию. Когда переменная х принимает какое - нибудь из своих возможных значений, функция распределения увеличивается скачкообразно на величину вероятности этого значения. Причём при переходе слева к точкам разрыва функция сохраняет своё значение.

            Функция f(x) называют дифференциальной функцией распределения или плотностью распределения непрерывной случайной величины Х.

 

Геометрический смысл плотности распределения вероятностей f(x) заключается в следующем: зная f(x) можно вычислить вероятность того, что случайная величина примет значение, принадлежащее заданному интервалу (a, b):

                                                                       

                                          P(a < X < b) =  = F(b) – F(a).                                           (2)

 

 

Решение задач

 

 

 Пример 1.  Построить график ряда распределения значений частоты пульса в гипотетической группе из 47 человек по заданной таблице

Значения случайной величины уд/мин

65

66

67

68

69

70

71

72

73

74

75

Значения вероятности р (хi)

2

47

2

47

4

47

 

5

47

 

 

7

47

 

7

47

6

47

4

47

4

47

3

47

3

47

 

Решение:

По данным таблицы построен график, который является  многоугольником распределения вероятностей.

Пример 2. Подбрасываем 1 раз кубик. Пусть X = {1, 2, 3, 4, 5, 6} количество очков, выпавшее при бросании кубика. Записать соответствие между значениями случайных величин x и p  вероятностями принимать эти значения в виде «таблицы распределения вероятностей» или, коротко, «таблицы распределения».

Решение:

 

 

x

1

2

3

4

5

6

Р

1/6

1/6

1/6

1/6

1/6

1/6

 

 

2.Самостоятельнле выполнение заданий.

Вариант 1

 

Вариант 2

 

1. Устройство состоит из трех независимых элементов, работающих в течение времени Т безотказно соответственно с вероятностями 0,97,  0,97,  0,97. Составить закон распределения числа отказавших приборов

1. В квартире шесть электрических лампочек. Вероятность того, что каждая лампочка останется исправной в течение года, равна 5/6. Составить закон распределения числа отказавших лампочек

2. Вероятность появления события A  в каждом из восьми независимых испытаний рана 0,6. Составить закон распределения числа появлений события A

2. Вероятность попадания в цель при одном выстреле равна 0,75. Произведена серия из  четырех выстрелов. Составить закон распределения числа попаданий

 

3.Вывод.

 

 

 

Практическая работа №25 "Нахождение математического ожидания и дисперсии дискретной случайной величины, заданной законом распределения"

 

Цель работы: закрепление практических навыков решения задач на нахождение математического ожидания и дисперсии дискретной случайной величины.

 Ход работы:

1)повторение теоретического материала;

2)выполнение заданий;

3)вывод.

 

1.Краткое содержание теоретического материала.

Числовые характеристики случайных величин

 

         Закон распределения полностью характеризует случайную величину. Но при решении ряда практических задач нет необходимости знать все возможные значения случайной величины и соответствующие им вероятности, а удобнее пользоваться некоторыми количественными показателями, которые в сжатой форме дают достаточную информацию о случайной величине. Такие показатели называются числовыми характеристиками случайной величины. Основными из них являются: математическое ожидание, дисперсия и среднее квадратическое отклонение.

         Математическое ожидание характеризует положение случайной величины на числовой оси, определяя некоторое среднее значение, около которого сосредоточены все возможные значения случайной величины.

          Математическое ожидание дискретной случайной величины равно сумме произведений всех возможных её значений на соответствующие вероятности:

                                                                                                                               

        M(X) = x1p1 + x2p2 + … + xnpn =                                                            (1)                    

                                                                                   

Свойства математического ожидания

 

-   Математическое ожидание постоянной величины равно этой постоянной:

M(C) = C

 

-   Постоянный множитель можно выносить за знак математического ожидания.
                                                              
M (CX) = CM (X)

 

-   Математическое ожидание алгебраической суммы случайных величин равна алгебраической сумме их математических ожиданий:
                                                           
M (X ± Y) = M(X) ± M(Y)

 

          Две случайные величины X и Y называются независимыми, если распределение одной из них не зависит от того, какое значение приняла другая величина.

 

-   Математическое ожидание произведения независимых случайных величин равно произведению их математических ожиданий.

-   M(X ∙ Y) = M(X) ∙ M(Y)

 

-   Математическое ожидание отклонения случайной величины от её математического ожидания всегда равно нулю:

-   M(XM(X)) = 0

 

         Математическое ожидание квадрата отклонения случайной величины Х от её математического ожидания М (Х) называют дисперсией случайной величины Х и обозначают D(X), т.е.

                                                  D(X) = M(XM(X))2                                                   (2)

                                     

         Дисперсия характеризует рассеяние (отклонение) случайной величины относительно математического ожидания.

         Размерность дисперсии равна квадрату случайной величины и её неудобно использовать для характеристики разброса, поэтому удобнее применять корень квадратный из дисперсии – среднее квадратическое отклонение. Эта величина даёт представлять о размахе колебаний случайной величины около математического ожидания.

 

                                                  σ (сигма) = √D(X)                                                      (3)

 

      Основные свойства дисперсии:

 

-   Дисперсия алгебраической суммы двух независимых случайных величин X и Y равна сумме дисперсий этих величин, т.е.

D(X ± Y) = D(X) ± D(Y)

 

-   Дисперсия постоянной величины С равна нулю, т.е.

D(C)=0.

 

-   Постоянный множитель С случайной величины Х можно выносить за знак дисперсии, предварительно возведя его в квадрат, т.е.

D(CX) = C2 D(X)

 

-   Дисперсия случайной величины Х равна разности между математическим ожиданием квадрата случайной величины и квадратом её математического ожидания, т.е.

                                                          D(X) = M(X2) – [M(X)]2                                                  (4)

 

Решение задач

Пример1.  Найти математическое ожидание дискретной случайной величины Х, зная закон её распределения.

Х

-1

0

1

2

3

р

0,05

0,2

0,4

0,3

0,05

 

Решение:

По формуле находим:

M(X) = -1 ∙ 0,05 + 0 ∙ 0,2 + 1 ∙ 0,4 + 2 ∙ 0,3 + 3 ∙ 0,05 = 11

 

 

Пример 2.   Случайная величина задана следующим рядом распределения.

X

-1

0

1

2

p

0,1

0,3

0,4

0,2

Найти математическое ожидание и дисперсию этой величины.

 

Решение:

Для нахождения математического ожидания воспользуемся формулой, а для дисперсии результаты вычисления сведём в таблицу

x

pi

xipi

xi – M(x)

(xi – M(X))2

(xi – M(X))2pi

-1

0

1

2

0,1

0,3

0,4

0,2

-0,1

0

0,4

0,4

-1,7

-0,7

0,3

1,3

2,89

0,49

0,09

1,69

0,289

0,147

0,036

0,338

1

0,7

 

 

0,81

 

Из таблицы следует, что  M(X) = 0,7; D(X) = 0.81

2.Самостоятельнле выполнение заданий.

Вариант 1

Вариант 2

1.Случайная величина X задана законом распределения:

2

3

10

0,1

0,4

0,5

Найти математическое ожидание М(X), дисперсию D(X) и среднее квадратичное отклонение σ(X).

1.Случайная величина X задана законом распределения:

0,1

2

10

20

0,4

0,2

0,15

0,25

Найти математическое ожидание М(X), дисперсию D(X) и среднее квадратичное отклонение σ(X).

2.Случайная величина X задана законом распределения:

-1

1

2

3

0,48

0,01

0,09

0,42

Найти математическое ожидание М(X), дисперсию D(X) и среднее квадратичное отклонение σ(X).

2.Случайная величина X задана законом распределения:

-1

1

2

3

0,19

0,51

0,25

0,05

Найти математическое ожидание М(X), дисперсию D(X) и среднее квадратичное отклонение σ(X).

 

3.Вывод.

 

 

 

 

 

 

 

 

 

Просмотрено: 0%
Просмотрено: 0%
Скачать материал
Скачать материал "Методические указания для студентов по проведению практических занятий по дисциплине ЕН.01. «Математика» для специальности 230701 «Прикладная информатика (по отраслям)» базовой подготовки"

Методические разработки к Вашему уроку:

Получите новую специальность за 3 месяца

Заведующий хозяйством

Получите профессию

Технолог-калькулятор общественного питания

за 6 месяцев

Пройти курс

Рабочие листы
к вашим урокам

Скачать

Краткое описание документа:

Сборник практических работ служит для организации практических занятий по математике в объеме 50 часов. Данное пособие предназначено  для студентов 2 курса специальности «Прикладная информатика (по отраслям)» и разработано в соответствии с рабочей программой по математике.

 

        Целью практических занятий является формирование учебных практических умений по математике и содействие оптимальному освоению  студентами учебного материала. Выполнение студентами практических работ направлено на обобщение, систематизацию, углубление, закрепление полученных знаний по конкретным темам, формирование умений применять полученные знания на практике, формирование профессионально значимых качеств таких,  как самостоятельность, ответственность, точность.

Скачать материал

Найдите материал к любому уроку, указав свой предмет (категорию), класс, учебник и тему:

6 662 863 материала в базе

Скачать материал

Другие материалы

План-конспект урока по математике на тему "Простые и составные числа" 5 класс по учебнику Дорофеева Г.В. и др.
  • Учебник: «Математика», Дорофеев Г.В., Шарыгин И.Ф., Суворова С.Б. и др. / Под ред. Дорофеева Г.В., Шарыгина И.Ф.
  • Тема: 6.2. Простые и составные числа
  • 01.10.2020
  • 1244
  • 21
«Математика», Дорофеев Г.В., Шарыгин И.Ф., Суворова С.Б. и др. / Под ред. Дорофеева Г.В., Шарыгина И.Ф.

Вам будут интересны эти курсы:

Оставьте свой комментарий

Авторизуйтесь, чтобы задавать вопросы.

  • Скачать материал
    • 02.01.2015 869
    • DOCX 1.7 мбайт
    • 12 скачиваний
    • Оцените материал:
  • Настоящий материал опубликован пользователем Грищенко Зинаида Николаевна. Инфоурок является информационным посредником и предоставляет пользователям возможность размещать на сайте методические материалы. Всю ответственность за опубликованные материалы, содержащиеся в них сведения, а также за соблюдение авторских прав несут пользователи, загрузившие материал на сайт

    Если Вы считаете, что материал нарушает авторские права либо по каким-то другим причинам должен быть удален с сайта, Вы можете оставить жалобу на материал.

    Удалить материал
  • Автор материала

    Грищенко Зинаида Николаевна
    Грищенко Зинаида Николаевна
    • На сайте: 9 лет и 3 месяца
    • Подписчики: 1
    • Всего просмотров: 6107
    • Всего материалов: 10

Ваша скидка на курсы

40%
Скидка для нового слушателя. Войдите на сайт, чтобы применить скидку к любому курсу
Курсы со скидкой

Курс профессиональной переподготовки

Интернет-маркетолог

Интернет-маркетолог

500/1000 ч.

Подать заявку О курсе

Курс повышения квалификации

Методика обучения математике в основной и средней школе в условиях реализации ФГОС ОО

72 ч. — 180 ч.

от 2200 руб. от 1100 руб.
Подать заявку О курсе
  • Сейчас обучается 432 человека из 74 регионов
  • Этот курс уже прошли 5 549 человек

Курс повышения квалификации

Педагогическое проектирование как средство оптимизации труда учителя математики в условиях ФГОС второго поколения

36/72 ч.

от 1700 руб. от 850 руб.
Подать заявку О курсе
  • Сейчас обучается 85 человек из 35 регионов
  • Этот курс уже прошли 1 415 человек

Курс повышения квалификации

Особенности подготовки к сдаче ЕГЭ по математике в условиях реализации ФГОС СОО

36 ч. — 180 ч.

от 1700 руб. от 850 руб.
Подать заявку О курсе
  • Сейчас обучается 188 человек из 55 регионов
  • Этот курс уже прошли 1 699 человек

Мини-курс

Неорганическая химия

8 ч.

1180 руб. 590 руб.
Подать заявку О курсе
  • Сейчас обучается 20 человек из 14 регионов

Мини-курс

Художественная гимнастика: диагностика и технические аспекты

3 ч.

780 руб. 390 руб.
Подать заявку О курсе

Мини-курс

Дизайн интерьера: от спектра услуг до эффективного управления временем

3 ч.

780 руб. 390 руб.
Подать заявку О курсе
  • Сейчас обучается 20 человек из 14 регионов